Preview only show first 10 pages with watermark. For full document please download

Career Launcher Solutions Book 2

   EMBED

  • Rating

  • Date

    December 1969
  • Size

    5.9MB
  • Views

    1,313
  • Categories


Share

Transcript

Contents MBA Test Prep 1. Explanations: Fundamentals of Logical Reasoning and Data Interpretation 1 2. Explanations: Fundamentals of Ratio & Proportion 71 3. Explanations: Fundamentals of Time, Speed & Distance 93 4. Explanations: Fundamentals of Grammar 119 Explanations: Fundamentals of Logical Reasoning & Data Interpretation Data Interpretation 150 − 45 × 100 = 233.3% 45 3. b Growth rate of HLL = 4. e Sale of P & G in 2002 = 25% of 150. Sale of P & G in 2003 = 20% of 375. Practice exercise – A1 From the pie charts, the sales figures in rupees (in crores) are: Ratio of sales in 2002 : 2003 = Company 2002 2003 HLL 45 150 P&G 37.5 75 Henkel 22.5 75 Nirma 30 56.25 Others 15 18.75 Above table has been made to explain the things. You need not make it. Also, in pie-charts of this sorts, it is very advantageous to identify the ratio and percentage increase of total size of pie in the year after year. In this case the ratio of the total size of market is 6 : 15 i.e. 2 : 5 1. b 375 − 150 × 100 = 150% 150 Alternately, since we know the ratio of the pies is 2 : 5, one just needs to do the following 25% of 2 : 20% of 5 i.e. 50 : 100 5. a Detergent market grows by 10% of 150 = Rs. 15 crore. 6. a It is equivalent to that of the whole market = 10%. 7. a 10% annual growth for 2 years means 21% growth ab ). 100 Sales in 2005 = 1.21 × 375 = Rs. 453.75 crore. (using the formula, a + b + 8. d Since one would already have found the ratio of the pies, before starting, as 2 : 5, one could also Growth in sales of P & G = (0.2 × 375 – 0.25 × 150) = Rs. 37.5 crore. HLL = (0.4 × 375 – 0.25 × 150) = Rs.112.5 crore. Henkel = (0.2 × 375 – 0.15 × 150) = Rs.52.5 crore. Others = (0.05 × 375 – 0.1 × 150) = Rs.3.75 crore. Costs of HLL in 2002 = 30 × 1.1 = Rs. 33 crore. Profit = 45 – 33 = Rs. 12 crore. Profit percentage = 5−2 3 = = 150% have mentally just calculated 2 2 2. e 9. a Explanations: Fundamentals of Logical Reasoning & Data Interpretation 12 × 100 = 36.4% 33 Margin = Pr ofit 12 × 100 = × 100 = 26.67% Sales 45 Henkel = 75 − 22.5 × 100 = 233.3% 22.5 P&G= Alternative method: Since growth in total sales has been 150% from 2002 to 2003. So we can change the pie chart of 2003 as HLL = 40 × 2.5 = 100% P & G = 20 × 2.5 = 50% Henkel = 20 × 2.5 = 50% Nirma = 15 × 2.5 = 37.5% Other = 5 × 2.5 = 12.5% These percentages are of market size in 2002, i.e. Rs. 150 crore. Now, visually, one can figure out the minimum growth in sales is in others. 0.25 × 150 1 = . 0.2 × 375 2 75 − 37.5 × 100 = 100% 37.5 ⇒ Difference = 133.33% Alternative method: If we keep base for 2002 and 2003, then (As shown in the solution of question 2) Henkel = 50 − 15 7 = × 100 = 233% 15 3 P&G= 50 − 25 = 100% 25 MBA Test Prep Page: 1 10. c The ratio will not change because if the total sales are doubled, the sales of Nirma will also double for both the years. 56.25 15 = . 30 8 Again one could have used the ratio of pie 2 : 5 effectively and answer would have been 15% of 5 : 20% of 2 i.e. 75 : 40 i.e. 15 : 8 Hence, the ratio will be 11. c Surf Excel = 30% of HLL. HLL = 30% of detergent market. Surf Excel = 30% of (30% of detergent market) = 9% of detergent market. 12. c Profit = 25%. Hence, margin = 20%. Profit amount = 20% of sales = 0.2 × 22.5 = Rs. 4.5 crore. 13. a Since all the companies have the same expenditure, the company with the maximum sales will have the highest profit as well as profit percentage. 14. d Sale Cost = × 100 100 + Pr ofit percentage 150 × 15% 22.5 = 18 crore. = 1.25 1.25 Henkel’s cost in 2003 = 19.8. Henkel’s sale in 2003 = 75. Cost of Henkel in 2002 = Profit percentage = 15. c change in any year with respect to the base year. An index cannot give the actual value for any year unless the actual value and the index of any one year is given to us. e.g. if, for the given graph, actual sales in 1994 (sales index 120) is Rs. 144 crore, then we can say that: Sales index of 120 = Sales of 144 crore.  144  Sales index of 1 = Sales of   = Rs. 1.2 crore.  120  Sales index of 100 = Sales of Rs. 120 crore. Thus, we now have the conversion factors, i.e. to convert sales index to sales : Multiply index with 1.2 and to convert sales into sales index : Divide sales by 1.2. Thus, now we can determine the index of sales for any year if the sales (rupees in crores) is known and also the sales (rupees in crores) of any year if the sales index is known. In the solutions below the abbreviations being used are: SV = Sales value CV = Cost value PV = Profit value 1. e Index does not give the actual value. The profit index is 110. 2. a Sales in 1993 = Rs. 500 crore = Index of 100. In 1998, SI = 154, which is 54% above the index of 1993. Thus, sales of 1998 should be 54% more than the sales in 1993. Thus, sales in 1998 = 500 × 1.54 = Rs. 770 crore OR Index of 100 = Sales of Rs. 500 crore. Index of 1 = Sales of Rs. 5 crore. Index of 154 = Sales of 154 × 5 = Rs. 770 crore. 3. c In 1996, CI = 110 and CV = Rs. 550. 55 × 100 = 278%. 19.8 S − 0.8S × 100 = 25% . 0.8S If both sales and costs increase by 10%, then sales will be 1.1S and cost will be 1.1(0.8S) and hence the profit percentage will remain at 25%. P & G’s profit in 2003 = In 2002, CI = 190. Thus, CV = Practice exercise – A2 Index is a representation of actual value. A year is designated as the base year and the index for the base year is taken as 100, e.g. in the given graph for sales, cost and profit, the base year is 1993 (index = 100). For 1994, the sales index is 120, which means that sales in 1994 is 20% more than the sales in 1993 and sales in 1995 (index 131) is 31% more than in the base year (1993) sales. Similarly, the cost index of 98 in 1994 indicates that the cost has gone down by 2% over the base year of 1993 and in 1996 the cost has gone up by 10% over cost in 1993. (Index = 110) SI = Sales index CI = Cost index PI = Profit index 550 × 190 = Rs. 950 . 110 4. e It is not possible to find the value across indices. With one value of cost we can find out all the values of cost but none of the values of sales or profits. 5. e Gross profit = (Sales – Cost), since we do not have SV and CV for various years, the gross profit cannot be calculated. 6. b In 2003, PI = 130 and PV = Rs. 780. 780 × 110 = Rs. 660 . In 2001, PI = 110. Thus, PV = 130 Index is used to compare data over a large number of years and to determine trends. Because the base year value is always taken as 100, it is easy to determine the percentage Page: 2 MBA Test Prep Solution Book-2 7. a In 1993, SI = CI = 100,SV = Rs. 300 and CV = Rs.120. Thus, SV in 1998 = 154 × 3 = Rs. 462 and CV in 1998 = 153 × 1.2 = Rs. 183.6. Gross profit = 462 – 183.6 = Rs. 278.4. In the given question, we cannot determine the profit because it is defined as (Sales – Cost – Tax) and we do not have the tax for 1998. 8. b In 1993, SI = CI = 100,SV = Rs. 300 and CV = Rs.120. Thus, SV in 2003 = 188 × 3 = Rs. 564 and CV in 2003 = 166 × 1.2 = Rs. 199.2. Gross profit = 564 – 199.2 = Rs. 364.8. 9. c In 1993, SI = 100 and SV = Rs. 700. Thus, SV in 1998 = 154 × 7 = Rs. 1078. In 1996, CI = 110 and CV = 600. 600 = 834.54. 110 Gross profit = 1078 – 834.54 = Rs. 243.45. Practice exercise – A3 1. e We need the total number of students opting for finance in three institutes to arrive at the average. 2. c Average salary at IIML (60 × 550000) + (40 × 420000) + (50 × 725000) 150 = 574000. Rather than multiplying by 60,40 and 50 you can also multiply by 6,4,5 and then divide by 6 + 4 + 5 = 15. = 3. a Calculating as above, average salary at IIMK = 75900. Average salary of IIMK – IIML = 759000 – 574000 = 185000. 4. b Average salary at IIMB Thus, CV in 1998 = 153 × 10. b  100  Thus, new SI in 1993 =   × 100 = 83.33.  120  11. a 12. b Only 3 years — 2000, 2001 and 2002 — have their CI more than the CI for 2003. So for these years, CI will be more than 100. 13. b Total Sales Value for the period 1993-98 = 1200 + (1.2 × 1200) + (1.31 × 1200) + (1.52 × 1200) + (1.62 × 1200) + (1.54 × 1200) = Rs. 9828 1200 or (Sum of SI from 1993 to 1998) × . 100 Sales Value for the period 1993-98 = 9828. (Refer question 13) Total Cost Value for the period 1993-98 800 = (100 + 98 + 95 + 110 + 121 + 153) × 100 = 5416. Difference = 9828 – 5416 = Rs. 4412. 15. c 5. d The average salary for finance in IIMB is significantly lower than that for systems. Since the number in finance has gone up, the average salary will be lower than what we calculated for question 4. Thus, the answer is 626. (Check by calculation) 6. c Now since the number in systems has increased, the average salary will be more than the average salary in question 5 by a large margin. So the answer is 667. 7. c Average salary for finance in IIMA, IIMB and IIMC Current PI in 1996 = 125 and PI in 2003 = 130. New PI in 1996 = 100.  100  Thus, new PI in 2003 =   × 130 = 104.  125  14. a (50 × 540000) + (45 × 453000) + (110 × 775000) 205 = 647000. = Current SI in 1993 = 100 and SI in 1994 = 120. New SI in 1994 = 120. 1537 = 139.72 11 Average of Sales Index – Average of Cost Index = 147.27 – 139.72 = 7.54 Average of Cost Index = Explanations: Fundamentals of Logical Reasoning & Data Interpretation (540000 × 50) + (520000 × 90) + (725000 × 45) 185 = 624000 = 8. b Average salary of the remaining students of systems = (500 × 850000 – 1000000 × 200) 300 = 750000. Instead of the above calculation, one could also have done the following mentally : Of 5 students with average salary 850000, 2 are in dotcom companies with average 1000000. Thus they contribute 300000 more than overall average and thus on an average each of the 3 non-dotcom students would have a salary of (850000 – 100000) = 750000. MBA Test Prep Page: 3 9. b Average salary of IIMA finance students placed in (1 × 620 – 0.6 × 800) = 350. 0.4 Alternately, as explained in the oral calculations for solutions to 17, in this question 60% could have been taken as the fraction 3/5 and thus 5 students have average of 620. Of these 3 have average of 800 and thus contribute a total of 540 more to the sum than the average. Thus the remaining 2 students on an average has 620 – 270 = 350 6. a Rephrase the question as, “Which crop has the highest yield over the years?” Answer is barley. 7. b Percentage increase in production of maize in 2000-01 = (9.99 – 8.06) × 100/8.06 = 24%. Production in 2002-03 = 9.99 × (1.24)2 = 15.36 Mn T. Yield = Production/Area. Thus, yield = 15.36 × 1000/6.1 = 2518 kg/Ha. India = 10. d Alternative method: We can approximate, though some risk is involved. But whenever you approximate, you need to understand how much error you are introducing and whether your actual answer would be more than or less than the calculated answer. Percentage increase in production of maize in Rather than taking number as 150 and 50, to reduce calculations, we can take their ratio 3 : 1. Hence, average = (3 × 453 + 1 × 575) = 483. 4 10 − 8 × 100 = 25%. 8 Production in 2002-03 = 10 × (1.25)2 = 10 × 1.5625 = 15.62 Mn T. But actual answer would be less than this. Let us say → 15.30 Mn T 2000-01 = Practice exercise –A4 1. a Land used for paddy in 2001-02 = 12.71 Mn Ha. Total land used in 2001-02 = 29.93 Mn Ha. Percentage of land used for paddy = or 2. c 12.71 × 100 = 42.5% 29.93 13 × 100 ~ − 43.3% approximately. 30 Productivity = Production/Area = Yield. Yield of ragi in 2002-03 = 1327 kg/Ha. Yield of ragi in 2001-02 = 1378 kg/Ha. Percentage change in yield 1378 − 1327 × 100 ; 3.72% 1378 Calculation technique: One needs to calculate 51/1348. 10% of 1378 is 137.8 and hence 5% is 68.9. Thus 51 will be less than 5% and only one option is left. Now yield = 8. c It can be visually figured out that the year would be 2000-01 therefore total production would be 35.52 Mn T. 9. a In the year 2000-01, the production should be 2.53 + 2.58 i.e. the production should be a little more than double the original production. Thus yield also should be a little more than double of the given yield i.e. a little more than 2 × 1329 = 2658. Only one option is in that range. 10. d Production of paddy in 2001-02 = 1602 × 12.71 = 20.36 Mn Ha. Production of paddy in 2000-01 = 12.81 Mn T. Increase in production = 3. b 4. c 5. d Barley. We do not need to solve it. Mere inspection reveals the answer. Production = Area × Yield. If yield increases by 10% and the area remains the same, then the production will also go up by 10%. Hence production in 2003-04 = Production in 2002-03 × 1.1 = 2.43 × 1.1 = 2.43 + 0.243 = 2.67 Mn T. Production = Area × Yield. Production = 521 × 12.71/1000 = 6.62 Mn T. 1 . Alternatively, we know 12.5% = 8 15.3 × 1000 = 2,508 kg/ma. 6.1 = 20.36 − 12.81 × 100 = 59% . 12.81 Alternatively, we can approximate as well. Production of paddy in 2001-02 = 1600 × 12.71 100 1 = 20 Mn Ha as = 12.5 8 8 × 100 Production of paddy in 2000-01 = 12.81 Mn Ha ~ 12.5 Mn Ha. = 1600 × Increase in production = 20 − 12.5 × 100 ~ 60%. 12.5 1× 100 = 6.5 Mn T. 8 × 1000 It would be higher than 6.5 Mn T as 12.717 > 12.5 So 521 × Page: 4 MBA Test Prep Solution Book-2 11. c 12. e 13. c In such questions use options. Start with an intelligent choice of option e.g. as maize appears in 3 of the four choices, check trend for maize. As trend for maize does not satisfy the required condition, safely mark the fourth option i.e. (c). The given data pertains to production and not sales. We cannot assume that 100% of paddy produced, was sold. Thus, data is inadequate to solve the given problem. Cost of production of barley = Rs. 1,250 per quintal. If there is no loss of produce, then selling price for 25% profit = 1250 × 1.25 = Rs. 1562.5. Loss in produce is 10%. Thus, for every 100 kg produced, only 90 kg can be sold. So for 25% profit, SP = (1562.5/0.9) = Rs. 1,736 per quintal. 14. b By rule of alligation, ratio of quantity available to PDS to open market = 2 : 5. Therefore, 2/7th or 28.57% is allocated for PDS. 15. d Yield required = 1518 kg/Ha. Amount of production = 8.06 Mn T. 8.06 × 1000 = 5.31 Mn Ha. 1518 Land should be less by (5.86 – 5.31) = 0.55 Mn Ha. 5. c The number accounted for by Hyundai and Daewoo = (10 + 25)% of 4 lakh = 35% of 4 lakh = 1.4 lakh. 6. a Number of Santros = 75% of 25% of 4 lakh = 75000. 7. d Number of Santros = 75000 (from question 6). Number of Zen = 20% of 40% of 4 lakh = 32000. Difference between Santro and Zen sales = 43000. 8. c Sales of Maruti 800 in Mumbai = 60% of 40% of 5% of total India sales = 0.012 × 24 lakh = 28800. 9. b Sales in 2004 = 1.15 × 4 lakh = 4.6 lakh. Thus, HM sales = 15% of 4.6 lakh = 69000. 10. c Maruti sales in 2003 = 40% of 4 lakh = 1.6 lakh. Increase in Maruti sale in 2004 = 20% of 1.6 lakh = 0.32 lakh. Thus, Maruti sales in = 1.6 + 0.32 = 1.92 lakh and total sales = 4 + 0.32 = 4.32 lakh. Percentage share of Maruti = (1.92/4.32) × 100 = 44.44%. 11. d Hyundai’s sales in 2003 = 25% of 4 lakh = 1 lakh. Hyundai’s sales in 2004 = 90% of 1 lakh = 0.9 lakh. Total sales in 2004 = 130% of 4 lakhs = 5.2 lakh. Hyundai’s share of sales = (0.9/5.2) × 100 = 17.3%. 12. a Total sales in 2004 = 5.2 lakh. (Refer question 11) Daewoo’s sales in 2003 = 10% of 4 lakh = 0.4 lakh. Daewoo’s share in 2004 = (0.4/5.2) × 100 = 7.7%. 13. b Sales of Maruti in 2003 = 40% of 4 lakh = 1.6 lakh. Increase in sales of Esteem in 2004 = 40000. Thus, sales of Maruti in 2004 = 2 lakhs. Total sales in 2004 = 1.2 × 4 lakh = 4.8 lakh. Percentage share of Maruti in 2004 = (2/4.8) × 100 = 41.67%. 14. c Sales of Esteem in 2003 = 16000. Sales of Esteem in 2004 = 40000 + 16000 = 56000. Sales of Maruti in 2004 = 2 lakh. (Refer question 13) Esteem’s share in Maruti sales = (0.56/2) × 100 = 28%. 15. d Sales of Opel in 2003 = 25% of 10% of 4 lakh = 10000. Thus, land required = Practice exercise – A5 1. b Number of Esteems = 10% of 40% of 2.5 lakh = 4% of 2.5 lakh = 0.1 lakh. 2. c 60% of 40% of total car sales = 48000, i.e. 24% of total sales = 48000. Thus, total sales = 48000/0.24 = 2 lakh. 3. a HM market share = 15%. Zen market share = 20% of 40% = 8%. Therefore, difference = (15 – 8)% = 7%. = 7% of 2.5 lakh = 17500. 4. c Number of Maruti cars sold = 40% of 2 lakh = 80000. Sale of Maruti 800 = 48000. Increase of Maruti 800 sold = 25% of 48000 = 12000. New total sales of Maruti = 92000 = 0.92 lakh. New total car sales = 200000 + 12000 = 2.12 lakh. 0.92 × 100 = 43.4% . 2.12 Thus, percentage points by which Maruti share has increased = 43.4 – 40 = 3.4 points. Percentage share of Maruti = Explanations: Fundamentals of Logical Reasoning & Data Interpretation MBA Test Prep Page: 5 Practice exercise – A6 7. b We are making table for convenience sake. You can avoid making it. The following figures, based on the graph, have been used for the calculations. Year Shareholders wealth Total debt Net fixed assets Net working capital 1999 125 410 180 350 2000 280 350 250 375 2001 475 430 335 550 2002 750 675 550 745 2003 1150 800 760 100 SW growth rate in 2003 = 745 – 550 = 35.45%. 550 1150 – 750 = 53.33%. 750 Thus, NWC is less than SW by 53.33 –35.45 3.33 = 33%. Alternatively, you can approximate, since choices are far apart. NWC growth rate in 2002 ≈ 750 − 550 200 4 = = 550 550 11 1  = 36.36%  = 9.09% 11  Following abbreviations have been used. SW = Shareholder’s wealth NFA = Net fixed assets NWC growth rate in 2002 = SW growth rate in 2003 ≈ TD = Total debt NWC = Net working capital 1150 − 750 400 8 = = 750 750 15 = 53%. 1. a 2. d Average of the total debt is 556. Hence, the total debt is above the average for 2 years (2002 and 2003). NWC in 1999 = 350, NWC in 2003 = 1000. If NWC in 1999 = 100, then NWC in 2003 = 3. d 1000 × 100 = 285.6 350 NFA growth rate in 2000 = NWC is less than SW by = 8. e Growth rate of NWC is less than that of SW for all the years. 9. c Growth rate of: NFA in 2000-2002 = 120% and TD in 2000-2002 = 92%. SW in 1999-2000 = 124% and SW in 2002-2003 = 53%. TD in 2001-2002 = 56% and NWC in 2001-2002 = 35%. TD in 2002-2003 = 23% and SW 2001-2002 = 58%. 10. a While doing this visually, the only doubt is whether for the year 2001 the NWC is above average or not. Assume the average as the value in 2001 and use the method of deviation. If sum of deviation is +, average is above the value of 2001 or else other way round. In this case the sum of deviation would be –200 – 175 + 225 + 450 which will obviously be positive. Thus average will be greater than that for year 2001. Hence, (a) is correct. 250 – 180 = 38.9%. 180 550 – 335 = 64.2%. 335 Hence, the difference is 25.3%. NFA growth rate in 2002 = 4. c Total growth = 1150 – 125 = 1025. 1025 = 256. 4 Thus, SW in 2004 = SW in 2003 + 256 = 1150 + 256 = 1406. Thus, average annual growth = 5. e 53 − 36 17 1 = ≈ = 33.3%. 53 53 3 Average annual growth rate 1150 –125 × 100 = 205% 125 (Refer question 4) Growth rate is less than 205% for all the choices. 6. d The word compounded has no relevance and the item which has grown the maximum will have the highest CAGR and average annual growth rate. So shareholder’s wealth has grown the maximum. Page: 6 MBA Test Prep Solution Book-2 Practice exercise – A7 8. c If initially the amount of electricity = 1, then at the end of period amount of electricity = 1.085 × 1.08 × 1.04 × 1.065 × 1.063 = 1.3796 or electricity will grow by 37.96% If growth rate of electricity in 2000-01 is replaced with growth rate of mining 1999-2000 (9.5%), then electricity = 1.085 × 1.08 × 1.095 × 1.065 × 1.063 = 1.4526 or electricity will grow by 45.26% Hence, difference = 45.26 – 37.96 = 7.3% 9. d The highest total production would be in 2001-02, because in the last year 2002-03, there is a negative growth and thus is less than the figure in 2001-02. 10. c The highest growth rate for the various years are: 1998-99-10%, 1999-2000-14%, 2000-01-7.5% 2001-02-6.5% and 2002-03-6.3% Thus, figure at the end of the period = 1.1 × 1.14 × 1.075 × 1.065 × 1.063 = 1.526 Hence, overall growth = 52.6% For questions 1 to 10: Again we are making the table, but you can avoid that. The following figures, based on the graph, have been used for the calculations. The table gives the percentage change over the previous year or the growth rates. Year General Mining 1998-99 8.5 7 Mfg. Elect. 8.5 8.5 FD GDP 10 4.2 1999-2000 13 9.5 14 8 8 5 2000-01 5.5 -2 6.5 4 7.5 4 2001-02 6.5 6 6.5 6.5 5.5 6 2002-03 4 -2 4.5 6.3 5.8 5 1. b Growth rate of manufacturing in 2001-02 = 6.5%. Thus, growth in manufacturing in 2001-02 = 6.5% of 80 = 5.2 Mining data is redundant over here. 2. d Average of GDP growth rate = 3. b 4. e 5. c 4.2 + 5 + 4 + 6 + 5.5 = 5% 5 Add the growth rate of 5 years for all the items, this sum will give the relative ranking of the growth rates. This sum for mining and electricity is less than that of fiscal deficit. In question of this type where one has to eliminate using options, choose which sector to start with very intelligently. Choose that one which appears in two options and thus with just one calculations, atleast you can shortlist your choice to two options. Practice exercise – A8 We have made the table for convenience sake, but you should not make it. Let production of : A in the various years be given by A2000, A01, A02 and A03 B in the various years be given by B2000, B01, B02 and B03 C in the various years be given by C2000, C01, C02 and C03 D in the various years be given by D2000, D01, D02 and D03 The following table, based on the graph, gives the values of the percentage share of production for A, B, C and D for the four years Year A B C D The chart gives us the percentage change from the previous year. With the GDP given, Fiscal deficit can not be calculated because the data does not give the relation between GDP and fiscal deficit. 2000 23% 37% 20% 20% 2001 23% 35% 22% 20% 2002 20% 30% 30% 20% Add the percentage change for the four items from 1999-2000 to 2002-03, the item which has the highest sum will end up with the highest figure. 2003 10% 30% 20% 40% 6. e Cannot be determined because the data gives us the percentage change only and we do not have the actual values for any of the years. 7. b GDP 1999-2000 = 3,09,000 × 1.05 = Rs. 3,24,450 crore Fiscal deficit 1999-2000 = 11,200 × 1.10 × 1.08 = Rs. 13,305 crore Fiscal deficit as percentage of GDP in 1999-2000 = 13305 13 = = 4.1% 324 324450 Explanations: Fundamentals of Logical Reasoning & Data Interpretation Let total production in 2001 = p, thus total production in 2003 = 1.21p Given C03 – A01 = 1320 thus 0.2 x 1.21p – 0.23p = 1320 thus p = 110,000 MT, and total production in MT in 2000 = 1,00,000 2001 = 1,10,000 2002 = 1,21,000 2003 = 1,33,100 1. a C2000 = 20% of 1,00,000 = 20,000 MT. C02 = 30% of 1,21,000 = 36,300 MT Thus, 36300 = 20000(1 + r /100)2 ⇒ r = 35% One could also have approximated it as follows: The production increases from 20k to 36k i.e. a increase of 80%. Thus CAGR has to be less than 40% and once could check with 35% MBA Test Prep Page: 7 2. e Given data pertains to production and not sales. 3. a Product B 4. d Cumulative production of B will still be the highest because the ratio of production for the various products for the various years is constant. 5. e Cannot be determined because the price of A is not known for the four years. 6. c The total revenue from selling B and D = 3k × 150 + 2k × 120 = 690k. Thus total cost in producing B and D 690k = 657k 1.05 Cost in producing D = 2k × 100 = 200k. = {Q cos t of one unit = 20% × Total03 – 23% × Total01 = 1320 20% × 0.95 × 1.1 × Total01 – 23% × Total01 = 1320 20% × 1.045 × Total01 – 23% × Total01 = 1320 This gives a negative production figure for 2001 and hence data is inconsistent. 10. e Practice exercise – A9 1. b Percentage change in the average price for 2000 = 50% 2001 = 16.66% 2002 = 14.28% 2003 = 37.5% One does not need to calculate this. One can just check it visually. The changes are 1, 0.5, 0.5, –1.5 and the base are 2, 3, 3.5, 4.5. The year where change is maximum and base is least will have the highest growth rate. 2. b Production of Maa-roti in 2000 = 30,000 and Production of Maa-roti in 2003 = 60,000 Average annual growth rate for 2000-2003 Rs.120 = Rs.100 } 1.20 ⇒ Cost in producing B = 457k 457 = 152.3 3 2.3 ×100 = 1.53% ⇒ Thus loss % on selling B = 150 ⇒ CP of 1 unit of B = 7. b Remember that in this question you cannot use the total production in 2003 as 133,100 because this was arrived at when there is a 10% increase in production in each year. So all calculations have to be done again for this question. Total production of B in 4 years = 37000 + 38500 + 36300 + 39930 = 1,50,630 Total sale of B in 4 years = 37000 + 38500 + 36300 + 39930 × 0.5 = 1,31765 Sales Value of B = 131765 × 400 = Rs. 5,27,6000 Total cost of B = 100 + 150630 × 200 = Rs. 30126100 Profit = 74.9% = 60,000 − 30,000 1 × 100 × = 33.3% 30,000 3 C03 – A01 = 1320 20% × Total03 – 23% × Total01 = 1320 3. b 40 × 2 + 30 × 3 + 50 × 3.5 + 40 × 4 + 60 × 2.5 220 = Rs. 2.97 lakh 20% × 1.5625 × Total01 – 23% × Total01 = 1320 = (0.3125 – 0.23) × Total01 = 1320 Total01 = 16,000 Total production in 2003 = 16,000 × 1.5625 = 25000 Thus required difference = 10% of 25000 = 2500 8. c 9. e All this problems asks is that what is the value of x% such that -5% and x% successively amounts to 21%. Thus 0.95 × k = 1.21 k = 1.2736 Thus growth in 2002 should be 27.36% Though data of growth rate given originally is changed, but since this question has nothing to do with actual production amounts and we do not have to use 1320 MT, the question is easy. This is very similar to question number 7. The original data of 10% increase every year is changed and hence we cannot use the production figures found earlier. With the new growth rates we have, C03 – A01 = 1320 Page: 8 Average price per car 4. e Cannot be determined because total cars produced by the car industry is not known for any year. 5. b Production of Maa-roti in 2001 = 50,000 Percentage share of Maa-roti = 50,000 × 100 200,000 = 25% 6. a Production 2001 = Production 1996 × 1.08 × 1.15, Production in 1999 = 200000 = 161030 1.08 × 1.15 7. e The assumption that production = sales is not valid unless it is specifically stated. 8. a Cars sold in 2000 = 80% of 30,000 = 24,000 Price per car in 2000 = Rs. 3 lakh Revenue in 2000 = 24,000 × 3 lakh = 72,000 lakh MBA Test Prep Solution Book-2 9. c 10. c Revenue in 2000 = 72,000 lakh (Refer question 8) In 2001, the entire production of 2001 and 20% 2000 production is sold = 56,000 Revenue in 2001 = 56000 × 3.5 = 196000 lakh Revenue 2000 + 2001 = 2,68,000 lakh Revenue for each year is given below: 1999 : 40 × 2 = Rs. 80,000 lakh 2000 : 30 × 3 = Rs. 90,000 lakh 2001 : 50 × 3.5 = Rs.1,75,000 lakh 2002 : 40 × 4 = Rs.1,60,000 lakh 2003 : 60 × 2.5 = Rs.1,50,000 lakh Thus, in 2001 Maa-roti has the highest growth. 11. a 2001. Calculate from question 10. 12. e Again, we cannot assume that production = sales. 13. c Average price per car for Maa-roti = Rs. 4 lakh Average price per car of car industry = Rs. 4.5 lakh Market share of Maa-roti = 62.5% Let average price of rest of the car industry = p, then 0.625 × 4 + 0.375 p = 4.5 Thus, p = Rs. 5.33 lakh Recollect that in case of averages, oral calculations are far better as follows : 62.5% is 5/8 and thus 8 items have an average price of 4.5 and 5 of them have an average of 4. Thus these 5 were “given” a total of 0.5 × 5 = 2.5 by rest 3 and thus rest 3 will have an average of 4.5 + 14. e FC = Fixed cost SP = Selling price 2.5 = 4.5 + 0.83 = 5.33 3 VC = Variable cost/unit N = Number of units sold Break Even Point = BEP is the production level at which the company is in a no profit or no loss situation. BEP = FC / (SP – VC), since the question does not give us all the required data, we cannot determine the BEP. 15. c 16. b 17. e 18. c 40,000 cars are produced in 1999 If 10% gets rejected, net production = 36,000 Turnover = 36,000 × 2 lakh = 72,000 lakh = 720 crore Number of Maa-roti Omni produced in 2001 = 15% of 50,000 = 7500 Since we do not have the average price per car for Maa-roti 800 for the two years, we cannot determine the ratio of sales revenue. 19. b Maa-roti Esteem Production 2001 = 20% of 50,000 = 10,000 Production 2002 = 22% of 40,000 = 8,800 Growth rate = 20. a 8800 − 10000 × 100 = −12% 10000 Refer to question 10 for data, simple annual growth rate = 150000 − 80000 1 × 100 × = 21.8% 80000 4 21. a Refer to question 9 for data. For CAGR apply formula for CI, A = P (1 + r/100)n A = 1,50,000 lakh, P = 80,000 lakh and n = 4 time periods, 150 = 80 (1 + r/100)4 thus r = 17% 22. e Only the average price per car is given. The number of cars produced is not given thus the average price per car for 2001-2004 cannot be determined. 23. a In 1999 Maa-roti produced 40,000 cars, which is 20% more than in 1998, thus production in 1998 = 40000 = 33333 1.2 Practice Exercise – A10 The following figures, based on the graph, have been used for the calculations. Abbreviations used are : Int. = Interest Depn. = Depreciation NP = Net Profit OI = Other Income Year OPBDIT Int. D ep n . NP Tax OI 1999 110 30 5 75 0 1 2000 285 80 20 165 20 2 2001 395 80 50 205 60 4 2002 520 110 80 285 85 10 2003 380 145 120 10 105 6 1. d Operating profit after interest and taxes but before depreciation = Net profit + Depreciation, this is highest for 2002. 2. c Refer to the table above, tax and depreciation have witnessed a constant growth across the years. Ratio of Maa-roti Gypsy produced in 2002 to 2001 5% of 40,000 200 = 10% of 50,000 = 500 = 2 : 5 Explanations: Fundamentals of Logical Reasoning & Data Interpretation MBA Test Prep Page: 9 3. b 4. b NP has grown from 75 in 1999 to 165 in 2000, hence percentage increase in NP for 1999-2000 = 120%. OPBDIT has grown from 395 in 2001 to 560 in 2002, hence percentage increase in OPBDIT for 2001-2002 = 41.8% Thus, difference = 120 – 41.8 = 78.2%. This set is DI type reasoning set, where the reasoning is primarily used in solving individual questions. 1. d Total bill amount: (150 × 1 + 50 × 3.5 + 25 × 7 + 14 × 5 + 15 × 2) + 99 = 699 2. b Total bill amount: (150 × 1+ 50 × 3.5 + 25 × 7 + 14 × 5 + 15 × 2) + 99 + 12% on (25 × 7) = 720 3. a Since, in STD calls it’s not mentioned that the call has been made to which operator, (like: Airtel, WLL etc.) one may get tempted to mark option (d). But upon solving, and considering all the possible cases, we get Interest grows from 30 in 1999 to 145 in 2003, thus simple annual growth rate  145 – 30  1 =  × = 95.8% 30   4 5. e Practice Exercise – A11 For CAGR, use the formula for compound interest 145 = 30(1 + r/100)4 thus r = 48.3% Alternative method: Total growth from 1999 to 2003 is 400%. Now, let us take a convenient value, say 50% (4 year) Use a + b + 50 + 50 + Rohan spent: On STD: 18 × 1.5(Airtel, GSM) = Rs 27 Or 18 × 2 (WLL) = Rs 36 Local: 60% of 120 = 72 min (because Airtel and GSM have same rates) @ 1 Re. min. ∴ Amount spent = 72 × 1 = Rs 72 40% of 120 = 48 min @ Rs. 2/min. ∴ Amount spent = 48 × 2 = Rs 96 Total Bill = (72 + 96 + 27 or 36) = Rs 195 or Rs 204 ab 100 50 × 50 = 125 100 125 × 125 ~ 400% 100 So CAGR is closer to 50%, hence the answer is 48%. 125 + 125 + 6. b The company would have been placed in the MAT category in 1999 because its tax for the year is zero. 7. a OPBDIT 2002 = 520 and OPBDIT 2003 = 380 OPBDIT 2003 as a percentage of OPBDIT 2002 = 380 = 73.1% 520 8.b NP and OPBDIT increase till 2002, after which both of them decrease. 9. a Interest in 1999 = 30, then interest in 2003 = 145, thus if interest in 1999 = 100, then interest in 2003 100 = 483 = 145 × 30 10. c The maximum depreciation was in 2003, hence maximum assets were acquired in 2002. Page: 10 Similarly for Mohan: On STD: 30 × 1.5 = Rs 45 or 30 × 2 = 60 Local: 70 × 1 = Rs 70 30 × 2 = Rs 60 Minimum and Maximum amounts that Mohan would have spent are Rs.175 and Rs.1907. The maximum possible difference is (Rs.204 – Rs.125) = Rs.59. 4. a Normal tariff for 40 SMS/month would have been = 40 × 1.5 = Rs 60 With SMS scheme: 40 × 0.60 + 35 = Rs.59 In fact, in order to benefit from the scheme, one needs to send a minimum of 39 SMS. So, option (a) is the correct choice. 5. d Option (a): 30 × 1.5 + 55 + 99 (Price of Plan) = Rs 199 (assuming local calls are made @ Re. 1) Option (b): 16 × 1.5 + 76 + 99 (Price of Plan) = Rs 199 (assuming local calls are made @ Re. 1) Option (c): 10 × 1.5 + 10 × 2.5 + 10 × 2.5 = Rs 65 Now local calls (since not mentioned) may be distributed like 5 × 2 (WLL/ Landline) + 25 × 1 (GSM/ Airtel) = Rs. 35 Total 65 + 35 + 99 (Price of Plan) = Rs 199 Therefore, bill amount of Rs 199 per month is possible in each of the three options (a), (b) and (c). MBA Test Prep Solution Book-2 ∴ Option (d) Total bill = 8 × 1.5 + 4 × 3.0 + 7 × 3.5 + (55 × 1 or 55 × 2) + 99 = Rs.202.5 or Rs.257.5 Hence (d) is correct. 6. a Checking options Option (a): 8 ISD calls can be distributed as: 4 calls to Rest of the world and another 4 calls to Gulf. Bill amount = 4 × 40 + 4 × 10 + 99 = Rs 299 Option (b): Checking with the maximum rate of both GSM and WLL, maximum possible bill amount = 35 × 3 + 10 × 3.5 + 99 = Rs 239 7. b Ear lie r Ne w Diffe r e nce A irtel 1.5 1 0.5 GSM/CDMA 1.5 1 0.5 2 1 1 WLL/Landline Therefore, to cover the additional charge of Rs.20/month  20  =  40 calls to Airtel/GSM One has to make   0.5   20  =  20 calls to WLL/Landline. and   1  Hence, in Option (a): No profit Option (b): profit = (35 × 0.5 + 10 × 1) – 20 = Rs. 7.50 Option (c): profit = (50 × 0.5) – 20 = Rs. 5 In order to have no loss from the plan, user have to balance out additional charge of Rs. 250 by making calls. For to be definitely sure, we will take that existing STD rate which will give us minimum difference when compared with new scheme ∴ STD of Airtel = Rs. 1.5/min Difference = 1.5 – 1 = 0.50/min 250 = 500 0.5 Minimum 500 calls has to be made such that I am definitely sure of having no loss from opting for this plan. ∴ Minimum calls = 9. b 10. b Rates in Delhi-Mathura segment: Outgoing calls to 8. c (b) Difference in the rates: In STD = 1.5 – 0.5 = Rs 1/min Amount saved = 1 × 14 = Rs 14 In local = 1.0 – 0.5 = 0.5/min Amount saved = 0.5 × 18 = Rs 9 Total amount saved = 9 + 14 = Rs 23 Additional charge paid = 5 × 5 = Rs 25 ∴ Loss = Rs (25 – 23) = Rs 2 Option (b) is the correct choice. Rest of the options need not be checked. Checking options: (a) Difference in the rates In STD = 1.5 – 0.5 = Rs 1/min Amount saved = 1 × 12 = Rs 12 In local = 1.0 – 0.5 = Rs 0.5/min Amount saved = 0.5 × 8 = Rs 4 Total saved amount = 12 + 4 = Rs 16 Additional Charge levied @ Rs. 5/day for 3 days = Rs 15 ∴ One can gain Re. 1 in this calling pattern. Explanations: Fundamentals of Logical Reasoning & Data Interpretation Difference in amount = 1 (Airtel local) – 0.5 = Rs 0.5/ call is saved Additional Rent paid = Rs. 5 daily To balance the additional charge, minimum no. of 5 = 10 calls 0.5 ∴ In order to GAIN one should make atleast 10 + 1 = 11 calls calls that should be made = 11. e This was a tricky one. There is no need to calculate anything. Absolute profit will always remain the same. Profit will always be : [100 × 1 (free calls) + 100 × 1.5 (free SMS) – Rs. 200] = Rs. 250 – Rs. 200 = Rs. 50 Note: Has it been said which will have “maximum profit percentage”, then one has to calculate. Be careful while reading the question. 12. a This one is also a tricky question. Total ISD calls made 12 min/month and 80% of the bill came from the Rest of the world. Let, calls to USA, Canada, Europe (Fixed Lines) be of ‘a’ minutes duration, calls to GULF, Europe (Mobile), SAARC be of ‘b’ minutes duration and calls to Rest of the World be of ‘c’ minutes duration. ⇒ 7a + 10b + 40 c = Total Bill (in Rs.) Or 7a + 10b = 20% of (7a + 10b + 40c) 1 (7a + 10b + 40c) 5 ⇒ 4(7a + 10b) = 40c ...(i) and also, a + b + c = 12 (total duration) ...(ii) Solving equations (i) & (ii), we get only one set as possible solution. (0, 6, 6) i.e. a = 0 minute b = 6 minutes c = 6 minutes ∴ calls to USA, Canada, Europe (Fixed Lines) were for minimum duration. 7a + 10b = MBA Test Prep Page: 11 For questions 13 and 14: The outgoing call and SMS charges are reduced to half. 13. b 4. c Since, 20% drop in the total bill was because of drop in the rates by 50%. 20 0.5 = 40% of the call-time during the Happy Hours. ⇒ He made 60% of the calls during the day and 40% during Happy Hours ∴ In order to get 20% drop, he needs to spend Ratio = 14. e The fact that the question exists means that only 1 of the 4 options can be right. Check each one till we find a team which, based on its score after 5 games, played can still reach the 2nd round. After tabulating the points, we can see that only Benfica has a chance to qualify for the next round. Other teams given in the options will never qualify for the next round. Sure-shot Approach: Ideally the points for the groups to which each option pertains should be calculated and noted for future use. (1 at a time: and not all at the start). People trying this question mentally stand a good chance at getting it wrong. 60 3 = 40 2 Since, details of local calls made and SMS sent is not available, percentage of local calls made during the day cannot be determined. Common Speed Breaker: Checking all 4 options inspite of having found the right option, just to be doubly sure, often costs students extra time in the exam. Be sure of yourself! Practice Exercise – A12 The set can be solved in two ways: A. Adding an extra column for points in Set A and tabulating all points before proceeding. This approach will involve repeating this step for atleast 60% of the teams again after question 6. This approach, though time consuming, is likely to give guaranteed results. For anyone who finds this set tough to conceptualize, this may be the preferred approach B. Take each question as and when it comes and tackle it on merit. 5. b Easy, the team which has the most money at this stage, which is the team which has won all the games it has played - Arsenal. This question is a rare example of where scanning the data rather than reviewing the options would give a faster solution. 6. c Add results / points for new instructions given and then use options. Need to be done only for the 4 options, not for entire data set. Anderlecht is yet to score a point so far in the tournament and playing “away” in the last match of the 1st round will result in a loss for them, thus earning ‘zero’ point from the competition. 7. a All home terms win their matches in their last first round matches. Lyon , Inter Milan, Barcelona, Liverpool and Arsenal have 16, 12, 13, 11 and 18 points respectively. 8. e Cannot be determined because the scores from the last set of matches of round 1 are not available 9. c Can be counted and is equal to 77. Short-cut: The total number of games won in the last round was 16 and this was an exception as there were no draws in this set, hence, the total number of game won in all 6 rounds has to be < 6*16 = 96. This rules out Option (d). Option (a) is less than 16 (last set wins) and has to be wrong. Simple observation shows that 38 cannot be the answer, so it has to be 77 10. c Review the options. Option (a): Even after losing the last match of their group, Juventas and Bayern Munich, with 12 points each will reach the next round, leaving behind club Brugge with 9 points. For questions 1 to 15: 1. c (32 teams) × (6 Games Played) / 2 = 96 (it takes two teams to play a match) matches will be played in round 1, 8 matches in round 2, 4 in Quarter-finals, 2 in Semi-finals & 1 in the final Total = 96 + 8 + 4 + 2 + 1 = 111 Short-cut: Answer has to be > 96 eliminating options a & b instantly Common-error – counting each match twice forgetting that two teams are playing and hence not dividing by 2 2. e Will be either 3 or 4 depending on whether the second round was a ‘home’ match or not. 3. b All teams play atleast 3 away matches in the first round and another 3 away matches in the quarterfinal, semi-final and final. Remember, ‘neutral’ venue is also an away match for any team. Page: 12 MBA Test Prep Solution Book-2 Option (b): Rangers by virtue of winning the last match of the group, will end with 9 points, Thus will qualify from group H as a ‘runners-up’ behind Inter Milan. Option (c): Lille, after losing the last match of the group (because Lille is playing away in the last match) will end up with only 6 points. So, from group D, Benfica and Villareal will quality with 8 and 10 points respectively. So, option (c) is the right answer choice. 11 a 12. c 13. c 14. a 15. d From the table given in the explanation of the previous question, it is obvious that Juventas is the only surviving team among the options given in this question. So option (a) is the correct answer. As Arsenal has won the most number of games (6) till this stage they are ahead of all teams on the money earners list. Juventus have won 5 & have 0 draws, Liverpool have won 4 & have 2 draws and are hence also behind. Rangers with 2 wins and 3 draw’s is not in the reckoning. The key in this question is that the actual values of amount earned need not be calculated. Number of students in XI Arts = 720 (Given), thus 0.24N + 0.048N = 0.288N = 720, thus N = 2500 We will have the following: LPS Outside Total 2000 400 2400 400 80 480 XI commerce 1000 200 1200 XI arts 600 120 720 Total standard X 2500 Total standard XI XI science 1. b As N = 2500. 2. a 2000 (solved above) 3. c (0.4 + 0.08) × 2500 = 1200 4. b Percentage of arts students in LJC standard XI = 5. c 480 (solved above) 6. a Number of science students in XI = 480 Number of science students in XI from LPS = 400 Percentage of science students in LJC standard XI Again doesn’t need to be calculated, the two teams records (wins, draws & losses) need to be compared and are found to be identical Options (c) & (d) are out as Arsenal has been knocked out as seen in the last question, leaving Juventus as an automatic choice and just a check required between Liverpool & Rangers. Since all ‘Draws’ took place in 1st round, comparing the number of draws by Liverpool and Rangers, we get Rangers had 3 draws, while Liverpool had 2 draws. So option (a) is the correct choice. We have no data on quarter-final results, hence this question only checks on the teams which have definitely been knocked out and hence cannot be in the final. AC Milan & Lyon have both been knocked out, making this an easy choice. who are from LPS = Explanations: Fundamentals of Logical Reasoning & Data Interpretation 400 = 83.33% 480 7. e We do not know what percentage of the girls took arts though we can find out the total number of girls. 8. c 10% of commerce stream = 10% of 1200 = 120 5% of the arts stream = 5% of 720 = 36, thus number of students in XII arts = 720 + 120 – 36 = 804 9. a 5% of arts stream = 5% of 720 = 36 6% of commerce stream = 6% of 1200 = 72, thus number of students in XII LJC = 2400 – 36 – 72 = 2292 10. c 8% of commerce stream = 8% of 1200 = 96, 10% of science stream = 10% of 480 = 48, thus change in number of commerce students = –48 Percentage change in commerce students = –48/ 1200 = –4% 11. e Though we know the ratio, we do not have the ratio of number of boys and girls in any stream or in total. 12. b Number of girls in science = 50% of boys in science Thus, girls form 33.33% of science = 480/3 = 160 Thus, number of girls in arts = 3/2 × 160 = 240 because the ratios Arts : Science is 3 : 2. Practice exercise – A13 If N is the number of LPS students who took standard X examination, then the number of students who passed the examination is 0.8 N. Number who joined the various streams: Science = 0.2 × 0.8N = 0.16N Arts = 0.3 × 0.8N = 0.24N Commerce = 0.5 × 0.8N = 0.4N Number of students joining from outside = 0.2 × 0.8N = 0.16N Ratio of these students in = Science : Arts : Commerce = 2 : 3 : 5, thus number of these students in Science = 0.2 × 0.16N = 0.032N Arts = 0.3 × 0.16N = 0.048N Commerce = 0.5 × 0.16N = 0.08N 720 = 30% 2400 MBA Test Prep Page: 13 13. b Number that failed in XI class = 10% 2400 = 240 From V, we get 14. b Number of boys in science stream = 320 (from question 12) Number of boys in arts stream = 720 – 240 = 480 Thus, ratio = 320 : 480 = 2 : 3 Trunk Box 25 pounds 5 pounds 20 pounds __ American __ ... (3) From II, we get 15. b Total number of students in standard XI after introduction of home science = 2400 + 300 = 2700 Percentage of home science students in standard XI = 300 / 2700 = 11.11% Trunk Box 25 pounds 5 pounds 20 pounds __ 10 pounds American ... (4) Logically, the left place in (4) will be occupied by 15 pounds belonging to Indian as per IV Logical Reasoning Practice Exercise – B1 For questions 1 to 5: Note that from statements I, IV and VI we can make out that Chandra lives in New Delhi. Now from V, we can make out that Aparna is not the dancer; and from II, since Chandra lives is New Delhi, she is also not the dancer. Therefore, Bharti is the dancer and lives in Kolkata. Also from VIII, Bharti is married to Ramsingh. Now since Chandra lives in New Delhi and Bharti in Kolkata, it is obvious that Aparna lives in Mumbai, and from IX, she (Aparna) is married to Mansingh. Hence, the result is as follows. Kolkata New Delhi Mumbai Sisters Bharti (Dancer) Chandra Aparna Brothers Ram Singh Bhim Singh Man Singh 1. b Trunk Box 25 pounds 5 pounds 20 pounds 15 pounds 10 pounds American Indian ... (5) From III, Swede can occupy the second spot from left only Trunk Box 25 pounds 5 pounds 20 pounds 15 pounds 10 pounds ... (6) Swede American Indian From VII, we get Trunk Box Crate/Suitcase Crate/Suitcase Carton 25 pounds 5 pounds 20 pounds 15 pounds 10 pounds German Swede American Indian Belgian Here 10 pounds weight will be owned by Belgian because it is the one which is not owned by German as it is a carton. Therefore, owned by Belgian and consequently 25 pounds will be owned by German. 6. c 2. a 7. d 3. a 8. e 4. c 9. b 5. a 10. e For questions 6 to 10: According to the question from VI, we get 20 pounds __ __ __ __ American ... (1) From VIII, we get Trunk 25 pounds Page: 14 __ 20 pounds __ American __ ... (2) MBA Test Prep Solution Book-2 For questions 11 to 16: According to the question, we have Persons Campsites Lakes States A E I M B F J N C G K O D H L P If we represent the data in the following way, we get –, H , J, N From (ii) (1) For questions 17 and 18: As given that the names of brothers and sisters do not begin with the same letter and Pinku and Gaurav are not Saroj or Sangeeta’s brothers, Pinku cannot be the brother of Pooja and hence he is the brother of Rakhi. Now we have that Gaurav cannot be the brother of Saroj, Sangeeta or Rakhi. Therefore, Gaurav is the brother of Pooja. As given that Saroj is not Ratan’s sister and Rakhi and Pooja can also not be the sister’s of Ratan (from above conclusions), Ratan is the brother of Sangeeta. Anil will have to be the brother of Saroj as this is the only valid combination left. Therefore, we have this table finally. B, –, K, – From (i) (2) (4) From (v) A, –, –, O Sister Pinku Rakhi Gaurav Pooja Ratan Sangeeta Anil Saroj From (iv) (3) D , F, –, – We have I camping on P ... from (iii) ... (3) Now, from (3), I and P have to be together. These I and P cannot be with (1), (2) or (4) as they are already occupied. Therefore, I and P will be there with (3). So we have –H J N 17. d 18. b 19. a A Brother O B K Relative speed of approach of the cyclists = (15 + 15) = 30 miles per hour. 30 Therefore, they will meet after = 1 hr. 30 Since the fly has travelled throughout this 1 hr (regardless of direction) at a speed of 20 miles per hour, total distance travelled by it is (1 × 20) = 20 miles. D F I P Logically, as all the states are occupied except M, M will come along with B and K. Therefore, we have the final arrangement depicted by the table given below. Person A B Campsite C D H F L a ke L K J I State O M N P 20. b 49 = 7 cigarettes. 7 ∴ The duration of time he will take to smoke these 3 hr = 5.25 hr (i.e. 5 hr and 7 cigarettes = 7 × 4 15 min). Now note that after he has smoked these 7 cigarettes, he will collect 7 more stubs (one from each), from which he will be able to make another 3 cigarette. This will take him another hr (45 min) 4 to smoke. Therefore, total time taken = 6 hr. He has got = And on the basis of this table we can solve rest of the questions very easily. 11. b 12. a 13. d 14. a 15. d Explanations: Fundamentals of Logical Reasoning & Data Interpretation 16. b MBA Test Prep Page: 15 For questions 9 to 13: On the basis of the given information, we arrive at the following sitting plan that does not violate any of the given conditions. Practice Exercise – B2 For questions 1 to 3: J Note: Jayesh is to the right of Alok, i.e. J, A. Pramod is between Bhagat and Subodh, i.e. B, P, S. Subodh is between Jayesh and Pramod. So the sequence is: Bhagat Pramod Subodh 1. a Alok is at the extreme left end. 2. d Subodh is in the middle. 3. a Statement I is superfluous. Jayesh Alok From III, we get _ _ _ _ _ E or B B E _ _ _ _ _ From IV, we get _ _ _ F G E B From V, we get the final sequence as E G F _ _ _ B E G F D C A H K E And on the basis of the above figure rest of the questions are solved as follows: 9. e K is seated between E and H. 10. c Three persons H, L and J or G, I and E are seated between K and F. 11. b The three lady members are E, H and G. 12. c J is to the immediate left of F. 13. a Clearly, J is a male member. For questions 14 and 15: According to the given question from II, we get B or F G I For questions 4 to 8: According to the question from II, we get D C L H isto ry C ivics ... (1) From III, we get ... (1) Now on the basis of the sequence given in (1), we can solve rest of the questions very easily. G eog raph y E ng lish ... (2) 4. e 5. a 6. c 7. d 8. e Page: 16 MBA Test Prep Solution Book-2 From IV (1) and (2), we get On the basis of the above table, rest of the questions can be solved very easily. 16. c 17. a 18. d 19. b 20. d H isto ry Practice exercise – B3 C ivics For questions 1 and 2: Check out the numbers that have four factors and try to understand, e.g. 6 – 1, 2, 3, 6. Note that in case of 1, 2, 4, 8, x = 2 (not 4), which is prime. Note that x and y will always be prime. Therefore, x . y = N Therefore, x . x . y = x . N G eo gra ph y E n glish ... (3) E con om ics Since history and civics cannot be at any other place than this, according to the given conditions. On the basis of this very arrangement, rest of the questions can be solved very easily. 14. c 15. d 1. a 2. d For questions 3 to 5: r Clearly, C gives us the clue that the science book is placed at the bottom. Thus, we know that there are three books between the civics and science books. 9 8 Clearly, history, civics and geography are the three books kept above the English book. To deduce this, no additional information is required. r 8 9 r–9 r r–8 For questions 16 to 20: From the given information in the question: From II, we get Dr Choudhary teaches zoology in Mumbai University. From III, we get Dr Natrajan is neither in Osmania nor in Delhi University. Therefore, he will be either at Mumbai or Gujarat University. Similarly, as he teaches neither geology nor history, therefore, he must be teaching physics or botany. ... (1) From IV, Dr Zia → Physics but as he is not teaching in either Mumbai or Osmania University, he must be teaching either in Delhi or Gujarat University ... (2) r 9–r From V, we get Dr Joshi teaches history in Delhi University. ... (3) From (1) and (2), we conclude that Dr Natarajan teaches botany. 8 University Subject Dr Joshi Delhi History Dr Davar Osmania Geology Dr Natrajan Gujarat Botany Dr Choudhary Mumbai Zoology Dr Zia Gujarat Physics Explanations: Fundamentals of Logical Reasoning & Data Interpretation r 8–r And from (1), (2) and VI, we get both Natarajan and Zia teach in Gujarat University. Finally, on summarisation we can prepare the following table. Names r 9 (r – 8)2 + (r – 9)2 = r2 ∴ r = 29, 5 3. b 4. c MBA Test Prep 5. b Page: 17 For questions 6 and 7: 1000009 = 293 × 3413 You could get the factors using options. Practice exercise – B4 For questions 1 to 4: The given information can be tabulated as follows. 6. e 7. a Department For questions 8 to 11: The prime numbers less than 45 are as follows: 2, 3, 5, 7, 11, 13, 17, 19, 23, 29, 31, 37, 41, 43. So there are 14 prime numbers. Now, since the average of the goals scored is also a prime number, for question (10), we can readily eliminate choice (a). Now the sum of all the prime numbers listed above is 281. So the sum of total goals scored must be less than 281. Note that only choice (b), i.e. 23 gives a total number of goals of (23 × 11) = 253 which is less than 281. Hence, average is 23. Thus, 23 is not the number of goals scored by any player. Of the remaining 13 numbers, whose sum is (281 – 23) = 258, we have to delete two numbers, such that the sum of the other 11 numbers is 253. That means the sum of those two numbers, (which are to be deleted) is 5. Obviously, those two numbers must be 2 and 3. 8. b 12. e 13. b 9. a 10. a Now, since we know that the number of compartments is less than the number of soldiers in each compartment, there have to be 359 compartments. For questions 14 and 15: Obviously, the maximum number of such points possible would be 3, where the points are the vertices of an equilateral triangle. If the points do not have to lie in a plane, then we can have four such points, where they form the vertices of a trapezoid. 14. d 15. b For questions 16 to 18: Let the length be x, and the breadth y. Since area = Perimeter, xy = 2(x + y). The only values of x and y which satisfy this equation are (x, y) = (4, 4) and (x, y) = (3, 6). Therefore, there are two such rectangles, and only one such square. Therefore, if the rectangle is not a square, its dimensions will be 6 by 3. Page: 18 17. a Male Mechanical Electrical Female Male Female Male Female Professor A E H, I L O, P S Lecturer B, C, D F, G J, K M, N Q, R T Further, following constraints are added. (i) (ii) Male lecturer in Civil Department × ∴ (B, C, D) × T… (Not allowed) Female professor × Male lecturer ∴ (E, L, S) × 11. b 222221 = 619 * 359. However, we do not know which is the number of compartments, and which is the number of soldiers in each compartment. Therefore, the answer cannot be determined due to insufficient data. 16. b Civil Female lecturer in Electrical Department B, C, D, J, K, Q, R (Not allowed) As given that BCD cannot be with T and ELS cannot be with BCDJKQR, then using these conditions we can solve rest of the questions very easily. (1) Choice (a) is not valid as no lecturer from electrical department is there. Choice (b) is not valid as no lecturer from civil department is there. Choice (c) is not valid as no lecturer from mechanical department is there. Therefore, choice (d) is the only valid choice as it justifies all the given conditions. (2) According to the question. HIL cannot go with BCDFG. ... (1) The delegation must have minimum three professors. ... (2) Representation from every department must also be there. ... (3) Checking from the above conditions, we get (a) is not valid as it violates (1). (b) is not valid as A, H, D are males. (c) is again not valid as it violates (1). (d) obviously, it is the valid choice because it fulfills all the conditions. (3) According to the question, only civil and electrical departments must be used. ..(1) Committee must have two professors and two lecturers. ...(2) 18. b MBA Test Prep Solution Book-2 Explanations: Fundamentals of Logical Reasoning & Data Interpretation MBA Test Prep Page: 19 For questions 7 to 9: Simple! You are being asked about the product of which two consecutive integers is equal to product of three consecutive integers. You have two such sets. (i) 1 × 2 × 3 = 2 × 3 (This is not possible in this case.) (ii) 5 × 6 × 7 = 14 × 15 For questions 17 and 18: Doctor cannot be a woman. (As the two women, Sujith’s mother and his wife are not blood relatives.) Son cannot be the doctor because he is the youngest of them and all people are his blood relatives. ⇒ Sujith is the doctor. Lawyer can be anyone except Sujith and his mother (Sujith’s mother also because of condition I). 7. e 17. a 10. d 18. c 8. c 9. e 2r For questions 19 and 20: The joker can be with Ajay or Sujith, since Sanjay cannot have it. Three queens can only be with Sanjay, as 3 queens plus a minimum of 1 king add up to a minimum of 4 cards. 19. d 2r 2r 2r 20. b Practice Exercise – B5 For questions 1 and 2: Here, the best method to solve these questions is going by choices. In (1) choice (b) is only valid choice as it is between 70 and 79 and is not a multiple of 6, whereas other choices are not valid as they do not follow the given criteria. In (2) obviously, none of the information is superfluous. Therefore, choice (e) is the answer. 1. b 2. e From III and IV, we get that Puneet arrived at the mansion after midnight and because the detective arrived at the mansion at midnight, therefore, Puneet is not the detective ... (i) Now the detective can be either Aditya or Vijay. From II, we get that the detective came at the second number (then only II is justified) . From III and V, we can conclude that Vijay was the earlier arriver between Puneet and Vijay, therefore, he is not the detective. Similarly, from (i) Puneet is not the detective. Therefore, logically, it is Aditya who is the detective and arrived at the mansion at the second number. ... (ii) From IV, I and VI, we conclude that Vijay came first. Aditya (the detective) came second and Puneet came third. Now from VI Puneet was not the murderer and from I and IV it was Aditya, the detective, who was the murderer . ... (iii) Using (i), (ii) and (ii) rest of the questions can be solved very easily. 4. b Page: 20 For questions 11 to 13: We have P + Q + R + S = 45 1 and (P + 2) + (Q – 2) + 2R + S = 45 2 2(P + Q) = R + S …(i) …(ii) ... (iii) Solving the equations, we have their ages as follows. For questions 3 to 6: According to the given conditions in the question. 3. c In fact you will get a hexagon joining the centres of all the external circles. 5. c 6. a P Q R S ? ? 10 20 11. b 12. c 13. a For questions 14 to 18: If a room has an odd number of visitors, it will be closed. Any room number that is twice a perfect square will have an odd number of visitors. The room with the largest such number (twice a perfect square) will be the last room to have an odd number of visitors. Note that the 38th room with an open door will be the 38th room whose number is not twice a perfect square, which happens to be 88. Note that if only occupants of 2 to 1000 do the task, then only 1 person, i.e. the occupant of number 1000 will go to room number 2000, (since 2000 is a multiple of 1000), and this room will therefore be closed. (Since it is initially open.) Number of room closed upto 1000 = 22 Since there are 22 twice a perfect square numbers. Number of remaining room = 500 Number of twice a perfect number between 1000 and 2000 = 9. All rooms with twice a perfect square between 1000 and 2000 will be open. MBA Test Prep Solution Book-2 ∴ Number of room closed = 22 + 500 – 9 = 513 As regards question 18, anyone who lives in a room with a number greater than 1000 will obviously have to visit only that particular room, as it will not have any multiple which is not greater than 2000. 14. c 19. e 20. e 15. b 16. a 17. a For question 6 to 10: The best method to solve this question is to make a table and fill the places according to the information given logically. On analysing the information given in the question, we arrive at the following table. 18. b Student Optional Subject The only such number below 200 is 153. M Geography (given) English (given) Therefore, Q is 5. N Geography (given) Biology (given) You have two such numbers: 370 or 371. Therefore, R may be 0 or 1. O Geography (given) Physics (given) P English (given) Geography (given) Female student (given) Q Chemistry (given) Physics (given) R Physics (given) Chemistry (given) Practice Exercise – B6 For questions 1 to 5: According to the given information in the question, we get O is a bank manager and is a male (as he is married to a lady professor) ... (X) Q is a medical representative and is a male (as he is the son of M ... (Y) N is a chartered accountant and is a female (as she is the daughter-in-law of L) ... (Z) As given in VI that businessman is married to the chartered accountant, this would mean that the businessman cannot be L (as N, chartered accountant, is the daughter-in-law of L), O, Q and P (as they are having different professions, other than business). Therefore, logically M is the businessman with whom N is married. So M is businessman (male). ... (A) Given that P is an unmarried engineer. 6. d O (M ale/ Bank M anager) M P (U nm arried Engineer) 2. c 8. e 9. a 10. d Students Choices A B C D E F 1 R M R M M R 2 B P B C P B 3 C B T T B P R ⇒ Reading C ⇒ Cricket P ⇒ Painting B ⇒ Badminton M ⇒ Music T ⇒ Tennis Note: The best way to solve this problem is to fill the information given logically and complete the table. On the basis of the given information the third choice for both C and D would be tennis. L (Fem ale/Professor) 11. c (M ale/Businessm an) Son 7. a For questions 11 to 15: The given information can be tabulated as follows. Here as we know the profession of all the members except L and only profession unclaimed is professorship, therefore, L is a professor and because L is the grandmother of Q, therefore, L is female ... (B) From III, II and (B), we can very clearly conclude that O, the male bank manager is married to lady professor L. Using (A) and (V), we get that M is the son of L and O. From (IV), we get that Q and P are kids of M ... (C) So, all the conclusions using (X), (Y), (Z), (A), (B) and (C) can be depicted with the help of this family tree with the help of which rest of the question can be solved very easily. 1. d Compulsory Subject 12. d 13. c 14. c 15. b N (D aughter-in-law /C hartered accountant) Q (Son/M edical R epresentative) 3. a 4. e 5. b Explanations: Fundamentals of Logical Reasoning & Data Interpretation MBA Test Prep Page: 21 For questions 16 to 20: The given information can be tabulated as follows: Name Subject City A Philosophy (given) Hyderabad ... from (i) B Mathematics (given) Bangalore ... from (ii) Economics (as D does not teach economics therefore, C teaches it.) Delhi (given) D History (as only history is left) Jaipur (given) E Geography (given) Lucknow ... from (iv) 18. c 20. b C 16. b 17. d 19. d Solving the two equations, we get x = 5z and y = 19z. Therefore, the ratio y : x = 19 : 5. 19 + 5 = 24 which means that there must be at least 24 ladoos as the cats have an integral number of ladoos. If there are 30 ladoos, the monkey must have cheated the cats of 6 ladoos (i.e. 30 – 6). 5. b 6. c For questions 7 and 8: From the given data, we can make the following table with the help of which rest of the questions can be solved very easily. Male (40) Female (30) Married 7 12 Unmarried 5 0 Married 8 3 Unmarried 20 15 Total 40 30 Above 25 Below 25 Practice Exercise – B7 For questions 1 and 2: Five such pairs are possible. (10x + y) – (10y + x) = 36 ⇒ x–y=4 ∴ Numbers are 15 and 51, 26 and 62, 37 and 73, 48 and 84, and 59 and 95. 7. b 8. d For questions 3 and 4: x = 10a + b, x2 = 100a2 + 20ab + b2 y = 10 b + a, y2 = 100b2 + 20ab + a2 ⇒ In the square, middle number (ten’s place) will always be the same. 122 = 144, 212 = 441 132 = 169, 312 = 961 For questions 9 and 10: For every plate of ice cream, there are 2 employees. For every plate of sambar, there are 3 employees. For every plate of chicken, there are 4 employees. ∴ Number of employees must be a multiple of the LCM of 2, 3 and 4, i.e. 12. Only choice (d) of question 9 satisfies this condition. Another way of looking at this is as follows. Let the number of employees be x. x Therefore, the number of plates of ice cream = . 2 3. b Therefore, the number of plates of sambar = 1. e 2. b x . 3 x Therefore, the number of plates of chicken = . 4 4. b For questions 5 and 6: Initially: When the white cat gives z ladoos to black cat Black cat gives z ladoos to white cat Page: 22 Black cat x x+z x–z White cat y y–z ⇒ (y – z) = 3 (x + z) y+z ⇒ (y + z) = 5 (x – z) Therefore, x + x + x = 13 x = 65 2 3 4 12 Therefore, x = 60 (employees). Number of ice cream dishes = 9. d 60 = 30. 2 10. a MBA Test Prep Solution Book-2 For questions 11 and 12: 21 can be factorized as 3 × 7. Thus, to have 21 hand shakes the combinations for number of males in the two families can be as follows. P R C a se 1 7 3 C a se 2 3 7 For questions 19 and 20: Suppose the holiday lasted for x days. ∴ There were 11 nice morning. ∴ On x – 11 days it rained in the morning. Similarly, on x –12 days it rained in the afternoon. It never rained in the morning as well as afternoon. ∴ We have x – 11 + x – 12 = 13 or x = 18. Therefore, the holiday lasted for 18 days. It obviously never rained in the morning as well as in the afternoon. C a se 3 1 21 19. a C a se 4 21 1 20. e Practice Exercise – B8 Let there be x females in P family and y females in R family. Then for the first case, number of kisses will be 7y + 3x + xy = 34. Note that y = 2 and x = 4 satisfy the equation. So the total number of females will be 6. This will be true for the second case also. For case 3, total number of kisses will be equal to 1y + 21x + xy = 34. 1. a Shikha is to the left of Reena and Manju is to her right. Rita is between Reena and Manju. So the order is: Shikha, Reena, Rita, Manju. In the photograph, Rita will be second from left. 2. a B is to the right of D. A is to the right of B. E is to the right of A and left of C. So the order is: D, B, A, E, C. Clearly, A is in the middle. Note: That x = 0, y= 34 satisfy the equation. Therefore, the number of females will be 34. This will be true for case 4 also. Thus, the number of females can be 34. 3. b Q is to the left of R and to the right of P, i.e. P, Q, R. O is to the right of N and to the left of P, i.e. N, O, P. S is to the right of R and to the left of T, i.e. R, S, T. So the order is: N, O, P, Q, R, S, T. Clearly, Q is in the middle. 4. a S is sitting next to P. So the order S, P or P, S is followed. K is sitting next to R. So the order R, K is followed because R is on the extreme left. T is sitting not next to K or S (as given). So the arrangement will be R, K, S, P, T. Clearly, P and K are sitting adjacent to S. 5. a Clearly, the order is Anuradha, Rashi, Monika, Sulekha, Abha because Rashi is not adjacent to Sulekha or Abha and Anuradha is not adjacent to Sulekha. Therefore, Anuradha is adjacent to Rashi. 6. c Putting the conditions given in the question, the position of swimmers can be: 11. e 12. e For questions 13 and 14: 1×2×3=2×3 5 × 6 × 7 = 14 × 15 The choices in question 14 give the answer. 13. b 14. c For questions 15 and 16 ABCABC is divisible by 2 since it is even. Also, according to the divisibility rule of 7, ABC – ABC = 0 → The number is divisible by 7. ⇒ It is divisible by 14. For 16, data is insufficient. 15. a 16. a For questions 17 and 18: 1001 = 7 × 11 × 13 1 -year-old. Therefore, my 2 only possible age is 11 x 13 = 143 years. One of my greatgrandsons is 7-year-old and one of them is 1-year-old. 1 2 3 4 5 A C/D B D/C E 1 2 3 4 5 A D C B E I am obviously more than 95 17. c 18. c 7. e Positions of B and E violate rule 1. Explanations: Fundamentals of Logical Reasoning & Data Interpretation MBA Test Prep Page: 23 8. a There is only one possible arrangement, which satisfies the rule, i.e. 1 2 3 4 5 B D A C E For questions 16 to 20: According to the given question, Fa rgo C ad illa c From I ... (i) Fia t 9. d D is to the left of C, i.e. C, D. B is to the right of E, i.e. B, E. A is to the right of C, i.e. A, C. B is to the left of D, i.e. D, B. From the above statements, the correct order is: A, C, D, B, E. Clearly, D is sitting in the centre. From II ... (ii) Fa rg o Note: It is given that A, B, C, D and E are sitting facing you. So your right and left will be considered as left and right respectively. C ad illac From III, we get Fiat For questions 10 to 14: According to the question From III, Indian is wearing a green cap and a Jacket ... (1) From VI, Kurta is worn alongwith red cap and sits next to Japanese ... (2) From VIII, T-shirt with white cap combination is seated at one end. So from (1), (2), (3), VII and I we conclude that the Japanese wear a shirt of yellow colour. From IV, V, VI and VII, we conclude that the placement of people will be like (1) (2) (3) (4) German American Japanese Indian B e dford M aru ti A m b assa d or Fa rg o C a dilla c M erce de s [lo gica lly it ha s to be he re on ly ] From IV Fiat From (2) and IV, we arrive at the following table with the help of which rest of the questions can be solved very easily. A m ba ssa do r Nationality German American Japanese Indian Clothes T-shirt Kurta Shirt Ja cke t C aps Whitecap Red cap Yellow ca p Green ca p 10. d 11. c 15. a From I, we get France, America, India ... (i) From (II), we get India, Australia, Japan, China ... (ii) Combining (i) and (ii), we get the correct sequence as: France, America, India, Australia, Japan, China. The two flags in the centre are of India and Australia. Page: 24 12. c 13. c 14. c Fa rgo Hence, the sequence of cars is as follows: Fiat, Bedford, Maruti, Ambassador, Fargo, Cadillac, Mercedes. 16. d Clearly, Maruti is in the third place and Mercedes in the seventh, i.e. Mercedes is fourth to the right of Maruti. 17. b Clearly, Cadillac is in the sixth place, to the immediate left of Mercedes, which is in the seventh place (from the top). 18. d On the sides of the Cadillac are the Fargo and the Mercedes. 19. a Clearly, Maruti is in third place (from top), and is to the immediate left of the Ambassador, which is in the fourth place. 20. c To the right of Ambassador are Fargo, Cadillac and Mercedes. MBA Test Prep Solution Book-2 7. c Practice Exercise – B9 For questions 1 to 5: On the basis of the given information following graph can be constructed and from the graph, we can easily find the answers of the rest of the questions. Let A, M, P denote the sets of people liking apples, mangoes and pineapples respectively. n(M) = 20% of 10,000 = 2000 n(A ∩ M) = 5% of 10,000 = 500, n (M ∩ P ) = 3%of 10,000 = 300. n(A ∩ M ∩ P) = 2% of 10,000 = 200 We have to find number of people belonging to the shaded region in the following Venn diagram. M I 1 km M J 1 km 0 .5 km K L P 0 .5 km H A Number of people who likes only mangoes 1 km G D = n ( A ′ ∩ M ∩ P′ ) 1 km 1 km B 0 .5 km E C ( = n M ∩ ( A ∪ P )′ A ) = n (M) – n (M ∩ ( A ∪ P )) 1. d = n (M) – n (M ∩ A ) ∪ (M ∩ P ) 2. a = n (M) – n (M ∩ A ) + n (M ∩ P ) – n ((M ∩ A ) ∩ (M ∩ P )) 3. d = n (M) – n (M ∩ A ) + n (M ∩ P ) – n (M ∩ A ∩ P ) = 200 – [500 + 300 – 200] = 1,400 4. c 5. d 6. a 8. d Using the data given, the seven friends are sitting in the following order. EAGBDFC Therefore, E was sitting fourth to the left of D. 9. c Given information can be rewritten as (Nidhi – Pavbhaji) __ × ... (i) (Dhruv – Kriti) __ ü ... (ii) According to the given question, we have At 12 noon (Waiting) After announcement (Waiting) Left audience At 12.50 p.m. Required percentage = Total audience 180 Male 108 Female 72 108 72 36 72 (108 + 18) 126 36 36 90 36 90 250 % = 83.3% × 100 = 108 3 Explanations: Fundamentals of Logical Reasoning & Data Interpretation (Parul – Movie) __ × ... (iii) (If beach – Paanipuri) __ ü ... (iv) (Nidhi – Beach) __ × ... (v) (Circus – Pavbhaji) __ × ... (vi) (Harsh – Parul) __ × ... (vii) Clearly (ii) and (vii) implies Dhruv – Kriti, Harsh – Nidhi, and Amit – Parul. Also (iv) and (v) implies that Nidhi did not eat paani - puri. Also using (i), we conclude that Nidhi and Harsh ate chaat. Now clearly only the following combinations of place to visit and food items are possible. MBA Test Prep Page: 25 They are Movie – Pavbhaji or Chaat (using iv) Beach – Paanipuri Circus – Chaat, Paanipuri Applying elimination method and using (iv) we get beach – paanipuri, and using (vi) we get circus – chaat as the true combination and finally Movie – pavbhaji Thus we have Harsh – Nidhi – Circus – Chaat as one correct combination. Then the other combinations are: Using (iii), we get Amit – Parul – Beach – Paanipuri and obviously Dhruv – Kriti – Movie – Pavbhaji. 10. c 13. c On the basis of the analysis of the information given, the combinations of the movies and their type and the number of prizes they bagged is as follows. D Love story C Horror 2 A Action 1 B Kids 3 4 5 6 7 8 9 – × Sunday Physics – – – – Physical education Now, using (VIII) there is a gap of two days between computers and biology. This means that computers examination is either on 2nd or 5th and biology is either on 5th or 8th. If we take the second case, i.e. computer 5th and biology 8th, then using (VII) mathematics will be on 6th but this does not satisfy the (V) condition given. Now we consider the other case, i.e. computer 2nd and biology 5th. Using (vii), we have mathematics 7th and using (v) chemistry 6th. The remaining engineering drawing is on 8th, the only day left. So, in tabular form we have 2 Computer 3 Holiday Sunday 4 Physics Monday 14. d 15. c Biology Tuesday 6 Chemistry Wednesday 7 Mathematics Thursday 8 Engineering draw ing Friday 9 Physical education Saturday The combinations are as follows. A G B F Beach Rose C E Movie D H Chocolate The combinations are as follows. A G B F Beach Rose C E Movie D H Chocolate 16. b If we take first statement of I as false, then discount is with saree and wristwatch is free with it. The only condition where the logic of one statement is false and other true is justified, is possible when second statement of II is false and first statement of III is false. On this basis, we conclude that cap is free with bedsheet and T-shirt is free with shirt. Hence, choice (b) is the correct answer. 17. d The given information can be depicted with the help of following diagram. Saturday 5 None So (c) is the answer. We get the following table directly from the information given. 2 3 (As it has won maximum awards) Ten nis 8 80 4 30 6 90 S w im m ing 1 90 11. b 12. a Y is intelligent in mathematics and physics. Also Y isintelligent in chemistry and physics. 3 45 4 60 (d) is not possible as rubber and watches cannot be exported together. (b) and (c) are not possible as timber and garlic both are exported. So (a) is the answer. 6 75 B illiards 460 + (430 – 190) + (345 – 190) = 460 + 240 + 155 = 855 Page: 26 MBA Test Prep Solution Book-2 18. a 19. a 20. a Members playing only tennis = 880 – (430 + (460 – 190)) = 880 – (430 + 270) = 880 – 700 = 180 Members playing only billiards = 675 – (460 + (345 – 190)) = 675 – (460 + 155) = 675 – 615 = 60 So 180 + 60 = 240 Since the given information is not sufficient (because nothing is mentioned about the positioning of Vinay) to definitely depict the exact seating arrangement, ‘none of these’ is the correct choice. 4. d Option (a) has D and G both which is not allowed. All others are allowed. Therefore, choice (a) is the correct answer. On the basis of the analysis of the given information, we arrive at the following diagram with the help of which question can be solved easily. A nu P allavi Both B and C cannot be selected. So the other door paint has to be A. So A is always selected. R am ola Practice Exercise – B10 M a nd ira N ata sh a 1. d All of D’s children are in Z. So (d) is the answer. 2. c From the question, we get that _ Sapna _ _ _ _ _ _ _ 1 2 3 4 5 6 7 8 9 ... (A) Given that (i) Megha, Sapna and Riya cannot sit at 1 or 9. (ii) Beena and Megha does not have anybody sitting adjacent to them. (iii) There is only one empty chair between Megha and Riya. (iv) Charu is adjacent to both Jiya and Riya. From (iv) it is very clear that Jiya - Charu - Riya or Riya - Charu - Jiya will be the sitting arrangement ... (v) From (v) and (iii), we get Megha — Riya - Charu - Jiya ... (vi) From (i) and (ii), we conclude that Beena and Megha must have nobody adjacent to them. It means they must have at least one place empty adjacement to them. Now based on all the conditions (i), (ii), (iii), (iv), (v) and (vi), we get the only possible and valid arrangement as 1 X 8 9 X Beena ... (vii) From (vi), we get Megha will sit at chair number 7. 3. e 2 3 4 5 6 7 Sapna Jiya Charu Riya X Megha One of the possible sitting arrangements based on given information is depicted by the diagram given below. Vinay S he hul / M ilind K om olika 5. b Applying the rules given in the question, only two combinations are possible. A Cricket A Cricket B Football or B Cricket C Football C Cricket So A always play cricket. 6. b (a) is not feasible as it does not satisfy condition (i). (c) is not feasible as it does not satisfy condition (iii). Again option (d) does not satisfy condition (i). The only feasible group is given by option (b). 7. c Clearly, using (II) and (III) Poonam and Poornima do not stay on 1st floor and Priya and Priyanka do not stay on top floor. Using (I), Priya does not stay on 1st floor. So the only option for the girl staying on 1st floor is Priyanka. Thus, Poornima stays on 2nd floor, so Priya stays on 3rd floor and Poonam on the 4th floor. 8. a Using (II) and (VII), their full names are Sunil Sachdeva, Rohit Sehwag and Sandeep Sharma. Using (III) Mr. Sachdeva – Purse Now, using (V) Mr. Sharma – Cosmetics and Mr. Sehwag – Saree. Thus, (IV) implies that Himani is Mrs. Sehwag. Using (I) Dhwani is Mrs. Sachdeva and thus Vidhi is Mrs. Sharma. Thus can be tabulated as follows. K ira n A nil R aje sh Name Surname Gift Wife’s name Sunil Sachdeva Purse Dhwani Rohit Sehwag Saree Himani Sandeep Sharma Cosmetics Vidhi S he hul / M ilind Explanations: Fundamentals of Logical Reasoning & Data Interpretation MBA Test Prep Page: 27 9. e Let x be the number of employees at branch A. Then, 320 – x is the number of employees at branch B. 11. c Gunjan and Piya have to be together. ∴ Sameer would not be there according to (ii) and hence Priyanka would not be there according to (v). So the four girls are Shikha, Aastha, Gunjan and Piya. Hence, Anand and Vineet would not be there according to (iv) and (iii). Hence one of either Biswas and John will be there according to (vii). Since Biswas and Aastha have to be together, the member of the team are the ones given by choice (c). 12. c Clearly, using (III) the only salaries that satisfy this statement is Rs. 6,000. 10 % of 320 = 32 100 At branch A, number of absentees = 20% of x Total number of absentees = At branch B, number of absentees = 7 9 % of 13 (320 – x) Now, ⇒ 20 100 x+ (320 – x) = 32 100 13 × 100 Q 6000 = x 1 (320 – x) = 32 + 5 13 ∴ Kishan’s salary is Rs. 6,000 ... (1) Using (IV) and (V) Ganesh’s salary is not Rs. 10,000 ... (2) Also Ganesh – EXCEL Convergys – Rs. 10, 000 ... (3) Then (VI) and (I) implies that Shiv’s salary is either Rs. 12,000 or Rs. 15,000 and Shyam’s salary is either Rs. 6,000 or Rs. 7,500 ... (4) Using (VII) person working in Daksha gets Rs. 15,000 ... (5) Using (1) and (4), Shyam’s salary is Rs. 7,500 ∴ Shiv’s salary is Rs. 15,000 and hence he works in Daksha [Using (5)]. Thus, Ravi’s salary is Rs. 10,000 and he works for Convergys (Using 3) Using (III), Kishan works in Global Vantedge and Shyam in GE Capital. ⇒ x = 60 ⇒ 320 – x = 260 But the number of females present on the day is 30% of the total employees present. But it is not given that how many of those who were absent are females. So, we cannot find the total number of males at branch B. 10. d Let us assume that they carried N chapattis. Let N = 3x + 1 ... (i) He eats his share. Number of chapattis left = 2x Let 2x = 3y + 1 ... (ii) The second friend eats his share. Number of chapattis left = 2y Let 2y = 3z + 1 ... (iii) The third friend eats his share. Number of chapattis left = 2z. Let 2z = 3w + 1 ... (iv) All three of them get up and eat their shares. 3 × 10,000 5 Name Call Centre's name Salary (in Rs.) 81 65 W+ 8 8 For even W – N will not be an integral number for Ram Convergys 10000 Shyam GE Capital 7500 W = 1, N = 73 4 Kishan Global Vantedge 6000 Shiv Daksha 15000 Ganesh EXC EL 12000 Using (i), (ii), (iii) and (iv) we get N = 77 W = 3, N = 2 235 4 W = 7, N = 79 ∴ Minimum number of chapattis = 79 W = 5, N = Page: 28 13. c The information given can be written in a compressed from as follows. (i) Aman, Algebra, 2 (ii) Ankur, __ , 4, Scientist (iii) __, Botany, 1, Teacher (iv) Ankit, __, __, Businessman (v) Amrita, Geometry, __, Architect (vi) Mathematician and Physics expert stay on same floor 2. MBA Test Prep Solution Book-2 Using (i), (ii) and (iii). Aman is not a scientist or teacher and Ankur is neither good at algebra nor at botany. Again (i), (iv) and (v) imply that Aman is not a businessman. So, by elimination method, Aman is an architect. Similarly, Ankur is either good at zoology or physics. But using (vi), Ankur is a zoology expert. One of the mathematician stays at 2nd floor and that is Aman. So, Amrita is not the one staying on 2nd floor. Hence, by elimination she stays at 3rd floor. Thus the other person staying on 2nd floor is Ankit and he is the physics expert. So using (iv), Ankit, Physics, 2, Businessman is the correct combination. DS Practice Exercises 5. e Neither statements I and II alone nor taken together can give remainder as remainder will vary according to the numbers. ∴ Answer is (e). 6. e Statement I does not have data to find the cost of each album. Statement II gives the total amount spent but we cannot find out the percentage of sales tax per copy. 7. d Combine both the statements to find the answer. 8. e Since nothing about expenditure and the rate of tax is mentioned, the salary cannot be calculated. 9. d Let the total number of students appear in the examination be x. Statement I will give the information about passed students. Statement II is also not complete in itself. Combining the statements I and II, we get Practice exercise – C1 1. d Statement I is very tempting as 4 = 4 × 1, but both these numbers can be negative also. Hence, I is not enough. Statement II says that both A and B are positive. So both statements taken together solve the problem. 2. a Since all the 3 are odd and z – x = 4, they have to be consecutive. 3. c By statement I, a + 1 = a + a + 2 ∴ a = –1 By statement II, a(a + 2) = –1 ∴ a = –1 ∴ Answer is (c), i.e. it can be found out using either statements alone. 4. d 50 51 x + 10 = x 100 100 Hence, x can be calculated. 10. b Statement II clearly gives that a can of beer is costlier. Thus, (b). 11. b CP = 12. d Only statement I or II alone is not sufficient. Combining statements I and II, we can calculate the list price as (Rs. 1485 × 12) = Rs. 17820. Hence, discount is Rs. 2,320. 13. a From statement I, cost price = Rs. From statement I, we get a–2 a a+2 –1 1 3 2 SP ⇒ SP = 1.5 CP 3 Profit percentage is 50. 15 and 12 6 4 Hence, profit can be worked out. So statement I is sufficient. From statement II, we have no idea about the selling price and hence the profit. So statement II alone is insufficient. selling price = Rs. –3 –1 1 –5 –3 –1 A ll 3 case s w e g e t ‘– 1 ’ So we do not know what the average will be from statement I only. Using only II we can say that the numbers are –1, 1, 3 or 1, 3, 5. Using both the only possibility is –1, 1, 3 and hence we can find the average. ∴ Answer is (d). Explanations: Fundamentals of Logical Reasoning & Data Interpretation 14. e Knowing the highest and the lowest scores tells us nothing about the other scores. So statement I is not enough. Statement II is very tempting but students must realize that suppose 2 people take the GRE, one scores 2300 and the other scores 1000. Surely the average is not 2000. So statement II cannot give us the answer. The two statements taken together also cannot answer our question. MBA Test Prep Page: 29 15. c Statement I indicates that the number of 50-paisa coins is 2 and the number of one-rupee coins is 3. Statement II independently gives the same result. 16. d A relation between a and c can be found using both statements I and II, i.e. From statement I, it can be found that the total of the deposits = 75 × 5 = $375 18. c d = Kt , u = 0 from statement I, 1205 = K. 2 1205 25 We need to determine D ⇒ 25 K = 10 7. e Let the amount of money Prem has be P, and the amount that Jagdish has be Q. Statement I gives the inequality P ≥ J + 100. Statement II gives the inequality P + J ≤ 500. It is obvious that we cannot get the answer from statement I alone or statement II alone or even from statements I and II together. 8. e Both the statements do not give any information about the speed limit. Thus, (e). 9. e It is not specified which tap is opened and which one is closed, and what part of the tank was initially full. 10. d From statement I, we cannot find the hours. –D 9 120.5 120.5 × 100 , D9 = × 81 25 25 D10 – D9 = Statement II repeats statement I. Hence, the question cannot be solved. a+c a = +1 c c 17. a D10 = 6. e x y + = 1 which enables 5 3 us to get the answer. By using both the statements together we get the answer. From statement II, we get 120.5 120.5 (100 – 81) = (19) 25 25 Note: You do not have to calculate2 the actual answer. From statement II, 490.4 = Kt 11. b 1  Statement II gives time required by B as  × 10  2  490.4 100 Proceeding as above, D10 – D9 can be determined. 2 K . (10 ) ⇒ K = 19. e 20. d = 5 days. Hence, statement II alone is sufficient. There are more unknowns than the number of equations. Hence, both statements I and II are not sufficient. 12. e The statements given do not give any information about the number of rooms to be vacuumed. Both statements are required. 13. b From statement I, we can only get an equation in terms of x and y, but not the value of x and y. From statement II, we can get the value of x as 0. Practice exercise – C2 1. a Statement I gives only the relative efficiencies and no idea of time taken is there. So statement I is insufficient. x 2 + 18 = x , where x is 3 2 the initial amount of money he took to the mall. Using only statement I, 2. a a+b a  a  =  + 1 :  − 1 a−b b  b  3. e Both the statements give the same information. 4. e 10 students in statement II may not be the remaining beyond the 30 given in statement I. 5. e Using both statements I and II, we still cannot say anything about the number of students in the class. Page: 30 14. a 2pq + pq2 =2+q pq 15. b Statement (I) gives us an inequality which is not enough to answer the question. Statement II indicates that g is greater than h because irrespective of the sign of the integer, the integer whose cube is greater will obviously be the greater one. Therefore, statement II alone is enough. 16. d From the given condition, a6 = b6 But since the power is even we do not know whether a = b. But from I and II we can conclude that both ‘a’ and ‘b’ are +ve. Thus, a = b ∴ a3 – b3 = 0 ∴ Answer is (d). MBA Test Prep Solution Book-2 17. d From I, we have r = 0 or r = 3. So it is not sufficient. From II, we have r = (–1) or r = 3. So it is also not enough. The two statements taken together give r = 3. 18. d Statement I says that there could be 25 or more books. Statement II says that there could be 25 or less books. The two statements taken together give us the answer as 25 books. 19. a x2 + 2xy + y2 = (x + y)(x + y) 20. e From I and II, we get 2 equation in two variable. But they are dependent. Thus, eventually we get one equation in two variables. Which has infinite solution set. Thus, (e). Practice exercise – C3 1. b Statement I can be factorized as [x – 3][x – 1] = 0 giving two possibilities, statement II is also a quadratic equation but when factorised it yields (x – 1)2 = 0 2. b Statement I does not give any information, since if ‘x’ is –ve and ‘y’ is +ve, then also x2y is +ve and if x is +ve and y is +ve, then also x2y is +ve. Statement II tells us either both x and y are +ve or x is +ve. both –ve. In either case y 3. d 4. e 5. b From statement II, we get x2 < 1 x2 – 1 < 0 (x + 1)(x – 1) < 0 –1 6. e 2x > 6 implies x > 3, while 3x < 10.9 implies x < 3.6333. Hence, the answer cannot be conclusively determined. 7. d From statement II, we get x as positive. Hence, using this in the first statement we can infer that x > y. 8. d Since the value of x2 – 7x + 12 either increases or decreases depending upon the domain of x, both statements I and II are required to answer the question. 9. a Statement I implies that x can be between 1 and 2 or –1 and –2. Statement II implies that x is between 1 and 2. 10. a Statement I is true only if a = b = c = 0 11. d From statement I, we can say 1 = 4 >1 0.25 From statement I, we have x² – 7x ≥ 8, means x² – 7x – 8 ≥ 0. So (x – 8)(x + 1) ≥ 0. So x ≤ –1. And x ≥ 8. And combining with statement II, we get x = 8. Both the conditions are not sufficient to say if xy > 1. Since if x = 0.25 and y = 10 ∴ xy = 2.5 > 1 and if x = 0.025 y = 1.2 xy < 1 ∴ Answer is (e). From statement I, we have 1 x 1 = 0.5 < 1 2 Thus, statement II is needed where x < 1 ∴ Answer is (d). 12. e <1 Consider x = 4 1 ∴ = 0.25 < 1; for x > 1 4 Now consider x = –4 1 = − 0.25 < 1 ; for x < 1 −4 So even if ‘x’ is > 1 or x < 1, statement I is satisfied. Thus, we cannot say from that if x > 1 or x < 1 ∴ Explanations: Fundamentals of Logical Reasoning & Data Interpretation +1 Thus, we can say that x lies between +1 and –1. Thus, x is not greater than 1. Thus, (b). 13. a m = n + p cannot be said if n > p. n From statement II, n > m, i.e. n = km, where k > 1 n m Again it cannot be said if n > p. Combining, = p n 1 ∴ = n + p, i.e. n + p < 1 x It cannot be determined if n > p. From statement I, From statement I, a = 40° ∴ a + b + c = 180° ∴ b + c = 140° And x + b + c + y = 360° ∴ x + y = 360 – (b + c) = 360 – 140 = 220 ∴ The answer is (a). MBA Test Prep Page: 31 Page: 32 MBA Test Prep Solution Book-2 Explanations: Fundamentals of Logical Reasoning & Data Interpretation Page: 33 From option (d), it can be seen that fact (v) is worthless as far as the data about weights go. Also, facts (i) and (ii) do not confirm D as the lightest among all. Therefore, option (c) consists of sufficient facts and hence, is our answer. For questions 4 to 6: Another new type of question set. Basically, instructions would have given you the idea that the question is not to be answered but the information required to find the answer to the question must be singled out from the sea of data. We analyze the facts one by one in a tabular format. Fact # (i) (ii) (iii) Height C>D D>B A>B (iv) - (v) SUMMING UP 4. a A C>B D A C > >B D Weight C>D B>C B>A C A >D - B> C >D A Here, we are interested in the weights of B and D relative to each other. We see that the first two facts tell us that B > C and C > D. Thus, both of them combined have told us that B > C > D and hence, B > D. Note that even (iii) and (iv) combined would have directed us to the result (B > A > D). But in the options, we had a. (i), (ii) B>C>D b. (ii), (iii) No information about D c. (i), (iv) All information (about weights) in fact (i) is already contained in fact (iv). d. (ii), (v) Fact (v) does not tell anything about weights and (ii) alone is insufficient. So, only option (a) contains all the facts required to answer the question. The answer to the question asked is "YES", but we have nothing to do with that answer. 5. e Here, a sequential procedure was to be followed. First, we had to find who is the shortest individual. Then, we had to find who is the heaviest individual. Finding both of them can confirm if the same individual is referred or not. Hence, from facts (i), (ii), (iii) and (iv) we get to know that B is the shortest as well as heaviest. So, option (e) is correct. 6. c From option (a), it becomes clear that B or C cannot be the lightest (because they are heavier than somebody). But lightest could be either A or D. From option (b), information revealed is that C or A is not the lightest. But still, we cannot be sure about D because information about B is still concealed. From option (c), we come to know that neither B nor C nor A is the lightest. Hence, D has to be the lightest. Page: 34 In questions 7 to 11, the task was not to identify the option containing sufficient facts, but to tell the minimum sufficient number of facts required. In this scenario, redundancy and intersection of the information in the facts must also be kept in mind while marking the answer. In these questions, the ultimate result acquired through the synthesis of all the information given by all the hints taken together would be employed. 7. a In the table, we can see that C is the tallest. Height order of C can be computed by fact (v) alone, which directly tells that C is the tallest. Minimum 1 fact required. Option (a). 8. c From the table, B is the heaviest. To know that B is the heaviest, we must make sure that there is at least one person heavier than each of A, C and D. For that purpose, we require facts [(ii) AND (iii) AND { (i) or (iv) } ]. Minimum 3 facts required. Option (c). 9. a We can see in the table that D is not the shortest. To confirm this, we need just one information that could tell us that there is at least one person shorter than D. Fact (ii) tells us just that. Only one fact required. Option (a). 10. b From the table, D is the lightest. To know that D is the lightest, we must make sure that there is at least one person lighter than each of A, B and C. For that purpose, we require facts [(iv) AND { (ii) or (iii) } ]. Minimum 2 facts required. Option (b). 11. c B is the shortest. To know that B is the shortest, we must make sure that there is at least one person shorter than each of A, C and D. Only facts [(ii) AND (iii) AND { (i) or (v) } ] can confirm the premise that B is the first in ascending order of heights. Minimum 3 facts required. Option (c). MBA Test Prep Solution Book-2 For questions 12 to 20: This is a typical kind of set similar to that featured in CAT 2005. The questions asked you to decipher the data and then ascertain the dependency of the two statements specific to each question. Hence, it must be understood that these kinds of questions ask for a bit more than forming cases or inferring a pictorial / tabular simplification of the information. The data can be synthesized as: Five ladies Colour of sarees Mrs. Laali Red Mrs. Neelima Blue Similarly, number of affairs of L is greater than that of someone else. So, it can never be minimum (6 for orange). Information 3: In Adorable colony, H has the maximum number of affairs. Interpretation: From the table, we can see that the maximum number of affairs in Adorable colony is 4, which are those of the lady in Red and the lady in Green. Hence, H is wearing either Red or Green. Now, we must move on to the questions. 12. c Straight from the information 3, we deduced that H is in red or green. So, none of the two statements is definitely true. However, exactly one of them is true. Hence, option (c) 13. a Option (a): Statement I tells that green is the colour of the saree worn by K. And since, from information I, if total affairs of K = 10, total affairs of N will be 9, that means N is wearing blue coloured saree. So statement I confirms statement II. Note that we assumed statement I to be true and concluded the truth of statement II. Hence, this option is correct. Option (b): If statement II is assumed to be true, then according to information 1, i.e. K = 1+N, K has 10 affairs. Hence, K wears green saree which makes statement I true. So option (b) is not correct, because it concludes statement I to be false. Option (c): If statement I is assumed to be false, then K doesn’t wear green saree. Hence, she wears either red or black saree. That means, number of her affairs = 7. Thus, number of affairs of N = 6 (orange saree). So statement II is also false. Hence, option (c) is not correct, because it concludes statement II to be true. Option (d): None of the two statements is definitely true because there are other cases possible. 14. a 2 affairs in Affection colony means either red or green. And Mrs. Hariyaali is in one of red or green sarees (from information 3). Hence, statement I is true. 2 affairs in Adorable colony means either blue or orange. And from information 1, we have seen that K cannot wear blue or orange. Hence, statement II is also true. 15. d S cannot be green and L cannot be orange from information 2. Hence, only statement II is true. 16. c Information 2: S has lesser affairs than L Interpretation: S < L. First things first. S cannot be maximum(Green) and L cannot be minimum(Orange). That is because number of affairs of S is less than someone else’s. Hence, it cannot be maximum (10 for green). We know that K wears the red or green or black coloured saree only. Hence, blue is not possible. Green coloured saree for K is possible. So from the options, we see that option (c), which says that statement II is true and statement I could be false, is the correct option. Now information 1, 2 and 3 must be combined for necessary deductions. If N is not wearing orange, she must be wearing blue. And N(blue) => K(green). This way, H (green or red) has to wear red. And since S < L, S (orange) and L(black) is the only combination possible for N wearing blue. Explanations: Fundamentals of Logical Reasoning & Data Interpretation MBA Test Prep Mrs. Hariyaali Green Mrs. Kesari Orange Mrs. Shyama Black However, the colour of the saree worn by an individual lady in particular is not known. Only possible source of information to come to any kind of conclusion regarding the colour of saree each lady wears is the following table along with three additional information given in the question. Number of extra-marital affairs Colony Colour of the saree Red Blue Green Orange Black Affection 2 4 2 3 1 Adorable 4 2 4 2 3 Fondness 1 3 4 1 3 Total 7 9 10 6 7 Since time is what we fall short of while attempting such crucial questions in any exam, we must turn as economical for that resource as possible. Therefore, Mrs. Laali will be addressed as L, Mrs. Neelima as N, Mrs. Hariyaali as H, Mrs. Kesari as K and Mrs. Shyama as S from now on. Information 1: K is having 1 affair more than N Interpretation: Number of affairs of K has to be exactly one more than that of N. The possibilities from the table above are N = 6 and K = 7 or N = 9 and K = 10 There is no other possibility. So it is clear that: (i) N wears the orange saree or blue saree (ii) K wears the red or green or black saree Also, N (blue) ⇒ K (green) and N (orange) => K (black / red) Page: 35 Now, we assume that N is wearing orange. Hence, K will be wearing black or red. When K wears red, H (green or red) wears green and hence S wears black and L wears blue (because S < L). When K wears black, H wears green or red and accordingly, S wears red or blue obeying S < L. The exhaustive list of possibilities Red (7) Blue (9) H N K S L K L H N S S L H N K H S L N K 17. a 18. c 19. d Green (10) Orange (6) 20. b Black (7) Option (a): If statement I is true, H doesn’t wear green. That means, H wears red. Correspondingly, S could wear orange or blue, and not necessarily orange. Option (b): H not wearing green means H wears red. In that case, S can wear blue or orange. But when it is given that S wears orange, we can see from the table that H wears red for sure (and hence not green). So given that statement II is true, we can infer that statement I must be true. So, this is the right option choice. Option (c): If statement I is false, it means that H wears green. Then S wears black or red but never orange. Hence if statement I is false, statement II would necessarily be false and not true. Option (d): If statement II is false, S wears black or red or blue saree. Hence, H may or may not wear the green saree. Minimum number of affairs = 6 (orange). Neelima not wearing orange means she wears blue. In that case, Shyama will wear orange (see table) and she would be having only 1 and not 3 affairs in Fondness colony. Neelima wearing orange means Shyama could be wearing red or blue or black. 3 affairs in Fondness colony is true for lady wearing blue saree and lady wearing black saree. Shyama could have 1 affair in Fondness colony if she happens to wear the red saree. Hence, statement II could be true if statement I is false. LRDI Practice Exercises 1. b We know that S cannot wear green. And K cannot be orange because K can be red or green or black only. Hence irrespective of the conditions, both statement I and statement II are definitely false. There are just two hotels having grade ‘A’ and exceeding 13,450 customers, viz. Taj Residency and Mandar International. 2. c Just count the hotels with grade ‘A’ or ‘B’ and having more than 11,500 customers per year. 3. b There are just five hotels with their names starting with alphabet ‘M’ or ‘R’. Of these Madhuban Deluxe has the highest customers per year and which is greater than 1,500, the second highest. So even with increased customers for other hotels, Madhuban Deluxe will have the highest customers per year. 4. b Same explanation as in question 3. 5. b In metro, valuation is > 10 times the equity. In A circles, valuation is more than 4 times but less than 5 times. In B and C circles, valuation is more than 5 times, so valuation is least for A circles. 6. c Combined share of metro and A circles = Rs. 1,48,350 million Total valuation of all circles = Rs. 2,02,900 million Thus, percentage share of metro + A circles Option (a): If we take statement II to be true, L has maximum (4) affairs in Affection colony. So she wears blue saree. In that case, H has the maximum number of affairs (green saree). Hence if statement II is true, statement I is also true. This option claims that statement I is false. Hence, not the right option. Option (b): If statement I is taken to be false, H doesn’t have maximum affairs and therefore, she doesn’t wear green. So she wears red. It implies that L could wear black or green. Therefore, L can never have maximum affairs in Affection colony (which is true for lady wearing blue only). So statement II is certainly false. Hence, this option is not correct. Option (c): Both the statements are not independently true, as can be seen from the table. We do have cases when H does not have maximum number of affairs. Hence, this option is not valid. Option (d): H having maximum affairs means H is wearing green (10 affairs). In that case, L definitely wears blue (as can be seen from the table) and hence she has the maximum affairs in Affection colony (4 affairs). So, statement II is necessarily true if, statement I is true. So, this is the right option choice. Page: 36 Practice exercise – D1 = 7. c 148350 < 74%, i.e. 73.18% 202900 Total losses written off is Rs. 1,000 crore = Rs. 10,000 millions. Thus, the increase in valuation for Metro and A circles is Rs. 4,444.4 million and Rs. 5,555.5 million respectively. Valuation to equity ratio has to be equal for A and B circles by reducing the MBA Test Prep Solution Book-2 equity of A circles, thus 72656 × 7875 = Rs.12,858 million. x= 44500 Thus, percentage decrease in equity of A  12858  = 1 −  = 16%  15300  8. e Total valuation for Delhi : 81250 × 0.42 = 34,125 Maharashtra : 67100 × 0.25 = 16,775 Karnataka : 67100 × 0.22 = 14,762 ⇒ Delhi : Maharashtra : Karnataka = 34.1 : 16.7 : 14.7 9. d By observation, we can say that E has registered maximum increase. 10. b (i) 11. e 12. e 13. b Percentage change 44500 67100 + 5556 = or 7875 x Price of share A on day 1 = Rs. 198 Price of 100 shares on day 1 = 198 × 100 = Rs. 19,800 If we encash 50 shares of A on day 5, we get 201 × 5 = Rs. 10,050 And on day 6 = 203 × 50 = Rs. 10,150 Gain in case of share A is (10050 + 10150) – 19800 = 20200 – 19800 = Rs. 400 (ii) Investment of share B on day 1 = 1012 × 50 Total investment = Rs. 50,600 Total investment on day 1 on share C = 52 × 50 = Rs. 2,600 (iii) Investment of share B on day 1 = 1012 × 100 If 50 shares of B and C are encashed on Days 5 and 6, then amount encashed = (1034 + 1067) × 50 – 1012 × 100 = Rs. 3850 Amount of share B on day 6 = 1067 × 50 = Rs. 3,350 Amount of share C on day 6 = 56 × 50 = Rs. 2,800 Total return = 53350 + 2800 = Rs. 56,150 Gain = 56150 – 53200 = Rs. 2,950 So, (ii) is better than (i). We can observe from the table that the percentage increase for stock E is more than 20%. Stock C has shown the least increase in absolute terms but in percentage terms stock D is the least. Volatility is not defined whether it is in percentage terms or in absolute or something else. So none of these is the correct choice. Day 4 = 199 × 20 + 1030 × 30 + 54 × 25 + 414 × 15 + 62 × 10 = 3980 + 30900 + 1350 + 6210 + 620 Day 4 = 43060 Day 5 = 201 × 20 + 1034 × 30 + 55 × 25 + 417 × 15 + 63 × 10 = 43300 Explanations: Fundamentals of Logical Reasoning & Data Interpretation 14. e = Day 5 − Day 4 43300 − 43060 = Day 4 43060 = 28050 × 100 = 0.55% 43060 Percentage increase in gross assets from 2001-02  800   to 2005-06 =   − 1 × 100 = 515 .4%  130   15. b Total disbursements from 2002-03 to 2004-05 = 560 – 65 = Rs. 495 crore 16. b Maximum percentage increase in gross profit is for  33   1995-96 =  21  − 1 × 100 = 57.1%    For questions 17 to 21: 17. b Route Carrier No. of Fare Fuel (Rs.) tickets 3 Rs. 14100 2820 I Jet Airw ays II Indian Airlines 4 Rs. 13640 III Air Deccan 2 Rs. 3510 2728 912 Rs. 6460 18. b Total expenses = Fare + Time not used in working Wage = Rs. 200 / hr ⇒ By air = Rs. 2,375 + Rs. 200 × 2 = Rs. 2,775 and By train = Rs. 2,286 + Rs. 200 × 9 = Rs. 4,086 So, profit = Rs. 4,086 – Rs. 2,775 = Rs. 1,311 (Since total expenses are lower for sending him by Air Deccan). 19. d Neither remuneration nor number of employees is known. 20. c Check the options, (c) is maximum. 21. d The last digit can be found only if we know which number is higher. Hence, both the statements are needed. 22. d Overall winning percentage is the weighted average of percentage won in the 2 years, with the number of matches being the weights. Thus, we need both the statements. MBA Test Prep Page: 37 23. c 24. c If he was dot on time for the class as per his watch, 7. d We can write the equation as 3rd (r + d) = 0, r can 8. d Shares are bought and not sold, so no tax is paid, since tax is paid only when shares are sold. 9. a Shastri = 7 × 6 = 14 3 Akram = 10 × 6 = 20 3 be − 3 or r can be 0. Hence using either of the statements alone, one can say that r = − 3. 25. d Combining (I) and (II): Time taken by Shyam = 2 hr Ratio of speed = 2 : 3 So time taken by Ram 2 = 3 Razzaq = 12 × 20 t t = 45 min So Ram reaches at 11.40 a.m. + 45 min = 12.25 p.m. Practice exercise – D2 1. a Binny = 9 × 10. c 18  1 1 1  10 + 3 + 5  = 1−  + +  = 1−   = 1− 30 30  3 10 6    Shastri = 6 4 2 = 1− = = 10 10 5 Binny = 3. d 32 = 16 2 It seems, either Maninder or Shastri is the answer. Maninder = Skin = 1 = 10% 10 Proteins = 16% ⇒ 4. c 12 =4 3 Quantity of water in a body of 50 kg = 70% of 50 kg 70 × 50 = 35 kg 100 10 × 100 = 62.5% 16 Percentage requirement of proteins and other dry elements = 30% 6 = 27 2 31 32 < 3 3 Azad = = 6 = 72 1 32 > 10 3 Akram = Part of body made of neither bones nor skin nor Muscles 11. d 2. e P ratio has not decreased by more than 20%. So E no share is bought. So (0, 0) 1 he would have been 22 min late. 2 21 12 = 1.7 = 1.9 , Shastri = 11 7 12. e We cannot find the runs scored as we do not know the extras. 13. e Number of literate females in India in 1991 = 39.4% of 430 million = 169.4 million 14. e Cannot be determined because we do not have the population (the weights for average) for the states. 15. e Number of illiterates in Maharashtra in 2001 = 36.9% of 19 million = 7.01 million 16. e Because the infant population of states is not given. Hence, the data is insufficient. o  30  × 360  = 108° Therefore, the required angle =   100  5. b In value term, Godrej appreciation has been highest = 200 6. a Very clearly, Dabur, which has become almost 3 times in 2008 from 2006. Page: 38 For questions 17 to 21: 17. e Some people can be lawyers as well as women Also passage of the bill is not defined. ∴ We cannot find out the answer. 18. a Average age = MBA Test Prep 63 × 550 + 58 × 250 = 61.43. 550 + 250 Solution Book-2 19. c 20. b Minimum case is when all the Rajya Sabha female members are Hindus, i.e. 42. So, minimum number of Hindu males is 64. Percentage of Congress(I) members = = 21. b 4. b Aggregate deposits for the year ended March 1999 = (717271 – 111861) = Rs. 6,05,410. Hence, percentage increase in deposits  111861  =  = 18.5%  605410  379 = 47.37% 800 From statement I, we have Y > – |X|, which means Y is less than or greater than X. So statement I by itself cannot answer the question. From statement II, we have Y > |X| which means Y is greater than X. So it is sufficient. The sum of a, b and c cannot be found from statements I and II individually or together. 23. d Statement I alone is not sufficient as the total number of guests is unknown. Statement II alone is not sufficient as the proportion of single-scoop and double-scoop is not known. Combining statements I and II, we can find the solution. 25. a Japan 267 + 112 550 + 250 22. e 24. c 3. a 5. b  366003  × 100 = 51%    717271  6. e  800000 − 717271  × 100 = 11.5%   717271   7. b  480000 − 366003  × 100 = 31%   366003   8. c Statement I indicates that n is divisible by 1, the 2 prime numbers and their product. Hence, there are 4 different positive integers. From statement II, the factors of 8 are 1, 2, 4 and 8 itself, again 4 different positive integers. Using statement (I): a × b = 16 ⇒ a = 4, b = 4 a = 2, b = 8 a = 8, b = 2 In all the above cases, the last digit of (5 × ab)ab will be zero. So statement (I) alone is sufficient. Statement (II) alone is not sufficient. 9. d 2. b Increase in total population = 548 – 439 = 109 million Increase in Hindu population = 453 – 366 = 87 million Increase in Hindu population in total population increased = 356 × 10 ≈ 10% 3694 123 = 512 Population of Hungary = 0.24 622.72 – 549.8 × 100 549.8 = 13.3% Practice exercise – D3 Percentage = Hindu population in 1971 = 1.2 × 0.78 × 665.3 = 622.72 Percentage increase = 10. a 1. e For a credit deposit ratio of 60%, if deposit is Rs. 8,00,000 crore, then deposit should be Rs. 4,80,000 crore. 87 ≈ 80% 109 Population of Hindu, Jain and others in 1941 = 424 million Population of Hindu, Jain and others in 1961 = 641 million. Ratio = 424 2 ≈ 641 3 549.8 (550) ≈ ≈ 660 million 453.4 450 2 Population of Pakistan = 285 = 109 2.62 11. e Hindu in 1971= 549.8 × Others in 1971 = So difference = 403 Ratio = Explanations: Fundamentals of Logical Reasoning & Data Interpretation 75.5 × 75.5 (75)2 ≈ = 94 million 61.4 60 660 7 ≈ 94 1 MBA Test Prep Page: 39 12. a Clear from Data. 13. a Clear from Data. 14. a R.D. = 604 604 − = 37.75 − 30.2 = 7.55 16 20 J.K. = 503 503 − = 25.15 − 22.86 = 2.29 20 22 M.K. = 902 902 − = 43 − 39.21 = 3.79 21 23 23. e Combining (I) and (II): The oldest institute could be any one of the three, i.e. Career Launcher, Erudite, or IMS. Hence, it cannot be determined. 24. a The cyclicity of 2 is 4. Using statement (I), xy can be 43, 49, 83, or 89. So last digit of (72)xy could be 2 or 8. Using statement (II), xy can be 46 or 86. So the last digit of (72)xy has to be 4. 25. d Combining (I) and (II): c=x a=x–1 b = 2x d = 2x + 2 832 832 − = 39.62 −34.67 ≈ 5 20 27 Clearly R.D. has the highest difference. ∴ Option (a) is the correct choice. A.S. = 15. d a x −1 c x = and = b 2x d 2x + 2 ad = (x – 1)(2x + 2) = 2x2 – 2 bc = 2x (x) = 2x2 So Calculating the strike rate of the given bowlers we get (a) 3.2 for Harmission (b) 4.68 for Shane Warne (c) 4.26 for M. Muralidharan (d) 2.9 for Pollock As ad < bc, a c < b d Practice exercise – D4 For questions 16 to 20: 1. b Total amount spent on ad = Ad spent on (Print + TV + Others) In 1998 = 6824, in 2000 = 8988 Average annual growth rate of ad spent 16. c Except Reliance Growth, Birla Dividend Yield Plus and Templeton India Growth all others fit in the condition. Hence option (c) is the correct choice. 17. a Reliance Growth is maximum at 0.58. 18. a Templeton India Growth is 2nd best at 0.52. 19. a HSBC Equity – 0.09 Franklin India Blue-chip – 0.14 20. c Barring top – 3, all others are eligible.  4745   − 1 = 24.2% Print =   3820  21. d Combining both the statements, we can determine that S is standing at the middle position.  3542   − 1 = 44% TV =   2460  22. d Combining (I) and (II), we get the following sitting arrangement.  701  Others =   − 1 = 28.8%  544   8988   100 = 15.8% =   − 1 × 2  6824   2. b 3. e Stuti Amount spent on TV as a percentage of total ad  2952   = 38.4% spent for 1999 =   7687  S h ikh a R itu Growth rate of ad spent is the highest for TV as shown: 4. b 5. c Decreases continuously.  512.02    × 100 = 21.6%  2365.23  B a sis th S u kan ta M and ar Page: 40 MBA Test Prep Solution Book-2 6. b Exports of pearls in September 1998 have fallen by US $0.26 million (0.54-0.28) from the value in September 1997. Percentage decrease = 7. e  0.28   16.63  × 100 = 1.68%   8. e  2365   2860  × 100 = 82.7%   9. e 10. c 0.26 = 48.1% 0.54 (210 – 220) + (225 – 230) + (232 – 215) + (215 – 205) + (215 – 248) + (226 – 247) + (245 – 239) + (250 – 223) + (217 – 248) = – 40 Export earnings of jewellery + leather = 32% of 230 food grain imports = 46% of 225. Percentage of food grain imports that could not be  (0.32 × 230)  paid for = 1 −  × 100 = 28.88%  (0.46 × 225)  11. b In June, cotton exports = 27% of (100 – 7.25)% of 215 = 0.27 × 0.9275 × 215 = Rs. 53.8 million 12. e Chemical import = 0.23 × 225 = Rs. 51.75 million Export of jewellery = 0.18 × 230 = Rs. 41.4 million Difference to be covered by spices = Rs.10.35 million Spice exports = 0.07 × 230 = Rs.16.1 million Thus, percentage of export of spices spent in  10.35   × 100 = 64.28% chemical imports =   16.1  13. c For questions 14 to 16: Total matches played = 50 Total number of Losses = 44 + Y 11 + Z 2 Total matches played = number of loss + number of ties Total number of wins = Total number of losses 11 + Z So, 50 = 44 + Y + 2 ⇒ 2Y + Z = 1 Since Y and Z are non-negative integers. Z = 1, Y = 0 So, X = 19. Total number of Ties = 14. c 15. a 16. e For questions 17 to 19: 17. b Total oranges produced in May = 31 × 5 = 155 Checking the options, (1) 26th June Total oranges needed by Kitto = 1 + 2 + 3 + ... + 26 = 351 Total oranges produced till (and on) 26th June = 155 + 26 × 8 = 155 + 208 = 363 So, 12 oranges will be left. (2) 27th June, Kitto will be in need of 27 oranges. Taking a clue from option (1), maximum available orange = 12 + 8 = 20. So, it will run short of oranges on 27th June. 18. b Total oranges produced in May = 31 × 8 = 248 Checking the options, (1) 1st July Total oranges needed till (and on) 1st July = 1 + 2 + ... + 31 = 496 Total oranges produced till (and on) 1st July = 62 × 8 = 496 On 2nd July, kitto will obviously run start of oranges. So, option (b) is the answer. 19. e Total oranges needed till 1st July = 1 + 2 + … + 15 + 15 × 15 = 345 Total oranges produced till 1st July = 31 × 5 + 30 × 8 = 395 So oranges left on 1st July = 50. 20. d Combining I and II: There are 25 students, one girl and 24 boys in the class. Santosh’s rank is 24th in the class and since the girl is not 25th, she has to be above Santosh. Therefore, Santosh’s rank among the 24 boys is 23rd. So the answer is (d). Export earning from cotton in April in US dollar (0.27 × 220) = $1.41 million 42 Import of chemicals would cost 0.23 × 210 = Rs. 48.3 million For this to match $1.41 million, the exchange rate =  48.3   = 34.25 has to be   1.41  Explanations: Fundamentals of Logical Reasoning & Data Interpretation MBA Test Prep Page: 41 21. d Combining I and II, the code for ‘love you’ is ‘mike sika’ (from I). If we combine I with II, ‘you’ has to be ‘mika’. So ‘love’ has to be ‘sika’. The answer is (d). 22. e All that we can get after combining the information given in the two statements is that ‘God is’ coded as ‘mau pau’, but we cannot find out exact code of God. 23. c Statement I: Z has to be second. X has to be third. Y has to be first. Statement II: Y finished second. Z has to be third. X has to be first. Thus, both the statements individually give the answer. 24. e 5. a Total number of outside university students = 3 + 2 + 4 + 1 + 2 + 3 = 15 Total number of outside state students = 2 + 2 + 4 + 1 + 2 + 3 = 14 Total number of OBC students = 10 + 12 + 11 + 10 + 11 + 13 = 67 Total number of college III students: General category = 35% of 500 = 175 Others = 3 + 4 + 4 + 5 + 11 + 18 + 1 = 46 Hence, 6. d In college III, total number of handicapped, outside university, NRI and OBC students = 23 and total number of outside state, SC/ST and industrysponsored students = 23. 7. b In college IV, students admitted through quotas = 38. Students admitted through general quota = 14% of 500 = 70 Combining I and II: We known that A and C are males, and B and D are females. But we do not know the gender of E who could be a male or female. Hence, 25. e Combining I and II: Even after combining both the statements, we cannot determine who is the tallest. It could be either Rohit or Manish. When we say that Rohit is not taller than Manish, it means Rohit could be of the same or lesser height than Manish. (15 + 14 + 67 ) × 100 = 43.4% (175 + 46 ) 38 × 100 = 54.28% 70 8. d None of the above: the figures given are for increase in GDP. 9. a 1 × (1.025)4 × (1.018)4 = 1.1854 10. b It is clear from the table. 11. e Since we cannot take average value of GDP because of possibility of different GDP figures for different regions within a group. = 393 : 331 = 1.2 : 1 12. e Cannot determined because data of each region is not given. 2. d Among all the crops grown in Bihar, MP and Maharashtra, Jowar has the highest production. 13. e 3. c Production of wheat by Punjab as a percentage of Required cost = (240 × 1.5) + (1200 × 3) + (840 × 1.8) = Rs. 5,472 14. a Required average Practice exercise – D5 1. d In 1985, production of rice and wheat is 393 lakh tonne and 331 lakh tonnes respectively. Hence, the ratio of production of rice to wheat total production of India = 4. c 103 = 31.1% 331 Total number of OBC students = 10 + 12 + 11 + 10 + 11 + 13 = 67 Total number of industry-sponsored students =2+1+1+2+1+2=9 Total number of SC/ST students = 15 + 16 + 18 + 19 + 11 + 10 = 89 Page: 42 = or 15. d 48 × 1.5 + 48 × 1.6 + 48 × 2.4 + 48 × 2 + 48 × 1 ( 48 + 48 + 48 + 48 + 48) 1.5 + 1.6 + 2.4 + 2 + 1 = Rs. 1.7 5 The cost of transportation for each alternative are: (1) 300 × 2.8 = Rs. 840 (2) 200 × 3 = Rs. 600 (3) 300 × 1.5 = Rs. 450 (4) 400 × 1.1 = Rs. 440 Obviously, the cost of transportation of alternative (d) is the least. MBA Test Prep Solution Book-2 16. d Required percentage 1200 − 240 × 100 % = 400 % . = 240 Alternative method: In stead of using the formula for percentage growth = 24. d Combining I and II, we must have 6 silver and 2 bronze coins. No other possibility is there. 25. e Combining I and II, the only information we get is that C has four sisters. Now R has four siblings but how many brothers and sisters is not known. Hence, the number of uncles for K cannot be determined. So the answer is (e). Final − Initial × 100; we can use; Initial  Final  100 ×  − 1 ;  Initial  as it saves one mathematical operation of subtraction. Practice exercise – D6 1. e ≈ 9.2 : 28.2 ≈ 1 : 3 2. c Total sales = 3. b Answer is ‘The Decan Herald in 1999’. ‘Others’ is not included because it is not a newspaper. ‘Other’ here includes newspapers which are not mentioned. 4. b Deccan Herald gained Rs. 5.34 crore. 5. a Jadeja’s total = 164. Tendulkar’s average = 57.5 Hence, the ratio is 1 : 2.86. 14 220 × 7 × 100 = = 5.6% 250 × 1.1× 100 25 6. a Tendulkar’s points are 8 + 3 + 4 + 4 + 7 = 26. Dravid’s points are 4 + 10 + 10 = 24. Others are below this. 19. e 200% increase in 4 years. (From 3 crore to 9 crore or 50% average annual growth rate.) 7. a Since the total of 5 batsmen is more than 226 runs. 20. c Ratio = 165 = 55 . (Do not write data for every 3 year, try to do it mentally) This is maximum in 1996-97. 8. d Against England =  1200  − 1 × 100 = (5 − 1) × 100 = 400% So   240  17. b 1997-98, nearly 50% increase. (Can be figured out by looking at the slope) 18. a Company turnover in 1997-98 = Rs. 250 crore. Total industry turnover in 1997-98 = 250 × Total industry turnover in 1998-99 = 100 7 250 × 100 × 1. 1 7 1.32 × 100 = 17.4 crore 7.6 Company’s share in 1998-99 = Rs. 220 crore ∴ Company’s share = 21. d 22. c 23. e 18 = 18 1 31 = 7.75 Australia = 3 14 = 3.5 Sri Lanka = 4 Africa = Statements I and II alone are not sufficient to determine Ravi’s age. From statement I, we get Ravi’s age but from I, we get Ravi + 10 = 2 (Ram + 10) and from statement II, Ram = 5 years old. Therefore, Ravi = 30 –10 = 20-year-old. From statement I, 4 boys and 3 girls, i.e. every boy has 3 brothers and 3 sisters. From statement II, every girl has 3 brothers and 3 sisters ∴ The answer is (c). ‘x’, ‘y’, ‘z’ be the lengths of the ropes in the increasing order of length. Then from statement I, y + z = 10 and from statement II, x + y = 9 2 equations and 3 variables. Thus, cannot get ‘x’. ∴ The answer is (e). Explanations: Fundamentals of Logical Reasoning & Data Interpretation 38 = 19 2 71 × 200 ~ − 8 × 200 ~ − 1600 crore 9 9. e Sales = 10. b From a figure of Rs. 90 crore in 1996-97, sales touch Rs. 175 crore in 1999-2000. Since sales have less than doubled, so percentage increase < 100%. It has to be around 95%. 11. e We do not have data on market shares in 1997-98. MBA Test Prep Page: 43 12. a 40% of others = 40% of 5% = 2%, which is 1 of 4.5 18. c Since after the investment of Rs. 30,000, for every investment of Rs. 10,000, return from dairy is Rs. 900 whereas for the same amount, return from shares is Rs. 1,100. 19. b If equal amount is invested, than net return = Rs. 4,200, which is maximum in all the cases. 20. e It can be observed that if he invests Rs. 20,000 — Agriculture Rs. 10,000 — Dairy and Rs. 20,000 — Poultry Total return = Rs. 2,600 + Rs. 1,400 + Rs. 2,750 = Rs. 6,750 So, the percentage return = 13.5% 21. b Statement I gives us superfluous data. To find out the volume of the glass, we need its height and diameter. Hence, statement II alone is sufficient. 22. e No information has been given about the area or any sides of the square base of pyramid, we are only provided with distance from the square base from IInd statement. Hence we cannot determine the attitude of the pyramid by using the statements. 23. e You do not know the shape of the bathroom floor. Thus, the exact number of tiles cannot be determined. 24. e We still do not know the height of the room. 25. a Using statement I, we get 70 = k × π × ( 4.52 − r 2 ) × t k × π × 4. 5 2 × t × 100 Solving this we can find r. k is constant of proportionality to convert volume into weight. Thus answer is (a). Zenith home PC market share. 1 2 of 200 = × 200 4.5 9 = Rs. 44.44 crore ∴ Wipro home PC sales = 13. d Among all the products, E has maximum sale as well profitability. So obviously E will be having the highest profit. Sale = 60 lakh Profit = 20% 60 = 50 lakh 1 .2 Profit = 50 × 20% = 10 lakh Cost = 14. c 40 10 40 lakh × = 1.1 100 11 Profit earned by E = 10 lakh Profit earned by B = (Pr ofit )B 40 × 100 400 So (Pr ofit ) × 100 = 11 × 10 = 11 = 36.36% E 15. c Profit earned by A = 30 20 × = 5 lakh 1.2 100 Profit earned by B = 40 lakh = 3.6 lakh 11 20 15 300 Profit earned by C = lakh × = 1.15 100 115 = 2.6 lakh 50 7.5 300 lakh × = 1.075 100 86 = 3.5 lakh Profit earned by E = 10 lakh Total profit = 24.7 lakh Profit earned by D = 16. c The percentage share of the profits of A and E= ( Practice exercise – D7 1. a 15 × 100 = 60.63% 24.74 Percentage change in exports of automobiles excluding HCVs in 1993-94 to 1990-91 = 70 + 83 + 5 – 20 – 44 × 100 4234 = 94 × 100 = 2.2% 4234 For questions 17 to 20: 17. e It can be observed that shares give a flat 11% return. Now, making all the possible investment from Rs. 10,000 – Rs. 1,00,000, return in dairy does not become equal to the return obtained by shares. Short cut: Check it through options. Page: 44 ) 2. e We know the total number of two-wheelers in 1990-91 and 1993-94. But we do not know the number of two-wheelers which were exported to EU in 1990-91 and 1993-94. Hence, data is insufficient. MBA Test Prep Solution Book-2 3. d The total revenue of government through duty on exports = 33000 × 1.2 × 721 × 10 2 = 285 × 10 7 = Rs. 285 crore 4. d Revenue earned by LCVs = Rs. 1012 × 5.5 crore = Rs. 5,566 crore Revenue earned by LMCs = Rs. 823 × 4.2 crore = Rs. 3,456.6 crore Total revenue in US dollars 5566 + 3456.6 crore = $ 21.2 million = 42.5 5. d Cost of production (or) wage bill = Man-hour worked × Hourly wages × Days worked Now it is very clear from the table that year 2000 has the highest value for all the three parameters. 6. a It is clear that the least man-hours worked is in 1970 = 1300 × 12 = 15600 man-hours × 250 days 7. c 11. d Computed special price News − s tan d price for each 12. c EP (3 years) = 648 + (24 × 3) – 60 = 660 660 EP = = Rs. 18.33 issue 36 13. e 100% = 1,300 workers by working 12 hr per day can produce 550 tonnes. Hence, by working 24 hr per day they can produce a maximum of 1,100 tonnes. Hence, they can meet targets of 1980 but not of 1985. Cannot be determined as only market share is given. 15. d HP’s sales of peripherals = Hence, total market = 357 × 100 30.4 = Rs. 1174 crore 16. a Revenues from consultancy services = 8% of 6945 crore = 555.6 357 × 100 = 1,155 30.9 Hence, consultancy services as percentage of Revenues from PC servers = Special price per issue Percentage decrease from previous term 1 22 — 2 20 Approx. 9% 3 18 10% PC servers = 4 16 11.11% 5 13.5 15.6% EP = Rs. 22 Issue EP for 5 years = 810 + 0 – (12 × 25) = 510 for 60 issues. Family A yields = 15 × 14400 = Rs. 600 360 ⇒ Ratio = 480 : 600 = 4 : 5 18. e Family A spends = B spends = 48 × 4800 = Rs. 640 360 39 × 7200 = Rs. 780 360 ⇒ Required percentage = EP = Rs. 8.5 Issue Difference = Rs. 13.5 Explanations: Fundamentals of Logical Reasoning & Data Interpretation 18 × 9600 = Rs. 480 360 B yields = 9. e Effective price (EP) = 480 + 48 – 0 = 528 for 24 issues 555.6 × 100 = 48.1%. 1155 For questions 17 to 20: In the pie diagram, the expenses are given in total amount of Rs. 3,600. 17. b 10. b 2500 7 = Rs. 357 crore HP’s share of peripheral market = 30.4% For questions 9 to 12: Term 1250 × 100 ~ − 70 × 100 = 7000 crore 18 14. e Percentage increase in production is always higher than the increase in number of workers. 8. b 1250 = 18%, then MBA Test Prep 640 × 100 = 82% 780 Page: 45 19. b For family A, light = 18 × 4800 = 240 360 15 × 7200 = 300 36 ⇒ Difference = 300 – 240 = Rs. 60 (least) Practice exercise – D8 1. e 20. d 21. b 22. c 96 × 100 360 110 × 100 Percentage of expenditure of B on food = 360 ⇒ Ratio = 96 : 110 = 48 : 55 Percentage of expenditure of A on food = The factors of 630 = 2 × 3 × 3 × 5 × 7. By using the statement II, the sum of the 2 numbers is 153. Therefore, out of the factors of 630, (3 × 3 × 7 = 63) + (3 × 3 × 2 × 5 = 90) add up to give 153. Hence, the 2 numbers are 63, 90 and we can find the absolute difference between them. Hence, by using statement II only we get the answer. Hence, (b). From statement I, Required ratio = swing = 2. e No relation between voters turnout (votes polled) and percentage of swing are given. 3. c (iii) adds up to more than 200 seats. (ii) gives negative value for Socialists. 4. d For republicans to get 100 seats the vote swing must be 8.75% which is not an integer. 5. d The maximum observed value of ‘s’ is 10. Sam must have answered 10 questions at least on one day. The minimum he answers on any day is 5. From Monday to Thursday there are 14 queries from the previous week and 10 from that week. Hence in all there are 24 queries. So there is no way he could have answered 10 queries on any day from Monday to Thursday. So he must have answered the maximum (10) number of queries either on Friday or on Saturday. 6. b The queries received on Saturday have to be all answered on Saturday. On Thursday Sam has to answer atleast 1 query that was received that day and on Friday Sam has to answer at least three queries that were received on that day. All the other queries can definitely be answered with atleast a day’s delay. Hence the minimum number that has to be answered on the day of receipt is (4) + (5) + (1) + (7 + 3 + 1) = 21. For Example, they can plan their schedule in the 110 X=Y 100 90 Y 90 110 = × 100 X 100 100 Statement I alone is sufficient. 10 X 10 10 Y = × 100 11 100 Similarly, statement II alone is sufficient. From statement II, 23. d Combining I and II, we must have 6 silver and 2 bronze coins. No other possibility is there. 24. d Statement I is necessary and statement II is also needed as if we take: following manner: x 1 = <1 y 2 c = 98 ∴ x − 98 −97 97 1 = = > y − 98 −96 96 2 ∴ x− c 1 > y−c 2 This defeats the given case. Thus, both statements I and II are needed. ∴ Answer is (d). 25. e The IInd statement tells us that triangle QPR is a right angled triangle but the information is given only of perpendicular distance QR, no information is given about the base, so even by combining both the statements we cannot get the length of PR. It cannot be answered using both the statements. ∴ Answer is (e) Page: 46 If the Republicans win 97 seats, percentage of (97 − 65) =8 4 Democrats win 90 – 8(3) = 66 seats. For family B, light = Mon 8 by DI Anu 6 by LR Dudi Tue Wed 18 by Anu 6 by Dudi 18 by Anu 6 by Dudi Thu 10 by Anu 6 by Dudi QA 2 by Tiru 15 by 15 by 15 by Tiru Tiru Tiru VA 5 by Sam 5 by Sam 5 by Sam Fri 4 by Anu 1 by Dudi Sat (4)* + 2 by Anu (5)* + 7 by dudi 6 by Tiru, 5 (1)* + 9 by Dudi and by Tiru 4 by Anu 4 + (1)* (3)* + 4 by by Sam Sam (7)* + 3 by Sam ( )* indicates queries received on the same day. MBA Test Prep Solution Book-2 7. d 8. e 9. c Now from D there are again 6 ways by which A can reach his office Ao If all conditions have to be satisfied on each day atleast 21 queries have to be answered. The number of incoming queries on M, W, Th, Fri, Sat are all less than 21, hence there must definitely be some carry over from the previous day in the week for each of these days. Route DH-EO-CO-DO-AO DH-EO-DO-CO-AO DH-CO-EO-DO-AO DH-CO-DO-EO-AO DH-DO-CO-EO-AO DH-DO-EO-CO-AO Number of queries of the beginning of the week = 103. Number of queries received during the week = 107. Hence number of queries answered = (103 + 107) – 20 = 190 In all DI has (24 + 40) = 64 queries to be handled. Similarly LR has 37 queries to be handled. QA has 72 queries to be handled. VA has 37 queries to be handled. Therefore minimum distance that A can travel is 40 + 52 = 92 kms. 11. b Now range for dudi is 6 ≤ d ≤ 12; so at least on one day he answered 12 queries, so minimum number of queries answered by Dudi in the week = 6 × 5 + 12 = 42. 42 ≤ Number of queries that can be answered by Tiru in a week ≤ 77 Similarly, 58 ≤ Anu ≤ 98 and 35 ≤ Sam ≤ 55. If Dudi answers the LR queries than he has to move to other sections viz. either DI or QA. So, atmost queries of three sections could be handled by one single respondent. The three friends whose houses are at a minimum possible distance from the house of A are C, D and E. There are 6 ways by which A can visit C, D and E. Route AH-EH-CH-DH AH-CH-DH-EH AH-CH-EH-DH AH-DH-CH-EH AH-DH-EH-CH AH-EH-DH-CH 12 kms 13 kms 18 kms 12 kms 17 kms 19 kms 12. a BH  → DH  → DO  →BO 15km 12km 15km @Rs.1/km @Rs.3/km @Rs.2/km + 15 + 36 = Rs. 81 30 13. b For questions 10 to 13: 10. d AH-AO BH-BO CH-CO DH-DO EH-EO FH-FO Now combining each of the above distances with the distances given in table 2. i.e. Distance of AH from BH, CH, DH, EH and FH and so on, we find that for C, E and F the distances of their house from their respective offices is not less than the distance from the houses of any of his friends. Dudi in a week ≤ 66. As explained above, minimum number of queries answered by Tiru in the week = 25. 25 ≤ Number of queries that can be answered by Distance (kms) 54 53 52 59 60 52 Distance (kms) 48 44 40 45 39 46 Now, DH – E0 < EH – E0 and CH – E0 DH – C0 < EH – C0 and CH – C0 DH – D0 < EH – D0 and CH – D0 and for AH – CH – EH – DH the distance travelled is 40 Kms. Explanations: Fundamentals of Logical Reasoning & Data Interpretation The shortest route can be found through iteration 12km 13km 14km 16km 14km 13km D→E→F→B→C→A→D In all it takes 82 kms of travelling (or) Rs. 82/- For questions 14 to 17: The following values have been taken from the graphs for calculation Year 1995 1996 1997 1998 1999 2000 Value (Rs Cr) 400 310 150 200 590 750 Quantity (Mn kg) 205 175 130 150 265 420 14. c Value per kilogram for: 1995 = Rs. 19.50 1996 = Rs. 17.71 1997 = Rs. 11.53 1999 = Rs. 22.26 Thus, it is the lowest for 1997. MBA Test Prep Page: 47 15. c 16. e Increase in value of jute exports for 1996 = –22.5% 1998 = 33.3% 1999 = 195% 2000 = 27.11% The graph shows that the steepest incline is from 1998 to 1999. Hence, percentage change is maximum in 1999. 24. d Statement I indicates the time taken by Ravi to eat 24 eggs. Statement II indicates the time taken by Harish to finish eating 24 eggs. Together they give the answer. 25. e From statement I, we get that there are balls of 2 colours. This gives the probability of selection of Refer solution to question 14. 2000 = 17.85 1998 = 13.33 4 . However, it has no 11 information about the total number of balls. From statement II, we do not get any information about the total number of balls. the ball of other colour as Alternative method: Average Price = 19.50 + 17.71 + 11.53 + 13.33 + 22.26 + 17.85 6 = 17.75 Practice exercise – D9 1. c 17. b The price per kilogram has decreased in 1996, 1997 and 2000, has been worked out in questions14 and 16. 100 42137 Students please note that to calculate the exact value of this expression, we need calculator. Since options given are not very close to each other, we can approximate the values. And using approximations, we get the value of the 100 2650 = required ratio = (68600 – 42000) × 42000 42 = 63% = (68718 – 42137) × Alternative method: For 1995 cannot be determined, but otherwise growth is the highest in 1999 from 13.33 to 22.26. 18. e 1.25 × 10–3 times the world’s fresh cut flower exports = Rs. 100 lakh So the value of the world export of fresh cut flowers = 19. c 100 1.25 × 10 21. c 22. b 23. e lakh; 100 × 10 3 1 . 25 lakh = 8 × 104 lakh 2. c 20% of ‘Others’ = 4% of world’s export of cut flowers in 1990-91 = 17 lakh. So 4% of x = 17 lakh. ⇒x= 20. e −3 17 × 100 = 4.25 crore 4 Netherlands may be exporting flowers other than cut flowers. Using statement I and II we can independently find out the unknown side of rectangle which comes out to be ‘6’. So perimeter can be calculated by making use of both statements independently. ∴ Ans (c) From statement I, a + b = 3k but nothing can be said about C. Therefore, statement I is not sufficient. From statement II, c = 3p ⇒ 3(a + b) + c = 3(a + b) + 3p = 3(a + b + p) = 3q ⇒ 3(a + b) + c is divisible by 3. Hence, statement II alone is sufficient. The volume is unknown and the cost of each individual element except zinc is unknown. Cost of copper cannot be found. Page: 48 Required percentage growth Boo ks 1975 1980 Percentage g row th P rim a ry 4 21 37 6 87 18 6 6% S e co nd a ry 8 82 0 2 01 77 1 28 % H ig h er secon da ry 6 53 03 8 21 75 2 6% G ra du ate le vel 2 53 43 3 66 97 4 4% Hence, percentage growth is least for higher secondary books, viz. 26%. 3. b Again referring to the above table, we can see that the percentage growth rate is maximum for secondary level books, viz. 125%. 4. d It can be seen from the given table that though primary level books have shown a consistent growth, it has declined in 1978. On the other hand, even secondary and higher secondary level books have shown a consistent increase except for 1977 when it had declined. But the graduate level books have shown a consistent growth over the period. MBA Test Prep Solution Book-2 7. d For questions 5 to 8: Let us refer these hawkers as J, R, S and P respectively in place of Jayram, Rajaram, Sitaram and Peariram. From information (II), we know that P is selling Chatpata. From information (I), we gather that R do not sell either Fruits or Plastic Toys. So R must be selling Newspapers. So J and S are selling Fruits and Plastic Toys, not in any particular order. Regarding their revenue earned, we know that J + R + S + P = 1200. But from information (IV), S = 250. Therefore J + R + P = 950, or R + P = (950 – J) …(i) The amount to be paid to the hooligans by J and S are Rs. 35 and Rs. 62.50 respectively. In order to maximise their total ‘take-home’ revenue, the amount to be paid by R and P should be considered. We can observe that P has to pay a lesser percentage as compared to R. So we maximise the revenue earned by P at Rs. 249, (corresponding revenue earned by R is Rs. 351), to get the minimum amount to be paid to the hooligans. We can compile the result in the following table. Hawkers Jayram Rajaram Sitaram Peariram Revenue 350 351 250 249 earned (Rs.) Total 1200 From information (III), we can say that R+P + 50 = J 2 …(ii) Solving (i) and (ii), we get that J = 350 and (R + P) = 600 From information (I), we know that R > 300. But if, say R = 301, then P = 299, which is greater than 250. But from information (II), P earned the minimum revenue. So, P should earn less than S (Rs. 250). So, we can say that R > 350 and P < 250. We can compile the following table with this derived conclusion. Hawkers Jayram Rajaram Sitaram Peariram Items Fruits/ Plastic Toys News papers Fruits/ Plastic Toys Chatpata Revenue earned (Rs.) 350 >350 250 <250 5. b 6. e According to the question, S + P = R. We know that S = 250 and P = (600 – R) Solving we get, R = 425. Since Rajaram sold Newspapers, the answer is (b). Amount to be paid to the hooligans (Rs.) 35 70.2 62.5 37.35 205.05 Take-home' revenue (Rs.) 315 280.8 187.5 211.65 994.95 So, (d) is the right answer. 8. c As integral number of Chatpatas’ were sold so (600 - Rajaram’s salary) should be divisible by 7. Option (c) i.e. (600 –494) = Rs.106 is not divisible by 7. So Rs.494 cannot be the revenue earned by Rajaram on that day. 9. b 180 − 140 = 6.67 million tonnes (Approximately) 6 10. a In 1997, kharif crop production is 110 × 99 = 108.9 million tonnes 100 And rabi crop production in the same year is We know that either J or S is the Fruit-seller. J sold items worth Rs. 350 and S sold items worth Rs. 250. If one orange was sold at Rs. 2.50 per piece, then either 140 or 100 oranges were sold on that day by one of these hawkers. 116 × 81 = 93.96 million tonnes 100 ∴ Total production increases by 108.9 + 93.96 − 180 × 100 = 12.7% 180 11. c Given that 83% equals 110. So 100% should 110 × 100 = 132.5 million tonnes(Approx.) 83 So quantity of food grains India should have imported = 132.5 – 110 = 22.5 million tonnes (approximately) equal Explanations: Fundamentals of Logical Reasoning & Data Interpretation MBA Test Prep Page: 49 For questions 12 to 16: 14. c The maximum number of times when all the three on the podium can be from Ferrari is 6. For example, in all races from 4 to 9 the 2nd position is occupied by Ferrari. The minimum could be 0, when Ferrari occupies 2nd position in races say 1 to 3 and 10 to 12. 15. c Total number of points obtained by Ferrari team = 9 × 10 + 6 × 9 + 9 × 8 = 216 McLaren team = 5 × 10 + 2 × 9 + 5 × 8 = 108 Renault team = 3 × 10 + 0 × 9 + 3 × 8 = 54 Let’s assume the remaining podium positions were won by a single team; Y, then Y = [18 – (9 + 5 + 3)] × 10 + [18 –(6 + 2 + 0)] × 9 + [18 – (9 + 5 + 3)] × 8 = 1 × 10 + 10 × 9 + 1 × 8 = 108 ∴ The minimum difference in points between the top two teams = 216 – 108 = 108 16. a From the table it can be seen that all the three can never be simultaneously on the podium. 2 rd of the races i.e. 12 races. 3 Hence in (9 + 9) – 12 = 6 races it held both the 1st and 3rd positions. Ferrari was on the podium in McLaren was on the podium in 2 rd of the races i.e. 12 3 races. Hence since 5 + 5 + 2 = 12 in no race did it hold more than 1 position. 2 th of the races i.e. 4 races. 9 Hence it could have held the position (1, 3); (1, 3); (1); (3). Based on this the following chart could be drawn: Renault was on the podium in Race Ist 18 R IInd IIIrd 17 R R 16 R M 15 Others M R 14 M R 13 M Others 12 M F 11 M F 10 M F 9 F F 8 F F 7 F F 6 F F 5 F F 4 F F 3 F M 2 F M 1 F M The above chart represents one of the options in the races. Every other option would also satisfy the same condition. Ferrari in the 2nd position can range anywhere from race 1 to race 12 but not beyond it (since Ferrari held the podium in only 12 races) 12. c 13. b From the table we can see that there would be 6 races when both Ferrari and McLaren would definitely be on the podium. Renault had podium positions in 4 races. It won 6 podium positions. Hence it is possible only in the following cases: (1, 3), (1, 3), (1), (3). Hence the answer is (b). Page: 50 For questions 17 to 20: Suppose that the selling price is Rs. 100. 17. a 25% (of 30% (of 20%)) = 1.5%. 18. c 6 lacs = 0.6 million 19. a Appraisal cost = 6 million ⇒ Product Testing cost = 3 million. 20. b Error cost = 20% of quality cost = 1 × (20% of total cos t ) 5 = 1 1 × = 4% of total cost 5 5 21. c Statements I and II both individually give a relation between the capacities of water barrels A and B. 22. c From statement I, we easily get the answer. Since we know both the distance and speed. From statement II, x + 50 → x→ t xt = (x + 50) t 2 t = 45 2 ∴ xt = 45 45 + 25t = 45 2 ∴ t = 54 min Thus, we can find time from statement II also independently. Thus, (b). MBA Test Prep Solution Book-2 23. e From statement I, 1 1 1 + = A C 8 Case I: Vc R From statement II, 1 + 1 = 1 . Here we have two B C 6 equations and three unknowns. Hence, we need one more equation. 24. a Statement I clearly shows that x is either zero or negative, i.e. not positive. We do not get any information from statement II. Thus, (a). 25. c From statement I, the SP of a pair can be obtained while the CP can be obtained from statement II. Practice exercise – D10 1. d All are true. 2. e New demand is 114% of 175000 = 199500 pairs. So demand supply gap = 199500 – 175000 = 24500 pairs 3. a Actual demand in 1995-96 is 110% of 190 = 209000 pairs Actual production in 1995-96 is 85% of 190 = 161.5 thousand pairs So (Demand – Supply) = 209 – 161.5 = 47.5 × 1000 = 47500 pairs S If both C & D are moving in the anticlockwise direction, then … (ii) Vc – Vb = ± 40 Vb + Vd = 60 … (iii) Adding (i) and (ii) Va + Vc = 90 or 10 … (iv) Adding (iii) and (iv) Va + Vb + Vc + Vd = 150 or 70 Therefore, the average of the speeds of these cars is: 150 70 0r i.e. 37.5 mph or 17.5 mph 4 4 Case II: R If B’s speed is 20 mph. Then A’s speed = 30 mph C’s speed = 20 mph D’s speed = 80 mph Ratio of their speeds = 3 : 2 : 2 : 8 At that instant A, B, C and D passed through the points P, Q, R and S. As they passed these points, all of them were moving parallel to each other. Let their speeds be Va, Vb, Vc and Vd respectively. At that instant A & B were moving in the same directions (both clockwise), so we must have. Va + Vb = 50 …(i) Similarly C and D also moved in the same directions (either both in the clockwise direction or both in the anticlockwise direction) and their speeds with respect to B are 40 mph and 60 mph respectively. Explanations: Fundamentals of Logical Reasoning & Data Interpretation Vd F ig . C & D , b oth anti-c lock w ise Q Vb 5. a P Vb For Questions 4 to 7: 4. c Va Q Vd Vc Va P S If both C & D are moving in the clockwise direction, then Vd – Vb = +60 …(v) …(vi) Vb + Vc = 40 Solving, we get the value of the average of the speeds of the four cars i.e. 37.5 mph. Hence (a) is the only possible choice. MBA Test Prep Page: 51 6. b C observes that D is moving at a speed of 20mph. As C and D are moving in the same direction we must have: Vc + Vd = 20 From this equation, the maximum values of either Vc OR Vd can be 20mph only irrespective of the direction of C & D ...(i) Again, there are two cases: Case I: Both C and D drive in the clockwise direction. …(ii) Vb – Vc = +40 Vb + Vd = 60 …(iii) Case II: Both C and D drive in the anticlockwise direction. …(iv) Vb – Vd = 60 Vb + Vc = 40 …(v) Case II is impossible. As all the speeds are either positive or zero, equations (ii) and (iii) must be changed as following: Vb – Vc = 40 or Vb = Vc + 40 …(vi) {since Vc ≤ 20} Vb + Vd = 60 or Vb = 60 – Vd …(vii) {since Vd ≤ 20} From both equations, we get 40 ≤ Vb ≤ 60. Out of the given options, only option (ii) lies in the given range. Hence (b) is the correct choice. 7. d As E is stationary, the speeds observed by E, are the actual speeds of the cars. Let both the drivers C and D were driving with a speed of V mph. As C and D have the same speed, we need not find the solution under different cases as we did in the previous problems. We can write the following equations: …(i) Va — Vb = ± 50 …(ii) Vb + V = 60 V — Vb = ± 40 …(iii) Adding (ii) and (iii) we get: …(iv) V = Vc = Vd = 50 or 10 Subtracting (iii) from (ii) we get: …(v) Vb = 10 or 50 Putting both these values of Vb in (i), we get Va = 50 ± 50 or 10 ± 50 i.e. Va = 100, 0 or 60, – 40 As Va represents speed, it cannot be negative, hence Va = –40 is discarded. …(vi). Va = 100, 0 or 60 From (iv), (v) and (vi) we observe that Vb and Va do not have unique values rather they have more than one allowed value. As a result (Va, Vb, Vc, Vd.) can have any one of the following combinations of values: Page: 52 Case I: (Va, Vb, Vc, Vd.) have values (0,50, 10,10) OR Case II: (Va, Vb, Vc, Vd.) have values (100,50, 10,10) OR Case III: (Va, Vb, Vc, Vd.) have values (60, 10, 50,50) Options (a), (b) and (c) are not correct. If we check the options, only option (d) is correct. As V = 10 b and V = 60 (more than double) is a possible a combination of speeds. 8. b IT has the lowest acquisition value US $21.6 m. 9. b For 2001 - 03, average acquisition $ 1.6 b = $ 13.3 m 120 So 3 companies were acquired at lower than average cost - Expert Information Services, Alpharma, Dashiqiao. cost = 10. d The Australian companies were acquired at a higher value as compared to the others, Total = US $ 59.5 m Next is USA = US $ 39. 8 m, then UK and then China. 11. e London Stock Exchange has more than 15 companies listed. But how many exactly is not known. Also, a company may be listed in more than one international stock exchange. So we have no data to confirm that these more than 25 companies are all different. 12. c Top 3 acquisition value = US $ 386.4 m The total acquisition value for top 9 acquisitions in the period = US $ 461.5 m. Hence ratio = 5 : 6 For questions 13 to 15: The following table can be made: (all figures in rupee crores) 2004 2005 Revenue Profit % Profit Revenue Profit % Profit A 4600 15 600 6720 12 720 B 3900 30 900 5000 25 1000 C 2700 35 700 3840 28 840 D 6600 10 600 5000 25 1000 E 7700 10 700 8800 10 800 By the given information, A is Dixons and D is Eletropaulo. 13. a Here B is Coleco. The second highest profit in 2004 is Rs.700 crore. So both Alta Vista and Bultaco is the answer. MBA Test Prep Solution Book-2 14. b 15. d Here C is Alta Vista. The highest profit earned by any Company in 2005 is Rs. 1000 crore. So, from the choices, Coleco and Electropaulo is the answer. For questions 18 to 20: Designation Manager Assistant Manager Executive Salary 15x 12x 8x Name Anny Bobby Citra Expenditure 5y 4y 3y % increase in profit over 2004 of A: 120 × 100 = 20% 600 % increase in profit over 2004 of B: = 100 × 100 = 11.11% 900 % increase in profit over 2004 of C: = 18. d 140 × 100 = 20% 700 % increase in profit over 2004 of D: Anny does not draw the maximum and the Executive does not spend the minimum. Hence, Anny is not the Manager and Citra is not the Executive. Anny = Manager Assistant Manager 400 × 100 = 66.67% 600 % increase in profit over 2004 of E: = X1 − X5 X1 − X6 16. b 17. b X 2 − X5 X2 − X6 X2 − X 4 X2 − X 4 × Now if we analyse the options, Option (a) is wrong, since Citra cannot be the Executive. Hence, our conclusion can never be reached. Option (b) says that Bobby is the Manager. Since Citra cannot be the Executive, she has to be the Assistant Manager and therefore our conclusion is contradicted. Option (c) says that Citra is not the Assistant Manager. That means Citra is the Manager. Now, either of Anny or Bobby can be the Assistant Manager. Hence, the information in this option is not sufficient for us. Option (d) tells us that Bobby is the Executive. Since, Anny cannot be the manager, she must be the Assistant Manager. So this option is the correct choice. For questions 16 and 17: A total of 5 different arrangements is possible X1 − X3 X2 − X5 X4 − X6 X1 − X4 Citra Executive 100 × 100 = 14.28% 700 Here E is Bultaco. So C is either Coleco or Alta Vista. Hence, the data is insufficient. X1 − X4 Bobby × X3 − X 6 X3 − X5 X3 − X 6 X3 − X5 From the various possible arrangement shown above, there are two possible arrangements for (X1, X4). 19. c If the pairs (X1, X3), (X1, X5), (X1, X6), (X2, X6) or (X4, X6) are given, then all other pairs can be determined, while if the pairs (X1, X4), (X2,X4), (X2,X5), (X3,X5) or (X3, X6) are given, then all other pairs can’t be determined. Therefore required probability Total number of favourable cases 5 = = Total number of possible cases 10 Designation Manager Name Anny Assistant Manager Citra Salary 15x 12x 8x Expenditure 5y 3y 4y Savings 15x – 5y 12x – 3y 8x – 4y Manager spends the maximum. Hence, Anny is the Manager. Since Bobby is the executive, hence Citra must be the Assistant Manager Conclusion says that Citra saves the maximum. Savings = Salary – Expenditure Executive Bobby Bobby’s savings = 8x – 4y Bobby earns less than Citra and spends more than her. Explanations: Fundamentals of Logical Reasoning & Data Interpretation MBA Test Prep Page: 53 So, her savings are certainly lesser than that of Citra. Since, according to the conclusion drawn, Citra saves the maximum, we have (12x – 3y) > (15x – 5y) ⇔ 2y > 3x Therefore, if with the help of certain additional information, we can establish 2y > 3x, that will be the right answer choice. We have logically deduced that Anny is the Manager and Citra is the Assistant Manager. So, options (b) and (d) can be out rightly rejected. In option (a), Anny’s savings is twice her expenditure. Therefore, Anny’s salary must be three times her expenditure. ⇒ 15x = 3 × 5y ⇒ x = y and hence Citra is not saving the maximum. In option (c), Citra’s salary is twice her savings. Hence, her salary is also twice her expenditure. ⇒ 12x = 2 × 3y ⇒ y = 2x ⇒ 2y > 3x and hence this information is correct as well as sufficient to lead us to the stated conclusion. So, option (c) is the right choice. 20. a Therefore, Anny is the Assistant Manager and thus, Bobby is the Manager. Designation Name Salary Expenditure Manager Assistant Manager Executive Bobby Anny Citra 15x 12x 8x 4y 5y 3y 29 29 16 Savings x or y x or 4y 4x or 3y 3 4 3 Conclusion says that savings of Executive is Rs. 12,000 per month. Hence, 4x = 3y = 12000. Assuming the stated conclusion to be true, we can compile the following table. Designation Manager Manager Salary 15x Assistant Manager 12x Name Bobby Anny Citra 45000 36000 24000 Expenditure 16000 20000 12000 Savings 29000 16000 12000 Option (a) says that Executive spends Rs. 12000 per month. This enables us to find the stated conclusion. Option (b):Bobby is the Manager but monthly salary of Citra, if Rs. 36000 per month would mean that 8x = 36000 and hence, x = 4500 (misleading) Option (c): Anny is the Assistant Manager but if she saves Rs. 20000 per month, 4y = 20000 and hence, y = 5000 (misleading) Option (d): Manager’s salary = 15x Anny’s expenditure = 5y It gives x = 1600 (misleading) So, option (a) is the correct choice. Executive 8x Citra’s expenditure = 3y Case I: Citra is the Manager ⇒ 3y = 1 × 15x 2 ⇒ 2y = 5x From the first condition the number can be 97 79 88 From the second condition we get 11 22 33 : 99 ∴ From I and II combined we get number as 88. ∴ Answer is (d). 22. a Statement I implies that y is odd since the only way for the average of x and y to be a whole number is that x + y must be even. Statement II is not sufficient since either y or y + 1 must be even. Hence, x + y + 1 must be even. 23. e Since the ball that has been picked up is not identified, the probability cannot be found. 1 × 8x 2 3y = 4x If Citra happens to be the Manager, Then, Executive’s savings is either equal to (8x – 4y) or (8x – 5y), which can never be negative. But, if Citra is the Manager, then 2y = 5x. Therefore, both (8x – 4y) and (8x – 5y) become negative quantity, which is not possible. Hence, Citra has to be the Executive. ∴ Citra’s savings = 8x – 3y = 6y – 3y = 3y This saving is 25% less than the savings of the Assistant Manager. Hence, Assistant Manager must be saving 4y. ⇒ 12x – Assistant Manager’s expenditure = 4y ⇒ 9y – Assistant Manager’s expenditure = 4y ⇒ Assistant Manager’s expenditure = 5y Page: 54 21. d Case II: Citra is the Executive 3y = Executive Salary Citra’s savings = Citra’s expenditure = 1 × Citra’s 2 salary Citra’s savings is 25% less than that of Assistant Manager. Therefore, Citra is not the Assistant Manager. Designation Assistant Manager MBA Test Prep Solution Book-2 24. e Statement I gives probability of A winning but here we do not know the number of contestants. Statement II provides the information that there are only 2 contestants but we do not know the probability of a tie. Thus, answer is (e). 25. e In statement I, 2500 includes families who have neither of the devices and no data is given about such families. Hence, even statement II is not useful. 5. a Practice exercise – D11 1. b The most obvious one is rank number 5. Since H has scored less than each of A, B, C and J in all the three sections, its total also has to be less than these four students. By the same reasoning, total marks of H have to be more than D, E, F, G and I. Thus H is ranked 5th. Alternative method: ‘A’ sales has been more than all other manufacturer in all the years except 1995. 6. d Among, A, B, C and J, J could be at any of the ranks 1 to 4 i.e. J could be greater or less than each of A, B and C. Hence one cannot identify who ranks 1 to 4. By same logic between E, F, G and I, student E could be at any position from rank 7 to 10. Thus the student at rank 7 to 10 also cannot be identified. 2. a The minimum difference between marks scored by F and C in quant can be 10 and this will happen only when the difference between consecutively ranked students (F & D; D & G; G & H; H & B; B & C) is 2. (Observe that the sum total of the numbers in the denominator would be higher than ‘five times 410’ and hence, the required percentage would be a little less than 20%.) 7. c It can be easily observed that, since the difference is the highest in case of manufacturer C, the largest percentage growth would naturally occur for him, as the base is the smallest. 8. e Required ratio = (400 ÷ 500) = 0.8 9. d The figure for the highest sales of scooters over the period shown is 520 units which occurs in 1999 in case of the manufacturer A. For questions 10 to 12: Only those cities that have three roads emanating from them can be start/end city. 10. b 11. b The minimum marks that A can score is 19 in Quant, 20 in EU and 22 in DI i.e. a total of 61. The maximum marks that A can score is 25 in Quant, 23 in EU and 25 in DI i.e. a total of 73. Thus the required difference is 12 marks. 4. e As explained in answer explanation to first question of this set, the student at rank number 7 cannot be found as E could be ranked anything from 7 to 10. Explanations: Fundamentals of Logical Reasoning & Data Interpretation Three routes starting from 2 and 4 each. 12. d 13. c 3. e Required share 410   =   × 100 ≈ 19.5% 480 410 390 380 440 + + + +   D’s is definitely more than the total of F and I (as D has scored more in each of the sections compared to these students). Also D cannot be less than G (worst case scenario for D, as compared to E, is Quant -5, EU +4 and DI +2). Similarly D cannot be less than E. Thus 6th rank will be D. The other close option will be rank 10. However one cannot conclude that F will come last. E and F both could be tied for the rank 9 or 10. Consider worst case scenario for E and best case scenario for F. Thus E will score 8 more than F in EU and 2 more than F in DI. But F can make up for this difference by scoring 10 more than E in quant. Thus for rank 10, one cannot identify the student. The total sales over the period shown for different manufactures are given as follows: (a) For A = 440 + 480 + 470 + 500 + 520 + 510 = 2920 × 1000 units (b) For B = 400 + 410 + 415 + 415 + 420 + 430 = 2490 × 1000 units (c) For C = 380 + 390 + 390 + 400 + 420 + 495 = 2475 × 1000 units (d) For D = 360 + 380 + 400 + 415 + 440 + 500 = 2495 × 1000 units (e) For E = 480 + 440 + 440 + 420 + 425 + 435 = 2640 × 1000 units Thus, the sales of the manufacturer A is the highest. Total number of line employees with 3 to 5 years of employment = 140 Out of the above, the number that attended training on financial management = 80 ∴ The number of line employees with 3 to 5 years employment who did not attend training on financial management is (140 – 80), i.e. 60. MBA Test Prep Page: 55 14. b 15. a Number of employees with less than 3 years of employment who attended training on decisionmaking alone = (40 – 10) + (30 – 15) = 45 Similarly, the number of employees with less than 3 years of employment who attended training on financial management alone = (30 – 10) + (20 – 15) = 25 Answer = 45 + 25 = 70. Number of line employees with more than 5 years of employment who attended at least one programme = 50 + 40 – 30 = 60 Number of staff employees with more than 5 years of employment who attended at least one programme = 40 + 50 – 20 = 70 Total number of employees with more than 5 years of employment who attended at least one programme = 60 + 70 = 130 Percentage of employment with more than 5 years of employment who did not attend either workshop = Statement (A) indicates that the chief guest spoke longer than 25 min but does not say how much longer than 25 min. So statement (B) indicates that the chief guest spoke for lesser than 35 min. Hence, we can infer from statement (B) alone that the chief guest did not speak for more than 45 min. 24. e It is not possible to answer the question. Although (c) might seem to be a tempting offer, it is not, because we do not know whether Rashmi will go shopping if Ranjana goes. 25. d Both the statements are required, since from the statement (A), we get A +B+C B+C or 3A > A + B + C or A > 3 2 From the statement (B), we get A> A +B or 2B > A + B or B > A 2 Combining both the statements, we get B> (200 + 160) − 130 230 × 100 = × 100 = 64 200 + 160 360 16. e (50 + 40) + (40 + 50) – (30 + 20) = 130 17. d Middle management use highest percentage of spread sheet, which is 53 out of 150 = 23. b B+C < A C ∴ ‘B’ weighs most. Alternately: Statement (A) says there has to be at least one of B or C or both B and C less than A. Statement (B) says B is more than A. So combining we can say the order has to be C, A, B. 53 × 100 = 35.3% 150 18. e Option (e). Answer is 202. 19. b Top management have second highest proficiency in spread sheet. Just check the second highest value from the four of those given in the table. Practice exercise – D12 20. a 42 + 23 = 65 1. b 21. d From statement (A), he cannot watch a news channel at night. From statement (B), he can watch an entertainment channel only in the evening and night (combining with statement A). Also note, he watches only one type of channel in each of 4 parts of the day. Hence (d). 22. b Statement (A) is irrelevant. It only says Pakistan is the eventual winner. But nothing is said about points. Statement (B), explains the point system. Based on this, the maximum points possible to be scored by the winning team is (3 × 3 matches) = 9 points. Hence (b) Page: 56 79.37 − 68.75 80 − 70 1 −% = = 14.28% 70 7 68.75 2. e Demand = 1.15 × 102 = 117 MMT Gap = 117 – 98 = 19 MMT 3. e It can be checked by multiplying the All India supply figures by 0.3 and comparing it with southern India supply figures. No year satisfies that. 4. b Supply = 22 × 1.3 = 28.6 MMT Demand = 28 × 1.4 = 39.2 MMT Deficit = 10.6 MMT MBA Test Prep Solution Book-2 For questions 5 to 8: Each of the participants received at least one vote in round 1. If the minimum number of votes received is 2, then 4 of the 11 votes are accounted for, since two of the contestants were tied for the last place in Round 1. Payal has received 4 votes. Taking these votes into consideration, 8 votes are accounted for. The remaining 3 votes can be divided among the other two participants as either (3 + 0) or (2 + 1), both of which are not possible. (3 + 0) is not possible because each participant has received at least 1 vote and (2 + 1) is not possible because we have considered the lowest number of votes as 2. If the minimum number of votes received by two participants is 3 each, then including Payal’s 4 votes, 10 out of 11 votes would have been accounted for and therefore the remaining two participants cannot receive at least 1 vote each. Therefore, the only possible combination is when two participants receive 1 vote each (the minimum), Payal receives 4 votes while the other 2 participants receive 3 and 2 votes respectively. Further, one of the participants has received ‘0’ votes in round 2. i. That participant cannot be Priti because she has received 1 vote in round 2. ii. That participant cannot be Priyanka because Mr. Biyani has voted for her in round 2. iii. That participant cannot be Payal because the judge who voted for Poonam in round 1 voted for her in round 2 iv. That participant cannot be Pooja because 50% of the judges who voted for Payal in round 1 voted for Pooja in round 2. v. Therefore, it is Poonam who got ‘0’ votes in round 2. Further, it is given that the judge who voted for Poonam in round 1 voted for Payal in round 2. Therefore, Poonam would have got 1 vote in round 1. Also, Payal would have got 3 votes in round 2. (50% of votes from earlier round and 1 vote of the judge who voted for Poonam in round 1). The remaining 8 votes were divided between Pooja and Priyanka. For this to be possible and Priyanka to be joint second with another person, the only possible combination in round 2 can be: Priti : 1 vote, Payal : 3 votes, Priyanka : 3 votes, Pooja : 5 votes From condition II, Pooja got 2 additional votes in round 2. Therefore Pooja would have got 3 votes in round 1. Priyanka got 1 additional vote of Mr. Biyani in round 2 and ended with 3 votes. Therefore Priyanka would have got 2 votes in round 1. Explanations: Fundamentals of Logical Reasoning & Data Interpretation The number of votes received after the first 2 rounds were as follows: Pooja Payal Priti B'lore Delhi B'lore Priyanka Poonam Round 1 3 4 Round 2 5 3 1 2 1 1 3 0 Since Poonam was did not contest in round 3, Priyanka must be the other girl from Delhi. Further, the total number of votes in Round 3 is 13. (Poonam will also vote). From condition IV, the total votes won by Pooja and Priti (two girls from Bangalore) will be 7 while Payal and Priyanka (two girls from Delhi) together secured 6 votes in Round 3. 5. b If Priyanka received 2 votes in round 3, then Payal would have received 4 votes in round 3, because both of them together received 6 votes in round 3. 6. e Pooja was the person with the highest votes at the end of round 2. 7. d If Priti received 3 votes in round 3 and Payal received 50% of the remaining votes which is 5, then Pooja received 4 votes in round 3 while Priyanka received 1 vote in round 3. Therefore, option (d) is definitely true i.e. Priyanka received the minimum number of votes in round 3. 8. d Priti and Poonam are the 2 contestants who received the minimum number of votes (1 each) in round 1. For questions 9 to 11: Iqbal is watched by Bimal only. Beckham watches 3 movies — Hence, does not watch Iqbal which is screened at plaza. Movie Salaam Namaste Iqbal Movie Hall Satyam Chanakya Priya Plaza Bunty Ö Ö ´ ´ Babli Ö Ö ´ ´ Bonny Ö/´ Ö/´ Ö/´ ´ Bimal Ö Ö Ö Ö Beckham Ö Ö Ö ´ 9. c 10. e 11. e MBA Test Prep Page: 57 12. a Net revenue by the sale of scooters = 17. d 48 × 23784.1 = 11416.368 million 100 60 × 1307211 100 = 784326.6 Number of scooter units sold = 11416.37 × 106 784326.6 = Rs. 14,555.63 ≈ Rs. 14,500. Revenue per scooter = Alternative method: Net income of scooters 50 ~ 12000 = 24000 × 100 but it has to be less than 12,000, because both the approximate values are higher. So let us take 11,500 million. 60 = 780000 Total scooters sold = 1300000 × 100  4928   × 100 = 22 1999-2000 =   22400  18. b In 1999-2000, the value of manufactured articles and raw materials export = Rs. (22400 – 4928) = Rs. 17472 crore Since export in manufactured goods is twice that of raw materials, Rs. 17,472 has to be divided in the ratio 2 : 1, viz. export of manufactured goods = Rs. 11,648 crore and raw materials = Rs. 5,824 crore. Hence, the difference between raw materials and food = Rs. (5824 – 4928) crore = Rs. 896 crore. 19. d In 2000-01, the combined percentage of manufactured articles and raw materials = 77 and this is in the ratio 4 : 3. Hence, percentage of manufactured articles export = 44% and that of raw materials export = 33% Hence, the value of manufactured = 0.44 × 25800 = Rs. 11352 crore and the value of raw materials = Rs. 8514 crore. Hence, percentage of difference between the value of raw materials in-between 1999-2000 and 2000-01 11500 × 106 ~ 14500 Net realisation = 780000 13. a Take total sales volume as 100, then motorcycle sales volume = 22 25% increase = (25 × 22) ÷ 100 = 5.5 This 5.5 is the decrease in scooter sales volume ⇒ Percentage decrease in sales volume = (5.5 × 100) ÷ 60 = 9.1% 14. d If others is excluded, then of a total of 100, net revenue left = 89 Of this, 3 is from mopeds Hence, percentage of mopeds 3 × 100 = 3.4 % approximately = 89 15. c Net profit  20  48 + 17  25  21 + 3   =   +    × 23784.1million 100  100  100  100   = Rs. Net revenue per vehicle would be the highest for three-wheelers. (The ratio of percentage revenue and percentage volume sales is greater than 1, only for three-wheelers.) The change in the value of exports from 1999-2000 to 2000-01 = Rs. (11648 – 11352) crore = Rs. 296 crore. 21. e Combining (I) and (II): The only information we have is about the sitting positions of the highest and the lowest aged persons. Nothing can be said about the occupation of chairs 2, 3 and 4. Hence, we cannot find the age of C. 22. a Statement (I) alone: A → Q C → S B → E→ D → T B does not visit T, and E obviously cannot visit T. So D will surely visit city T. 23. e Statement A is not sufficient to answer the given question. Statement B indicates the days on which the club needs to be run but no information is provided regarding saturday and sunday. For questions 17 to 20: From the data that is given, we can find the following data (the explanation of how the following values were arrived at is given after the table): Item 1999-2000 2000-01 Food (percentage) 22% 23% Food (value) 4928 5934 Manufactured articles 11648 11352 Raw material 5824 8514 Total value of exports Rs. in crore 22400 25800 Page: 58 (8514 − 5824) = 31.6% 8514 20. a = (0.13 + 0.06) × 23,784.1 = Rs. 4,500 million 16. a Food related exports in 2000-01 = 0.23 × 25800 = 5934 So food related exports in 1999-00 = (5934 – 1006) = 4928 Hence, percentage of food related exports in MBA Test Prep Solution Book-2 24. e Statement A gives the number of users in December but does not indicate the volume. Statement B indicates some idea on the volume reduction and the number of maximum users. But, even together, they do not give any answer to the volume of water in the pool in the particular month of December. For questions 7 to 10: The following table can be drawn as per the information given. P A From statement A, R a hul 4 G au rav 2 C D Practice exercise – D13 1. a Interest earned = 7201 – 12 × 5 × 100 = Rs. 1,201 2. d Interest on 10 year 17409 − 12000 5409 ~ = −9:2 = Interest on 5 year 7201 − 6000 1201 3. b Interest on 250 monthly investment for 20 year = 130991 – (12 × 20 × 250) = Rs. 70,991 Interest on 500 monthly investment for 10 year = 87047 – 60000 = Rs. 27,047 How much less would be earned = 70991 – 27047 = Rs. 43,944 Alternative method: The total amount invested in both the cases will be same as half the amount is paid for twice the time. Thus difference in earnings will be the difference in amounts, i.e. 130991 – 87047 = Rs. 43,944 4. b 5. b 6. d Very clearly the growth has been minimum in 2 = 2.74% 1993-94 = 73 Ö Ö S T 2 Ö Ö 1 E Ö Ö Ö Ö Ö Since S cannot be supplied by 4 vendors (Q is being supplied by 4 vendors and each component is supplied by different number of vendors), hence S has to be supplied by all 5. Thus C who supplies just 1 component, supplies S. Further since Q is supplied by 4 vendors, A and B have to supply it as well. Thus table now looks like: Kamal ∴ Obviously, Rahul is older than Gaurav by 2 years. Statement B does not give any information. ∴ The answer is (b). R Ö B 25. b Q 4 A P (2+) Ö Q 4 Ö B Ö Ö ´ ´ C 1 R Ö S 5 Ö T 2 Ö ´ Ö D Ö Ö E Ö Ö ´ Ö Since P is supplied by at least 2, R has to be supplied by just 1 vendor and P has to be supplied by 3 vendors. Nothing row wise, D is the only vendor who can supply 2 parts (as all other supply more than 2). Thus table now looks as: A B P 3 Ö Q 4 Ö R 1 Ö S 5 Ö T 2 Ö Ö ´ Ö C 1 ´ ´ ´ Ö ´ D 2 ´ Ö ´ Ö ´ Ö ´ Ö Ö E Now it is obvious that P is supplied by E and thus E supplies 4 components. Only A can supply all 5 components and hence, B supplies 3. Thus final table is: P 3 Ö Q 4 Ö R 1 Ö S 5 Ö T 2 Ö Profit of banks in 1991-92 = 3% of 630 = 18.9 Profit of mutual fund in 1992-93 = 5% of 710 = 35.5 Difference = 16.6 A 5 B 3 Ö Ö ´ Ö ´ Profit to sales is maximum in 1994-95. It is 10%, otherwise in all the year, it is less than that. C 1 ´ ´ ´ Ö ´ D 2 ´ Ö ´ Ö ´ E 4 Ö Ö ´ Ö Ö Therefore, Explanations: Fundamentals of Logical Reasoning & Data Interpretation 7. a MBA Test Prep 8. c 9. d 10. a Page: 59 For questions 11 and 12: 11. b 12. b 13. b 16. c Given: B > L …(i) To be concluded: I > R In option (a), B > I and L > R. Combining it with (i), B > L > R and B > I. Therefore, conclusion cannot be confirmed for sure. In option (b), B < I and L > R. Combining it with (i), I > B > L > R. Therefore, conclusion has been confirmed. In option (c), B > I and L < R. Even if we combine it with (i), our conclusion cannot be confirmed. In option (d), B < I and L < R. Combining it with (i), L < B < I and L < R. Therefore, conclusion cannot be confirmed. Here, x2 > x ⇒ x2 – x > 0 ⇒ x(x – 1) > 0 ⇒ either x > 1 or x < 0 Our conclusion, which states that x < 0 would be true only when we come to know that other possibility, i.e. x > 1 is not the case. For that matter, if we know that x is a real number less than 1, we can say that x is not greater than 1 and hence, x < 0. So option (b) would be the correct choice. If you look at the trend, area under irrigation has been increasing over the year in minor as well as major. Only in 1974-75, in case of minor, it has decreased which suggests that some minor area has came under major. For questions 17 to 20: Australia won all 5 and Holland lost all 5. For India or Germany, 7 points can be scored as {W, W, D, L, L} or {W, D, D, D, D}. But since Australia defeated both of them, the only possibility is {W, W, D, L, L}. For Pakistan also, there has to be at least one W and at least one L ⇒ Possibility = {W, D, L, L, L} Total W = Total L ⇒ South Korea won 3 matches and lost one. Table is: P W L D Points India (Ind) 5 2 2 1 Germany (Ger) 5 2 2 1 7 Australia (Aus) 5 5 0 0 15 Holland (Holl) 5 0 5 0 0 South Korea (SK) 5 3 1 1 10 1 3 Pakistan (Pak) 3.42 0.59 − ~ 0.17 – 0.13 1970-71 = 16.9 + 2.9 3.1 + 1.4 5 1974-75 = 5 .2 0.78 − ~ 0.19 – 0.11 = 0.08 27.1 7.2 5.49 0.84 − 1975-76 = ~ 0.19 – 0.12 = 0.07 28 7.1 So, it is highest in 1974-75. 14. c Or ratio of gross cropped area to consumption of fertilizers should be highest Win Ö Loss ´ Draw ´ Holl ´ Ö ´ Pak Holl Ind + SK + Aus Ger SK Holl + Pak + Ger Aus Ind Ger Ind + Holl SK + Aus Pak Ind Pak + Holl Ger + Aus SK The matches between India and Germany as well as South Korea and Germany did not end in a draw. Hence, Pakistan and Germany match must have been a draw. ⇒ India defeated Holland and Pakistan only. 17. b The result between India and Germany must be a win in favour of Germany. 1970-71 = 174.8 174.8 = 1.11 + 3.42 + 0.59 5.12 18. a 10 173.1 ; 5.51 177 6.06 19. e 7 + 7 + 15 + 10 + 4 = 43 1971-72 = 20. c There were 2 matches which added in a draw in the whole tournament. 21. c Statement A indicates Bunty gets the middle most sequence. Statement B indicates Bunty is the last one in sequence. Hence each statement individually is sufficient. 22. d Statement A gives the number of hours Mr. Sharma walks on weekdays and Statement B indicates the number of hours he walks on Sunday. 1972-73 = 187.8 180.4 1974-75 = ; 1975-76 = 7.71 8.22 So, obviously, 1970-71 is answer. 15. e 4 Aus = 0.04 4.22 0.73 − 1973-74 = ~ 0.18 – 0.12 = 0.06 23.4 6. 1 1 7 In 1972-73 = 23.20 + 32.77 = 55.97 In 1973-74 = 24.00 + 34.20 = 58.20 More = 2.23 (Remember these figures are cumulative figures) Page: 60 MBA Test Prep Solution Book-2 23. d From statement (A), since ‘D’ always lies, B > W. From statement (B), since ‘c’ always speak truth, c = White hat. ∴ From statements (A) and (B), ‘A’ and ‘B’ have to wear black hats. ∴ The answer is (d). Persons A B C D Hat B B W W 24. e Both the statements are not sufficient since ‘A’, ‘B’, ‘C’ positions can be any of these. 15 25 A A C 15 B C 25 B 25 15 A B C And many such positions are possible. The answer is (e). 25. d Both the statements together give the length XY = 10 cm. Practice exercise – D14 For questions 1 to 5: It is given that each member represents one of the five schools. Hence, the only logic which needs to be understood is that if a particular school has won 1 red medal, the individual member winning 2 red medals cannot belong to that school. On this inference, it could be seen that Qadir, Rishi and Yousuf have 2 red medals each. Thus, they cannot be from Lucknow school or Ahmedabad school, because their respective cumulative red medal count is less than 2. On a similar reasoning, we can limit the possibilities The following table illustrate the possible linkage between the individual and the schools the represent. M e dals M e m be r s Re d Blue Pos s ible School White Gr e e n De lhi Indor e Luck now Kolk ata Ahm e dabad Puneet 1 1 1 1 Qadir 2 0 0 1 × × Rishi 2 0 0 0 × × Satyam 0 1 3 0 Tarun 0 1 1 2 × × × × Umesh 0 1 2 2 V ipul 0 1 0 1 Wasim 0 0 2 1 Xavier 0 2 1 0 Y ousuf 2 0 1 0 Zaheer 1 1 1 0 × × × × × × × × × × × × Till here, it can be seen that Satyam could be from Delhi school or Indore school. Let us assume that Satyam is from Indore school. Thus, his blue and white medals account for the entire tally of blue (1) and white (3) medals of Indore school. The remaining 2 Green and 2 Red medals of Indore school must have been won by other member(s) representing Indore school. These other member(s) surely cannot win any Blue or White medal. For any other member of Indore school, blue and white medal count should be 0 (only Qadir and Rishi). But even both of them combined cannot equal the green medal count of Indore school. Hence, this case is not possible and our assumption that Satyam is from Indore school is wrong. Therefore, Satyam must be from Delhi school. Explanations: Fundamentals of Logical Reasoning & Data Interpretation MBA Test Prep Page: 61 Putting Satyam to Delhi school, the remaining medal tally of all the schools is as follows Schools Medals Red Blue White Delhi School* 3 1 1 Green 1 Indore School 2 1 3 2 Lucknow School 1 3 2 1 Kolkata School 2 1 1 2 Ahmedabad School 0 1 2 2 *(without the medals won by Satyam) Now Xavier, who has won 2 blue medals, must be from Lucknow school. The remaining medal tally is Medals Schools Red Blue White Green 3 1 1 1 Delhi School Indore School 2 1 3 2 Lucknow School* 1 1 1 1 Kolkata School 2 1 1 2 Ahmedabad School 0 1 2 2 *(without the medal won by Xavier) Now, Lucknow’s one remaining red medal could come from Puneet or Zaheer. But Zaheer has won 1 each of red, blue and white medals and hasn’t won any green medal. If Zaheer is from Lucknow school, it won’t get just one green medal from some other member, because none of the members has won just one Green medal. Hence, Puneet is from Lucknow school. This completes the member list of Lucknow school. The remaining medal tally: Medals Schools Red Blue White Green Delhi School 3 1 1 1 Indore School 2 1 3 2 Lucknow School* 0 0 0 0 Kolkata School 2 1 1 2 Ahmedabad School 0 1 2 2 *(without medal won by Xavier & Puneet) Delhi school, which has got 3 red medals, must have got 1 red medal from a member and 2 red medals from another member. Only remaining member having 1 red medal is Zaheer. He must be from Delhi school. Remaining medal tally: Schools Medals Red Blue White Delhi School* 2 0 0 Green 1 Indore School 2 1 3 2 Lucknow School 0 0 0 0 Kolkata School 2 1 1 2 Ahmedabad School 0 1 2 2 *(without medal won by Satyam & Zaheer) Page: 62 MBA Test Prep Solution Book-2 Eliminating the ones whose schools have been confirmed and forming the table once again, M e dals Pos s ible School M e m be r s Re d Blue White Gr e e n De lhi Indor e Luck now Kolk ata Ahm e dabad Qadir 2 0 0 1 × × Rishi 2 0 0 0 × × Tarun 0 1 1 2 × × Umesh 0 1 2 2 × × V ipul 0 1 0 1 × Wasim 0 0 2 1 × Y ousuf 2 0 1 0 × × × × Only Qadir or Rishi could be from Delhi school, because remaining blue and white medals for Delhi are 0. If Rishi is from Delhi school, one remaining green medal for this school would not be possible. Hence, Qadir is from Delhi school. This completes the member list for Delhi school also. The possible schools for the remaining members would further reduce: Medals Schools Red Blue White Green Delhi School 0 0 0 0 Indore School 2 1 3 2 Lucknow School 0 0 0 0 Kolkata School 2 1 1 2 Ahmedabad School 0 1 2 2 M e dals Pos s ible School M e m be r s Re d Blue White Gr e e n De lhi Indor e Luck now Rishi 2 0 0 0 × × Tarun 0 1 1 2 × × Umesh 0 1 2 2 × × V ipul 0 1 0 1 × × Wasim 0 0 2 1 × × Y ousuf 2 0 1 0 × × Kolk ata Ahm e dabad × × × × If Tarun is from Indore school, remaining red and white medal tally for Indore school is 2,2 respectively. It could be seen that none of the remaining members have such a medal count. Even after combining the medal tally of two or more students, Red-White 2-2, combination cannot be formed. Hence, Tarun is not from Indore school. Tarun is not from Ahmedabad School either, because in that case only single White medal remains for Ahmedabad School, which is not available to any Student. Instead, he is from Kolkata school. Other member from Kolkata school could be Rishi only. This makes Yousuf a member of Indore school, because he is the only one remaining with 2 red medals. The possible schools for the remaining members would further reduce as follows: Schools Medals Red Blue White Green Delhi School 0 0 0 0 Indore School 0 1 2 2 Lucknow School 0 0 0 0 Kolkata School 0 0 0 0 Ahmedabad School 0 1 2 2 Explanations: Fundamentals of Logical Reasoning & Data Interpretation MBA Test Prep Page: 63 Medals Members Umesh Possible School Red Blue White Green Delhi 0 1 2 2 Vipul 0 1 0 Wasim 0 0 2 Indore Lucknow Kolkata × × × 1 × × × 1 × × × Ahmedabad The remaining medal tally of Indore school and Ahmedabad school are identical. Also, the total medals of Vipul and Wasim are identical to those of Umesh alone. Therefore, either Umesh is from Ahmedabad school and (Vipul + Wasim) are from Indore school or vice-versa. In summary, Delhi School Satyam, Zaheer, Qadir Indore School Yousuf, (Vipul+Wasim)/Umesh Lucknow School Xavier, Puneet Kolkata School Tarun, Rishi Ahmedabad School Umesh/(Vipul+Wasim) 1. d Option (a) is necessarily correct. Option (b) is also necessarily correct. Option (c) is necessarily incorrect. Option (d) could be correct, if Vipul and Wasim are the members of Indore school. Option (e) is necessarily correct. 2. e Either Indore or Ahmedabad. Hence, cannot be determined 3. b Three representative members. 4. a Among the options, only Puneet is from Lucknow school Page: 64 5. e Option (a) is inconsistent because Satyam is not representing Indore school. Option (b) is redundant because it is already known that Vipul and Wasim are fellows. Option (c) is consistent, but doesn’t give any data which could tell us the schools of Umesh, Vipul and Wasim. Option (d) is also consistent, but not sufficient. Option (e) clearly tells that 3 members represented Indore school. This implies that Umesh is a member of Ahmedabad school and (Vipul+Wasim) are members of Indore school. MBA Test Prep Solution Book-2 For Questions 6 to 10: Aggregate number of cars manufactured by all the companies across the twelve months of a year = 1200. Since aggregate number of cars manufactured by all the companies is same in each month, therefore number of cars 1200 manufactured in each month = =100. 12 In January, Mitsubishi manufactured 7 cars and Renault manufactured 37 cars. The 6 remaining companies have to manufacture at least 7 + 2 = 9 cars. But total cars in January = 100. Hence out of those 6 remaining companies, minimum cars any company could manufacture = 9 and maximum cars any company could manufacture = 11. Similarly, with the help of the given information, we can find the range of the number of cars manufactured by each of the companies in the 12 months. Mercedes Jan 9-11 Feb 7-26 Mar 32 April 8-19 May 27 June 7-20 July 8-23 Aug 10 -18 Sep 7-23 Oct 3 Nov 10-11 Dec 10 Mazda 9-11 7-26 9-16 8-19 6-22 7-20 8-23 10 -18 5 5-33 10-11 32 Nissan 9-11 7-26 9-16 8-19 4 40 8-23 10 -18 28 5-33 8 10 Mitsubishi 7 7-26 9-16 8-19 6-22 7-20 8-23 10 -18 7-23 38 10-11 10 Porsche 9-11 7-26 9-16 8-19 6-22 5 31 8 7-23 5-33 31 10 Ferrari 9-11 7-26 9-16 35 6-22 7-20 8-23 24 7-23 5-33 10-11 8 Renault 37 5 9-16 6 6-22 7-20 8-23 10 -18 7-23 5-33 10-11 10 Honda 9-11 34 7 8-19 6-22 7-20 6 10 -18 7-23 5-33 10-11 10 Total 100 100 100 100 100 100 100 100 100 100 100 100 The maximum and minimum number of cars manufactured by different brands alongwith their optimum index (O.I.) and average index (A. I.) is collated in the following table. Maximum Minimum Sum O.I. A.I. Ratio (O.I./A.I.) Mercedes 223 138 361 85 180.5 Mazda 236 116 352 120 176 Nissan 236 146 382 90 191 Mitsubishi 233 127 360 106 180 Porsche 235 136 371 99 185.5 Ferrari 252 135 387 117 193.5 Renault 224 120 344 104 172 Honda 214 119 333 95 166.5 0.470 0.68 0.471 0.588 0.53 0.60 0.604 0.57 6. c Maximum possible number of cars manufactured by Mitsubishi across all the 12 months = (7 + 26 + 16 + 19 + 22 + 20 + 23 + 18 + 23 + 38 + 11 + 10) = 233. 7. e Mazda is the company for which the Optimum Index is highest. 8. c Statement B and D are incorrect. 9. d The ratio of O.I. to A.I. is least for Mercedes. 10. c The minimum number of cars that a company possibly manufactured throughout the year is second largest for Mercedes. Explanations: Fundamentals of Logical Reasoning & Data Interpretation MBA Test Prep Page: 65 Maximum number of New Users who logged into the SIS exactly two times = 49 – 11 = 38. Total number of hits on the SIS on Thursday is 105 and total number of students who logged into the SIS on Thursday is 52. So a maximum of 52 New Users can possibly log into the SIS exactly two times on Thursday, this is possible when only one new User logged into the SIS exactly thrice on Thursday. Required answer = 38 + 51 = 89. For questions 11 to 15: 11. c Sunday: 131 = 25 × 5 + 2 × 3. Minimum possible number of students who logged into the website = 27. Monday: 49 = 9 × 5 + 1 × 4. Minimum possible number of students who logged into the website = 10. Tuesday: 92 = 18 × 5 + 1 × 2. Minimum possible number of students who logged into the website = 19. 14. d Since on Sunday and Thursday no Old User logged into the SIS, total number of students that logged into the SIS on the remaining five days of the week is 19 + 33 + 61 + 37 + 25 = 175. So to minimize the number of New Users that logged into the SIS, we need to maximize the number of Old Users who logged into the SIS on exactly three days. 175 = 57 × 3 + 2 × 2 Therefore at least 59 Old Users. 15. b 175 = 86 × 2 + 1 × 3. Maximum possible number of Old Users that logged into the SIS is 87. Therefore, number of Old Users that logged into the SIS lies between 59 and 87 (both inclusive). Wednesday: 157 = 31 × 5 + 1 × 2. Minimum possible number of students who logged into the website = 32. Thursday: 105 = 21 × 5. Minimum possible number of students who logged into the website = 21. Friday: 81 = 15 × 5 + 1 × 4 + 1 × 2. Minimum possible number of students who logged into the website = 17. Saturday: 63 = 12 × 5 + 1 × 3. Minimum possible number of students who logged into the website = 13. Therefore during this particular week the minimum possible number of students who logged into the website = 27 + 10 + 19 + 32 + 21 + 17 + 13 = 139. 12. b Maximum number of students who logged into the SIS on Sunday = (64 × 2 + 1 × 3) = 65 Similarly, maximum number of students who logged into the SIS on Monday, Tuesday, Wednesday, Thursday, Friday and Saturday is 24, 46, 78, 52, 40 and 31 respectively. Required difference between the maximum and the minimum possible number of students who logged into the SIS on Sunday, Monday, Tuesday, Wednesday, Thursday, Friday and Saturday is 38, 13, 27, 46, 31, 23 and 18 respectively. Therefore, on 4 days the required difference is not less than 27. 13. a Total number of hits on the SIS on Sunday is 131 and total number of students who logged into the SIS on Sunday is 49. To maximize the number of New Users who logged into the SIS exactly two times we need to minimize the aggregate number of New Users who logged into the SIS at least three times. Minimum number of New Users who logged into the SIS at least three times is when 11 New Users logged into the SIS exactly five times. Page: 66 For questions 16 to 20: Let a, b, c, d, e, f, g, h, i and j be the number of questions in sections I, II, III, IV, V, VI, VII, VIII, IX and X respectively. From the data given in TABLE 1, we can draw the following conclusions: a+b 2 = 28, a+c 2 = 21, b+c 2 = 28, a+d 2 = 26, a+e 2 = 24 a+f a+g a+h a+i a+ j = 35, = 38, = 30, = 32, = 28 2 2 2 2 2 Solving the above we get the following values a = 21, b = 35, c = 21, d = 31, e = 27 f = 49, g = 55, h = 39, i = 43, j = 35 Similarly the number of questions solved correctly in each of the given 10 sections can be easily calculated. The following table provides information about the number of questions in each section, number of questions solved correctly in each section and the difference of the number of questions and the number of questions solved correctly in each section. Sections Total I II III IV V VI VII VIII IX X Total 21 35 21 31 27 49 55 39 43 35 356 Correct 7 21 15 21 23 35 41 33 29 27 252 Total – Correct 14 14 6 10 4 14 14 6 14 8 104 Absolute Value of [Correct – 7 (Total – Correct)] MBA Test Prep 7 9 11 19 11 27 27 15 19 — Solution Book-2 16. d 17. d In section V, the difference between the number of questions and the number of questions solved correctly is the least. Number of questions solved incorrectly and number of questions left unattempted in section VI can be 3 and 11 respectively. Hence option (1) is possible. Number of questions solved incorrectly and number of questions left unattempted in section VI can be 6 and 8 respectively. Hence option (2) is possible. Number of questions solved incorrectly and number of questions left unattempted in section X can be 3 and 5 respectively. Hence option (3) is possible. If option (a) is possible, then option (e) is also 3 3 + 11 14  possible Q = = 11 11   11 Option (d) is not possible and hence the correct choice. 18. b The number of questions in sections VI, VII, VIII, IX and X is not less than the number of questions in section II. Hence there are 5 such sections. 19. c In seven sections namely I, II, III, IV, V, VI and IX. 20. b Maximum possible number of questions solved incorrectly in sections I, II, III, IV, V, VI, VII, VIII, IX and X is 1, 5, 3, 5, 5, 8, 10, 8, 7 and 6 respectively. So, total number of questions solved incorrectly = 58. Therefore, number of question left unattempted = 104 – 58 = 46. Minimum possible marks obtained by the student in the entrance test = 252 × 4 – (58 × 2 + 46 × 1) = 846 21. c Since 909909 are divisible by 9 and 567 is also divisible by 9. ∴ x is also divisible by 9. 22. e Number of books is not known. Thus, by using both statements also we do not get the answer. Practice exercise – D15 For questions 1 to 5: From information B and E, it is clear that surname of Asha is Singh and she has ordered one San Marino Imbottiti and one Sicilian Fugitive pizza. From information A, at least one of them must be in Solo size. So, possible total bill amounts of Asha Singh are Rs. 246, Rs. 372, Rs. 266 and Rs. 396 only. From information E, it is also evident that Esha’s surname is either Hazra or Malhotra. Let us assume that Esha’s surname is Malhotra. Then, from information C, the pizza, which is common between Esha and Isha, did not start with ‘M’. So the single Montreal Ham must have been ordered by Esha and from information F, this is a Solo size pizza. From information E, Esha Malhotra’s bill amount is Rs. 127 less than Asha. So this amount can be Rs. 119, Rs. 245, Rs. 139 and Rs. 269. Deduct Rs. 87 from these amounts, which Esha Malhotra must have spent on Solo size Montreal Ham. You will get Rs. 32, Rs. 158, Rs. 52 and Rs. 182 respectively, which must be the price of the other pizza Esha ordered. Out of these only Rs. 158 can be found out in the menu for a double size Raffaele’s Bianco. But, the pizza, which is common pizza between Esha and Isha, must have been ordered at least twice. So Raffaele’s Bianco cannot be the common pizza between Esha and Isha. So, Raffaele’s Bianco must have been ordered by Usha and not by Esha. Therefore, our assumption, that Esha’s surname is Malhotra is wrong. Esha’s surname is Hazra. She must have ordered Hawaiian Honeymoon in Solo size and one other pizza in Double or Family size. Again, the common pizza between Esha and Isha, must have been ordered at least twice. So Raffaele’s Bianco cannot be the common pizza between Esha and Isha. So, Usha must have ordered Raffaele’s Bianco. From information F, Usha must have ordered Raffaele’s Bianco in any size other than Double. Now, Usha’s surname is either Gupta or Malhotra. 24. d Statement I: (c + d)(c – d) = 5. This data is not sufficient independently. Combining with (II) we get c + d. Let us assume that Usha’s surname is Gupta. Then she must have ordered Godfather’s Choice in Double size. So, she must have ordered Raffaele’s Bianco in Solo size. So, possible total bill amounts of Usha Gupta are Rs. 214 only. From information E, Usha Gupta’s bill amount is Rs. 81 more than Esha. So Esha’s bill amount can be Rs. 133 only. Deduct Rs. 87 from these amounts, which Esha Hazra must have spent on Solo size Hawaiian Honeymoon. The remaining amount is Rs. 46, which must be the price of the other pizza Esha ordered. There is no pizza with this price tag in the menu. 25. c Using statement I, the orthocentre lies outside the triangle only in the case of an obtuse triangle. Thus, ABC is not an equilateral triangle. Using statement II, only in the case of an equilateral triangle does the circumcentre coincide with the orthocentre. Thus, ∆ABC is an equilateral triangle. Therefore, our assumption, that Usha’s surname is Gupta is wrong. Usha’s surname is Malhotra and Isha’s surname is Gupta. Usha must have ordered Raffaele’s Bianco in Solo size and Montreal Ham in Double. So her total bill is Rs. 245. That means Asha’s total bill is Rs. 372. So she must have ordered San Marino Imbottiti in Solo size and one Sicilian Fugitive pizza in Family size. 23. c Solving the given data x +y x − y y x 1 x y 1 + = + = x + = + y x y x y y x y x Thus, both statements can give answers independently. Explanations: Fundamentals of Logical Reasoning & Data Interpretation MBA Test Prep Page: 67 Now Isha Gupta must have ordered one Godfather’s Choice. Let us assume this was in Solo size. She has ordered another pizza common with Esha. We know, that Esha Hazra must have ordered one Hawaiian Honeymoon in Solo size and one other pizza in Double or Family size. The price of Godfather’s Choice is Rs. 12 less than Hawaiian Honeymoon in Solo size. But, from information E, Ms. Gupta’s total bill was Rs. 81 more than Esha. So, the other pizza must have created the difference of Rs. 93. But there is no such pizza, for which the difference of price for Double and Family size is Rs. 93. So our assumption was wrong. Next, let us assume that Isha Gupta ordered one Godfather’s Choice in Double size. That means Isha must have ordered the other pizza in Solo size. The price of Godfather’s Choice in Double size is Rs. 20 more than Hawaiian Honeymoon in Solo size. So, the other pizza must have created the difference of Rs. 61. But there is no such pizza, for which the difference of price for Solo and Double/Family size is Rs. 61. So our assumption was again wrong. So, Isha must have ordered one Godfather’s Choice in Family size. That means Isha must have ordered the other pizza in Solo size. The price of Godfather’s Choice in Family size is Rs. 118 more than Hawaiian Honeymoon in Solo size. So, the other pizza must have created the difference of Rs. 37. Only for Verona’s Army, the difference of price for Solo and Double size is Rs. 37. Now, finally we can collate the derived information in the following table, which can help us to answer all the questions: Name Asha Esha Isha Usha Surname Singh Hazra Gupta Malhotra Verona's Army Raffaele's Bianco Pizza 1 Pizza 2 Type Size Solo Solo Solo Solo Type Sicilian Fugitive Verona's Army Godfather's Choice Montreal Ham Size Family Double Family Double 372 262 343 245 Total Bill (Rs.) 1. c 6. b 7. a 8. d San Marino Imbottiti Hawaiian Honeymoon 2. a 3. b 30+25+40+5 for reminder mail, i.e. 100% space is used ∴ free space is 0% The deleted mails will go to trash and would still occupy space. So all will bounce. On day 3, 20 MB of space will be created in trash folder as mails deleted on day 1 will be automatically cleared. So, none of the mail will bounce. For questions 9 to 14: The maximum possible lecturers were short-listed. Since there are 5 age groups and 4 different disciplines; and there can be at most two lecturers of a particular age group representing a particular discipline, the number of lecturers short-listed = 5 × 4 × 2 = 40. Number of them selected = 5 + 20 = 25. Now, (i) Tells that out of these 25, the number of lecturers from various age groups is: Page: 68 4. c 5. e Young: 5 Middle-aged: 5 Senior: 4 Stalwarts: 4 Retired: 7 TOTAL 25 Now coming directly to (iii), Phys ics Che m is try M athe m atics Biology Young Young Young Young Middle-aged Middle-aged Middle-aged Middle-aged Senior Senior Senior Senior Stalw art Stalw art Stalw art Stalw art Retired Retired Retired Retired MBA Test Prep Solution Book-2 Note that the column for Mathematics is complete. And in Biology, count for Retired can be 2 at max. And there is no provision for Middle Aged in Biology. Each of them is having at least one lecturer. So assigning one to each of them, the table looks like: Physics Chem istry Mathem atics Biology Young (1) Young (1) Young (1) Middle-aged Middle-aged Middle-aged (1) (1) (1) Senior Senior (1) Senior (1) Middleaged Senior (1) Stalw art (1) Stalw art (1) Stalw art (1) Stalw art Retired (1) Retired (1) Retired (1) Retired (1) Thus, out of the two balance retired lecturers, at most one can go in Biology and in that case, the remaining retired and middle aged ones must go into Physics or Chemistry or both. Young (1) 9. d The balance lecturers are: Young: 1 Middle-aged: 2 Senior: 1 Stalwarts: 1 Retired: 3 TOTAL 8 Case I: Middle-aged lecturer goes into Physics. Now, out of the remaining two retired lecturers one has to go into Physics and the remaining one can go either in Chemistry or Biology. Going back to (ii), total number of lecturers for physics, chemistry and biology can be counted to lie between 12 and 20. Moreover, the total is a square. ⇒ Total in these three is 16 and there are 9 lecturers from mathematics stream. Proceeding, (iv) tells us that Rocky and Platy are Young physics lecturers. So the balance Young lecturer goes in Physics. And since none of the cells can have more than 2 and total for Mathematics is 9, we get a better version of the table as: Case II: Retired lecturer goes into Physics. Here also, the count for retired in Physics is full (2). Middle Aged cannot go in Biology and if he goes into Chemistry, the initial condition that "the number of Physics lecturers is greater than the number of Chemistry lecturers" can never get satisfied. So the middle aged goes into Physics and we get the same configuration as in case I Physics (7) Young (2) Chem istry (5 or 6) Young (1) Mathem atics Biology (9) (3 or 4) Young (1) Young (1) Middle Aged (2) Senior Middle Aged (1) Senior (1) Middle Aged (2) Senior (2) Middle Aged Senior (1) Stalw art (1) Stalw art (1) Stalw art (2) Stalw art Retired (2) Retired (1) Retired (2) Retired (1) The one remaining retired lecturer goes into either Chemistry or Biology. Hence the number of retired Chemistry lecturers cannot be determined. Physics (5 or Chem istry Mathem atics Biology (3 m ore) (5 or m ore) (9) or m ore) Young (2) Young (1) Young (1) Young (1) Middle Aged (1) Senior Middle Aged Middle Aged (1) (2) Senior (1) Senior (2) Middle Aged Senior (1) Stalw art (1) Stalw art (1) Stalw art (2) Stalw art Retired (1) Retired (1) Retired (1) Retired (2) 10. b It is already explained that the middle aged lecturer and at least one retired lecturer goes into Physics or Chemistry or both, the total is at least 12 and at most 13. Only option (b) is correct. 11. d Since Kandy is the lone retired Chemistry lecturer, the balance 2 retired Chemistry lecturers go into Physics and Biology (one each). So there are two retired Biology lecturers for sure. 12. c At least one of the retired lecturer must go into one of Physics or Chemistry. Thus the count for both of them cannot be one. Option (c) is not possible. And the balance lecturers now: Middle Aged: 1 Retired: 2 TOTAL 3 As of now, minimum lecturers for Physics as well as that of Chemistry are 5. But since the number of Physics lecturers is greater than the number of Chemistry lecturers, at least one of the balance three must go into Physics. Explanations: Fundamentals of Logical Reasoning & Data Interpretation MBA Test Prep Page: 69 13. d 14. a 15. a 16. b Laxman has scored 3,000 runs and has taken 60 catches. He has not taken any wickets. Even if we assume that he has scored all runs in centuries, then also his points would be: Runs → 3,000, Catches → 180, Centuries → 30 × 50 = 1,500 Total points 4,680, so grade E. Zaheer has scored 1,000 runs and taken 50 catches. He has taken 150 wickets (half of Kumble). 8 five wicket hauls. If we assume that he has scored all runs in centuries, then his maximum points would be: Runs → 1,000, Catches → 150, Centuries → 10 × 50 = 500, Wickets → 3,000, Five wicket haul → 400 Total points, 5,050 , so at the most he could be in grade D. Ganguly has scored 10,000 runs and taken 50 wickets. If we assume that he has scored all runs in centuries, then his points would be: Runs → 10,000, Centuries → 100 × 50 = 5,000, Wickets → 1,000 Total points 16,000, so grade A. Sehwag could score maximum 7,999 runs and taken 50 catches. He has taken 40 wickets. And taken 4 five wicket hauls. He has scored 10 centuries. His points would be: Runs → 7,999, Catches → 150, Centuries → 500, Wickets → 800, 5 wicket haul → 200 Total points 9,649, so he cannot be in grade B. For questions 17 to 19: 18. d If it breaks away at least five members, the total strength gets reduced to 495, 60 abstain (40+20) so for simple majority you need half of 435, i.e. 218, Congress has backing of 220. CPM's support would definitely be another way. 19. d If members of BJP, Others, CPM and RJD support a Congress led government then it would have a 75% majority. Hence four parties. 20. b Statement I is not sufficient as there are more than one sequences satisfying the question condition. Statement II is sufficient to answer the question. Since the first term is 0, the 3rd term will be 6. 21. a Statement I alone gives the relationship the three shapes in terms of size and is enough to answer the question. Statement II gives the data to find the area of the circle but not for the two others. Thus, the answer is (a). 22. c ‘C’ is a set of integers, and ‘x’ an element in C. Then from statement I, (x + 3) can also be an element in ‘C’ if we take x as negative integers and hence the condition is fulfilled. Similarly, with the help of statement II alone we can get the answer. 23. e There can be many different relationships possible. So statement I does not provide an answer. Statement II gives us a very general piece of information. The two statements combined together are not sufficient. 24. d From statement I, we get 15x + 20y = 130 The only possibility for x and y are (6, 2) and (2, 5). From statement II, we get x = 2 and y = 5, i.e. we have 5 tickets of $20 and 2 tickets of $15. Thus, the answer is (d). 25. d From statement I, we get the lowest = 20, highest = 10 From statement II, we get the two series are in AP. 20, 23, 26, 29, ..., 80 UPA (30 + 5 + 7)% = 42% If Congress led ⇒ 42% + 2% (of CPI) = 44% NDA = (20 + 2 + 2)% + 10% (“Others”) = 34% 17. d (i) –(8% (of SP) + 4% (of BSP)) = –12% (ii) 34% (of NDA) + 5% (of RJD) = 39% (iii) 55 members ⇒ –11% If all (i), (ii) and (iii) happen, then NDA = 39% which is more than 50% of (100 – 12 – 11) = 77% Page: 70 ∴ Average = MBA Test Prep 80 + 20 = 50 20 Solution Book-2 Explanations: Ratio & Proportion Solutions (Non MCQ) 7. Let father’s present age be F. Son’s present age be S. ∴ (F – 5) = 5[S – 5] and (F + 2) = 3(S + 2) or, F – 5S = –20 and F – 3S = 4 ∴ 2S = 24; S = 12 and F = 40 ∴ Ratio of ages of father and son = 10 : 3. 8. Mohan’s expenditure = 4x Mohan’s savings = x ∴ Income = 5x New income = (5x)(1.2) = 6x; new saving = 1.12x New expenditure = 4.88x Percentage increase in expenditure Level – 1 1. 2. (54 – x) : (71 – x) : : (75 – x) : (99 – x) Solving, x = 3 Ratio = 1 1 1 : : = 15 : 10 : 6 2 3 5 or first part = 15 × 465 = 225 31  4.88x − 4x  =   × 100 = 22%. 4x   10 × 465 = 150 Second part = 31 6 × 465 = 90 Third part = 31 Hence, the three parts are 225, 150, 90. 3. Ratio of first, second and third is 2x : 2x – 10 : 5x and 2x + 2x – 10 + 5x = 170 9x = 180 x = 20 So the three parts are 40, 30, 100 respectively. 4. If numerator = x and Denominator = 3x ∴ Fraction ⇒ 5. 6. x+y 1 = 3x + 3y 3 9. 3 4 12 , i.e. 3 : 2 : 3 : : 1 2 4 ∴ 3x + 2x + 3x = 1800 8x = 1800 ⇒ x = 225 Value of 50-paisa coin = 2 × 225 = 450 and number of 50-paisa coins = 450 × 2 = 900 coin is A 10. The angles of the triangle are If ∠A = 4x; ∠B = 5x; ∠C = 5x – 30 ∠A + ∠B + ∠C = 180 ⇒ 4x + 5x + 5x – 30 = 180 ⇒ 14x = 210 ⇒ x = 15 So ∠A = 60°, ∠B = 75°, ∠C = 45° 11. Let the initial number of employees = n. 15 x. 14 Ratio of total wages before and after the change  8  15  x  = 21 : 20. = (nx) :  n   9  14  The ratio of volumes of two cylinders is x : y and heights are p : q. 2 r 2 xq V1 πr1 p x d 2 xq = = = 1 = ⇒ 12 = 2 V2 πr 2q y py py d2 r 2 2 8 n 9 Let initial wages = x. New wages = ∴ C B Ratio of number of coins = 4 : 5 : 6 Suppose we have 4, 5 and 6 coins of Re. 1, 50-paisa and 25-paisa respectively. Then the total value = Rs. 8. Since the aggregate sum is Rs. 32 (4 times the sum that we have arrived at), the number of coins of the denomination Re. 1, Re. 0.5 and Re. 0.25 must be 16, 20 and 24 respectively. New number of employees = Ratio of values of 1 rupee, 50 paisa and 25-paisa 12. The given expression can be written as Q–p Q–q Q–r = = =k 2 5 7 p = Q – 2k; q = Q – 5k; r = Q – 7k So, Q = Explanations: Fundamentals of Ratio & Proportion Q – 2k + Q – 5k – Q – 7k 2 MBA Test Prep Page: 71 2Q = 3Q –14k Q = 14k p = 12k; q = 9 k; r = 7k; ∴ p : q : r = 12 : 9 : 7 13. Alternative method: There is a loss of 4 kg on replacement of 12 kg chair by 8 kg. This loss will be equally compensated between all the four chairs. So, there is a decrease of 1 kg in average. Ram Incomes 8x 11x Expenditures 7y 10y Savings 500 500 ∴ 8x – 7y = 500 ⇒ 19. When the same number is added to all the quantities, the average increases by the number added. Therefore, increase = Rs.1,000 20. New average will also increase by 20% of the present average. Therefore, new average = 15,000 + 3,000 = Rs. 18,000 21. Turnover from January to March = 12 × 3 = 36 Turnover from March to June = 14 × 4 = 56 Turnover from January to June = 36 + 56 – Turnover of March. Also average turnover from January to June is 12. ∴ 36 + 56 – Turnover in March = 12 × 6 So turnover in March = Rs. 20 crore. 22. If John leaves the group, number of students in the group will be 3. So average height Shyam 88x – 77y = 5500 11x – 10y = 500 ⇒ 88x – 80y = 4000 ∴ 3y = 1500, y = 500 ⇒ x = 500 ∴ Incomes of Ram and Shyam are Rs. 4,000 Rs. 5,500 their expenditures are Rs. 3,500, Rs. 5,000 respectively. 14. If B joined for t months then profit ratio = (450 × 12) : 300t = 2 : 1 ∴ ∴ 15. When A gets 60 points, B gets 40 points When A gets 60 points, C gets 45 points ∴ When C gets 45 points, B gets 40 points 1 When C gets 60 points, B gets 53 points 3 ∴ 16. 17. 18. 450 × 12 = 2 ⇒ t = 9 months 300t B joined after 3 months. C gives B 6 2 points in 60 3 = 23. Let Tom’s score be x.  72 × 4 + x  ∴  = 70 ⇒ x = 62 5   24. Time t = [20 men × 10 days × 5 hr ] = 25 days. [15 men × 8 hr ] 3 20 × 10 × 5 × 1.25 15 × 0.75 × 8 = 13.88 days (approximately). Multiplication by 1.25 is because of 25% extra work load and 0.75 accounts for the efficiency. 168 × 4 − 170 = 167.37 cm (approximately) 3 Cost of 20 kg apples = 20 × 40 = Rs. 800 Cost of 10 kg apples = 10 × 50 = Rs. 500 Total cost of 30 kg = 800 + 500 = Rs. 1,300 ∴ Average cost = 1300 = Rs. 43.33 per kilogram 30 Time t = 25. Time taken to ride 20 km at 10 km/hr = Time taken for another 20 km at 5 km/hr = 10 + 8 + 12 + 6 = 9 kg 4 Now a 12 kg chair is replaced by an 8 kg chair = Average speed = 40 = 6.67 km/hr 6 Average speed = Total dis tan ce Total time ∴ Average = 26. = Page: 72 20 = 4 hr 5 ∴ Total time taken = 6 hr Total distance covered = 40 km Average weight of the chairs 10 + 8 + 8 + 6 = 8 kg 4 i.e. a decrease of 1 kg in average. 20 = 2 hr 10 10 + 20 = 10 km / hr. 2 +1 MBA Test Prep Solution Book-2 27. Let total number of units = 100 Revenue when 50 units are sold at Rs. 20 per unit = Rs. 1,000 Revenue when 30 units are sold at Rs. 40 per unit = Rs. 1,200 Total price = Rs. 2,200 Average price = 28. 31. 2200 = Rs. 27.50 50 + 30 Averag e percentage If 2 friends leave the group, then 3 are left. New average = To a mixture containing 10% water, pure water (100%) is mixed to get the resultant solution containing 20% water. Applying the rule of alligation, average concentration of constituents 10% 100% Resu ltant average (6.5 × 5 ) − (7 × 2 ) = 6.16 kg 20% 3 Q uantities 10 80 29. Total score Average = Number of students Ratio of the volumes = 8 : 1 or 40 : 5. ∴ To 40 L of solution, 5 L of water must be added. Alternatively: 36 L of milk in the original mixture now becomes 80% of the mixture. Hence, the total volume of the new solution = 45 L. So the extra 5 L must be the water that was added. 30 × 70 + 20 × 60 = = 66% 50 30. Method 1: 7 . 10 Fraction of gold in first alloy = Fraction of gold in second alloy = 7 . 20 32. CPII 3 15 – 0 = = 0 – (–10) CPI 2 Applying rule of alligation, Averag e fractio nal concen tration of the con stituen ts 7 10 Using alligation, = 7 20 CPII Cost price of second shirt = CPI Cost price of first shirt and CPI + CPII = 900 Resu ltant averag e ∴ CPI = Q uantities 7 1 1 3 CPII = × 900 = 540 5 7 ∴ x – 10 = 20 − x ⇒ 2 × 900 = 360 5 7 7 + 7  3  21 20 10 x= =  = 2 2  20  40 ∴ Ratio of gold to copper = 21 : 19. Method 2: The first alloy is a 70% gold alloy and the second is a 35% gold alloy. If the two are mixed in equal quantities, the average concentration of gold in the 70 + 35 = 52.5% . 2 Therefore, the ratio of gold to copper is 52.5 :47.5 = 21 : 19. resulting alloy is Explanations: Fundamentals of Ratio & Proportion 33. 1 th . 6 Fraction of water in pure water (added) = 1. Fraction of water in the adulterated milk = Fraction of water in resultant mixture = 3 th . 8 ∴ Apply rule of alligation. Averag e fractio nal constitu tion of the constitu en ts 1 6 3 8 Resu ltant averag e Q uantities MBA Test Prep 1 5 8 5 24 Page: 73 Ratio of the volumes of the solution to the water added = 3 : 1 = 66 : 22. ∴ 22 kg of water should be added. 3 ∴ A woman gets 5 of what a man gets ∴ 6m + 12w + 17b Alternative method: 55 kg of milk is now 8 × 55 = 88 kg. Hence, new weight = 5 Hence, the amount of water added = 88 – 66 = 22 kg. 34. = 6m + 12 × 5 th of the new mixture. 8 or m = Rs. 2.5. Similarly, b = Rs. 1, w = Rs. 1.5 m, b, w are shares of a man, a boy and a woman, respectively. 37. 1 2 3 4 a = b = c = d 2 3 4 5 Then 10 7 7 is the ratio of and , 9 9 10 which are the average milk content in the initial and the final mixtures.) ∴ a + 4 a + 3 a + 8 a = 7300 i.e. b = Method 1: Let CP of 1 L of milk = Rs. 100. He mixed x litres of water. SP of 1 L of mixture = 100. ∴ SP of (1 + x) L of mixture = 100(1 + x). Percentage of water = 38. Let the worth of the properties of A, B and C be 2x, 3x and 5x respectively. Total = 10x 1 1 (3x) = x and (3x) = x to C. 3 3 A has 3x, B has x, C has 6x ∴ C sells 1 3 (6 x ) to A = x 4 2 3x 9 = x = Rs. 18,000 2 2 Hence, x = Rs. 4,000 ∴ Total worth of properties = 10x = Rs. 40,000. ∴ 39. A has now 3 x + Let A, B and C invested Rs. 5, Rs. 6 and Rs. 8, respectively for a, b, and c months. Then 5a : 6b : 8c is the ratio of profits which is given to be 5 : 3 : 1. ∴ 5a : 6b : 8c = 5 : 3 : 1. 1 1 ∴ a : b : c = 1 : 2 : 8 = 8 : 4 : 1. 2 men = 5 boys and 3 boys = 2 women 2 of what man gets. 5 1 woman gets 1× 3 or 1.5 times of what a boy 2 40. Total amount deducted is 15 + 10 + 30 = 55. Amount left = 535 – 55 = 480. Now Rs. 480 is divided in the ratio 4 : 5 : 7. So after deduction share of A = 4 × 480 = 120 , 16 share of B = 5 × 480 = 150 , 16 gets. Page: 74 5 B sells to A = Level – 2 ∴ 1 boy gets 2 ⇒ a = 2400, b = 1800, c = 1600 and d = 1500 0.25 × 100 = 20% . 1.25 Method 2: If the total cost of 100 L of milk is Rs. 100, the milkman is making a revenue of Rs. 125. Since he is selling at the same cost price, he must actually be selling 125 L. Hence, he is adding 25 L of water or 20% of the mixture is water. 5 3 2 a, c = a and d = a 8 4 3 3 SP − CP 100x × 100 = × 100 = 100x = 25 P% = CP 100 ∴ Volume of mixture = 1 + 0.25 = 1.25. 36. Assume that the persons in 4 battalions are a, b, c and d. 10 times the original 9 volume. Hence, if the original volume is 729 ml, the new volume = 810 ml. The increase is due to the addition of 81 ml of water. The volume of the solution is (Hint: the factor 35. 2 3 m + 17   m = 20m 5 5 MBA Test Prep Solution Book-2 7 × 480 = 210 . 16 ∴ Initial share of A = 120 + 15 = Rs. 135. Initial share of B = 150 + 10 = Rs. 160. Initial share of C = 210 + 30 = Rs. 240. share of C = 41. Ratio of contributions expected from the three men = (110 × 6) : (50 × 9) : (440 × 3) = 22 : 1 5 : 44. 15 47. 5 ∴ Second man must pay 81 th or 27 th of the total. 42. B should get = 44. 2 leaps of the dog = 5 leaps of the hare or 1 leap of the dog = 2.5 leaps of the hare ∴ 6 leaps of the dog = 15 leaps of the hare ∴ Ratio of leaps of dog to hare = 15 : 4. 48. 49. Total weight Number of students If water weights x kg/unit volume weight of gold = 19x/unit volume weight of copper = 9x/unit volume ∴ Weight of the mixture desired = 15x/unit volume Apply rule of alligation: Averag e of g old and cop per In a race of 100 m, A travels 100 m while B travels 95 m. This means in a race of 500 m, when A travels 19x 9x Q1 Q2 Resu ltant averag e Q uantities 500 = 475 m. 200 Hence, when B travels 475 m, C travels 475 × 19 x – 15 x 475 = 500 451.25. Therefore, when A travels 500 m, C travels 451.25 m. This means, A can give C a lead of 48.75 m. Please recollect you would have solved this problem in the chapter dealing with SI and CI. It is repeated here to explain solving the problem using alligation. On an average Shiv pays a interest rate of 7.2%. Explanations: Fundamentals of Ratio & Proportion Average = 62 × 5 − 70 + x ⇒ x = 60 5 Weight of new student x = 60 kg 500 500 m, B travels 95 × = 475 m. 100 Similarly, when B travels 200 m, C travels 190 m. So 46. Total weight = 25 × 10 kg 2 cases of weight 30 kg each are replaced by 2 cases of weight 32 kg each. ∴ Total weight = (25 × 10) – (2 × 30) + (2 × 32) = 254 kg Number of students = 10 So 60 = 13x + 100% of 13x 3 = 17x + 2500 5 if B travels 500 m C travels 190 × 2 × 10000 = 4000 5 Average is decreased by 2 kg. ∴ New average = 60 kg 26x 3 = 17x + 2500 5 130x = 51x + 7500 79x = 7500 x = 94.93 So the original prices are 13 × 94.93= Rs. 1234.08 and 17 × 94.93 = Rs. 1613.81. 45. Thus amount lent at 6% = 254 = 25.4 kg 10 ∴ The average will change by 0.4 kg. Let the original prices by 13x and 17x. So ratio for new prices = 8 − 7.2 0.8 2 = = 7.2 − 6 1.2 3 ∴ Average =   5100 × 3  (3200 × 4) + (5100 × 3) + (2700 × 5)  1248 = Rs. 459   43. Using alligation, we have 15 x – 9 x 50. = Q2 Q1 ⇒ 4 6 = 2 3 Let the cost price of first variety be Rs. 2x per kilogram. So the cost price of second variety becomes Rs. x per kilogram. Selling price of mixture is Rs. 36 per kilogram after making profit of 20%. So the cost price of mixture = MBA Test Prep 36 = 30 per kilogram 1.2 Page: 75 Applying the principle of alligation, 2x – 30 7 = 30 – x 3 6x – 90 = 210 – 7x 13x = 300 x = 23.07 So the cost price of first variety = Rs. 46.14 and cost price of the second variety = Rs. 23.07. 53. Applying the rule of alligation, Averag e percentage 12% –8% Resu ltant averag e 11% Q uantities 1 19 51. 3 Ratio of copper in first alloy = . 9 The ratio of the items that he is selling at a profit of 12% and at a loss of 8% is 19 : 1. 5 Ratio of copper in second alloy = . 9 Hence, he sold 1 Ratio of copper in new alloy = . 2 and Applying the principle of alligation, 54. 5 – 9 1 – 2 ⇒ 1 2 = QI Quantity of first alloy 3 QII = Quantity of second alloy 9 Apply rule of alligation for fractional water content. Q uantities 1 × 64 = 16 kg 4 3 × 64 = 48 kg . 4 1 5 8 then the original amount of the mixture = 5 . 13 Milk proportion in the total solution of second variety (VI ) = 9 . 13 7 . Milk proportion to total in the required mixture = 13 Applying the principle of alligation, 11 56 3 8 The ratio of the volumes of the alcohol mixture to water = 21 : 11. If only 8 L of water was added, Milk proportion in the total solution of first variety (VII ) = 3 7 Resu ltant averag e and quantity of second alloy = 52. 1 × 60 = 3 pens at a 8% loss. 20 Averag e fractio nal constitu tion of the constutu en ts QI 1 = QII 3 ∴ Quantity of first alloy = 19 × 60 = 57 pens at a 12% profit 20 Alcohol content in that = 55. 21× 8 . 11 4 21× 8 96 × = . 7 11 11 Let the volume of the mixture be x. Then 7 5 x, amount of B = x. 12 12 When 9 L of mixture is taken out, amount of A = 7 21 L; ×9 = 12 4 9 7 – 13 13 = VI 7 5 VII – 13 13 amount of A withdrawn = VI 2 1 = = VII 2 1 5 21 x+ . 12 4 (Since all the milk that was removed was substituted by water.) Page: 76 amount of A left in the mixture = 7 21 x− . 12 4 ∴ Final amount of B in the mixture = MBA Test Prep Solution Book-2 x – 98 5 = 100 – x 3 8x = 794 x = Rs. 99.25 per litre ∴ Cost of first solution = 99.25 + 2 = Rs. 101.25 per litre and cost of second solution = Rs. 99.25 per litre. 21   7 x−   4  7  12 = , x = 36. Now ratio of A and B = 21  9  5 x+   4   12 ∴ Amount of A = 7 × 36 = 21 L. 12 Alternative method: Use rule of alligation: After 9 L is taken out, the ratio of A and B is 7 : 5. To this is added pure B to obtain a solution containing A and B in the ratio 7 : 9. Considering the rule of alligation for fractional B content, Averag e fractio nal constitu tion of the constitu en ts 5 12 58. 118 = 59 calories . 2 Use alligation. = 104 – 59 Vm Volume of mango juice = = 59 – 54 V0 Volume of orange juice 1 9 16 Resu ltant averag e 7 16 Q uantities 45 Vm = 5 V0 7 48 Vm 9 or = V = 1 0 ∴ Ratio of the original solution left to that of the water added = 3 : 1 = 27 : 9. Hence, the initial volume = 36 L. ∴ 56. Amount of A initially = 7 × 36 = 21 L. 12 Let t be the time for which he travelled on foot. ∴ 8t + 16(7 – t) = 80 or t = 4 hr. The distance travelled on foot = 32 km. Alternatively: The problem could also be done using the principle of alligation. The average speeds on foot and on a bicycle are given and the resulting average speed 80    i.e. 7  is also given. We can find the ratio of   times and hence, the time taken for each part of the journey. 57. If the cost of second liquid is Rs. x per litre, then cost of first liquid is Rs. (x + 2) per litre. Selling price of mixture is Rs. 120, after making a profit of Rs. 20. So the cost price of mixture = Use Alligation 120 = 100 per litre . 1.2 312 = 104 calories . 3 9 L of mango juice contains = 54 calories and 9 L of mixture (orange and mango) contains 9 L of orange juice contains = So fraction of orange juice in 18 L mixture is 59. 1 . 10 Apply rule of alligation: Averag e co sts of ho rses –6% Resu ltant averag e Costs of horses 7.5% 0% 6 7.5 ∴ The ratio of the costs of the horses is 7.5 : 6 or 5 : 4. ∴ 5 : 4 is the ratio of cost of each horse. ∴ 5 × 1350 = 750 is the cost of one which was sold 9 at a loss of 6% and 4 × 1350 = 600 is the cost of the 9 other. (x + 2) – 100 5 = 100 – x 3 Explanations: Fundamentals of Ratio & Proportion MBA Test Prep Page: 77 60. = 40%. Now 30 L is withdrawn and replaced with Total fractional amount of milk Total fractional amount of water  3  × 100  . water. Final concentration = 30%  =  10  2 3 3 + + 3 4 5 = 40 + 45 + 36 = 121 = 20 + 15 + 24 59 1 1 2 + + 3 4 5 61. Hence,  40 − 30 10 1  = =  mixture. ∴ 40 40 4   Ratio of quantities = 2 : 5. Ratio of prices = 3 : 1. ∴ Ratio of the values of sugar to orange peels = 6 : 5. 1 th of the mixture = 30 L 4 So the volume at the end of first state = 120 L. Or ∴ 11 [5.2 − 0.8] = $2.4 worth is sugar in marmalade 6 62. 63. Fresh grapes contain 90% water. So 20 kg fresh grapes contain 18 kg water and 2 kg pulp. Now if 2 kg pulp contributes 80% of dried grapes, then the total amount of dried grapes = 2.5 kg. This is n 27 1 V – 3  = 1728 1  V  3 V–3 = 12 V Hence, the initial volume = 2 × 120 = 80 L. 3 Solutions Exercise 1 (Level – 1) 1. c n A 3 . Let A = 3x, B = 7x = B 7 A + B = 45; 3x + 7x = 45, x = 3 2. d x y x 35 3 5 = × = y 27 7 9 4 3. a 4 4 16  160  2 = =  =  81  240  3 ∴ Quantity of rum left = Let the fraction be x 1 7x 35 y 27 or, = = 3 1 3y 27 7 35 ⇒V=4L 1  240 – 80  =  1  240  If a, b and c are in continued proportion, the mean proportional is b. Therefore, b2 = ac, b2 = 8 × 72, b = 16 × 240 = 47.40 L 81 If a, b, c and d are in proportion, 5. e Third proportional, c = 6. b Let the number added be x. Then, At the first stage, the concentration of milk is 60%  3   = × 100  . Since the volume increases by 50%,  5  3 times of the original volume. 2 2  Hence, the concentration becomes  × 60%  3  Page: 78 576 = 24 3 27 a c , = = or 15 d b d or d = 135 is the fourth proportional. 4. b Level – 3 the volume becomes 45 = 4.5 10 B = 7x = 31.5 Final ratio of rum to total = Initial ratio of rum to total V – X ×   V  65. 3 times of the initial volume. 2 Final ratio of water to total = Initial ratio of water to V – X total x    V  As the process is repeated two more times. ∴ The number of times we do the same operation will be 3. 64. 1 th of milk has been removed from the 4 b2 30 × 30 = = 45 a 20 4+x 2 = , 12 + 3x = 18 + 2x 9+x 3 x=6 MBA Test Prep Solution Book-2 7. d A’s share = P × x 2000 = 3610 × x+y+z 4750 16. b = Rs. 1,520 Where P = Profit and x, y and z are respective shares of A, B and C. 8. c x 2 = y 7 Let x’s share = 2a, y’s share = 7a a + b + c 3k + 4k + 7k 14k = = =2 c 7k 7k 17. b x ' s share 2a 2 = = x ' s share − y ' s share 5a 5 9. d 10. a B' s part = = 11. d or 3A = 2B = C B 2 B + B + 2B 3 Option c and d: 4x = 84 x = 4; Possible. 18. b 19. b x 0.07 1 = = y 0.35 5 B’s money = 5 × 800 = 1000 4 3 × 1000 = 1500 2 Therefore, total amount of money = Rs. 3,300 C’s money = C’s share = 2 B’s share 2 B’ share 3 2 B' s share 2 3 = ⇒ required ratio is (2 B' s share – B' s share) 3 0.35 of x = 0.07 of y ∴ × 3960 3 × 3960 = 3 × 360 = 1080 11 Option a: Numbers = 9x, 3x 12x = 84 x = 7; possible. Option b: 8x = 84 ⇒ No whole number x is possible. 68 – x 3 = , ⇒ 272 – 4x 49 – x 4 = 147 – 3x ⇒ x = 125 A B C = = 2 3 6 a b c = = =k 3 4 7 a = 3k, b = 4k, c = 7k Let A’s share = 12. c 13. d 5x − 9 23 Let the number be 5x and 3x, then = 3x − 9 12 60x – 108 = 69x – 207; 9x = 99, x = 11 The first number is 11 × 5 = 55 15. a x x 68 17x 68 or, + = = , x = 28 7 10 10 70 10 21. b x’s share ⇒ 3x + 30 y’s share ⇒ 4x + 20 z’s share ⇒ 5x + 50 Sum is 9700 12x + 100 = 9700, 12x = 9600, x = 800 y’s share = 4x + 20 = 3200 + 20 = 3220 22. d Let B get’s A get’s C get’s x 3 y 2 4 = & = = y 4 z 3 6 ⇒ x:y:z =3:4:6 In a ratio A:B, A is called the antecedent and B is called the consequent. Let antecedent = 4x & consequent be 9x. 4x = 36 ⇒ 9x = 81. M 3 in 100 L mixture = W 1 Milk = 75 L, Water = 25 L After adding 200 L of water, water = 225 L and milk = 75 L Ratio = Let the two parts be x, (68 – x) x 1 68 x = (68 − x ) = − 7 10 10 10 4x = 3y = 2z ⇒ 14. d 20. c Rs. x Rs. x + 70 Rs. x – 80 x + x + 70 + x – 80 = 530 3x – 10 = 530, 3x = 540 x = 180 A ' s share 180 + 70 250 5 = = = C' s share 180 − 80 100 2 75 =1:3 225 Explanations: Fundamentals of Ratio & Proportion MBA Test Prep Page: 79 23. b A= 1 A 1 = , B, B = 2C or 2 B 2 Water available = 30 L – 6 L = 24 litres. Now, 16 litres of milk are added, ⇒ Milk = 40l + 16 L = 56 L Water = 24 litres ⇒ Milk : Water ratio is 56 : 24 = 7:3. B 2 = C 1 A:B:C=1:2:1 Shares of A, B, C are x, 2x, x 29. b B 2x 2x 2 = = = A + B x + 2x 3x 3 24. c 500 = 25 kg 20 Copper = 13 × 25 = 325 kg x= Initially, Number of boys = 250 Number of girls = 250 New Batch: 1 × 250 = 50 girls left & 25 boys joined 5 the class 30. d Let present age of son = x and present age of father = y (y – 4) = 6(x – 4) … (i) … (ii) (y + 12) = 2(x + 12) from (i) and (ii), y – 6x + 20 = 0 y – 2x – 12 = 0 4x = 32, x = 8 years, y = 28 years Ratio of their present ages = 27. d 5 × 80 = 50 liters 8 Water = (80 – 50) = 30 liters. When we take a 16 liter sample of this mixture. 5 Milk taken out = × 16 = 10 litres 8 Water taken out = (16 – 20) = 6 litres. ⇒ Milk available = 50 L – 10 L = 40 litres Page: 80 16 . 56 1. b B 7 = G 5 Let number of boys = 7x Let number of girls = 5x 7x + 5x = 72, x=6 Number of boys = 42 Number of girls = 30 12 more girls should be admitted to make the ratio equal. 2. d Let A’s income = Rs. 3x B’s income = Rs. 2x A’s expenses = Rs. 5y B’s expenses = Rs. 3y A’s savings = 3x – 5y = 3000 B’s savings = 2x – 3y = 3000 Solving (i) and (ii), x = 6000 B’s income = 2x = Rs. 12,000 C1 5 C2 5 = , = I1 8 I2 3 In 80 liters of mixture, milk = 2 7 Solutions Exercise 2 (Level 1) 28 =7:2 8 5 5 + Copper 13 8 40 + 65 105 = = = 8 3 64 + 39 103 Iron + 13 8 28. c Number is Let Father’s age be 7x, son’s age = 2x After 15 years, 7x + 15 2 = , 7x + 15 = 4x + 30, x = 5 2x + 15 1 Present age of father = 35 Present age of son = 10 Father’s age when son was born = 35 – 10 = 25 26. d Simplest form = Numerator = 2x Denominator = 7x 7x – 2x = 40, x = 8 Boys 275 11 = = Girls 200 8 25. a Let copper = 13x Zinc = 7x In 500 kg, 13x + 7x = 500 3. a Let the number to be added be, x. Then, 4. a … (i) … (ii) 7+x 1 = ,x=5 19 + x 2 Take 7 L of the solution of each mixture, (LCM of 5 + 2, 6 + 1, 4 + 3) ∴ net amount of water in the resulting mixture, 5 6 4 × 7 + ×7 + × 7 = 15 L 7 7 7 The net amount of alcohol is, 2 1 3 ×7+ ×7+ ×7 =6L 7 7 7 Ratio is, 15 : 6 = 5 : 2 MBA Test Prep Solution Book-2 5. c If the ratio is a : b, then a + b should divide 12 completely. Only 3 : 2 does not divide, the rest are all possible ratios. 6. d If the ship weighs 5 kg, weight above water = 3 kg. Weight submerged = 2 kg. Therefore, desired ratio = 2 : 3. ∴ Cost of glass lenses = 3 (52 − 40) = $4 . 1 (Compare with the above algebraic method.) 7. c 5% of x = 30. Therefore, x = 600. Therefore, number of entrants who lost = 570. 8. c If the ratio is a : b, then a + b should divide 8 completely. Only 36 does not fulfil this condition. 9. e Considering the ratio, B’s + C’s shares = 20x and A’s share = 5x. Hence, the second sentence gives no additional information. Since the total amount is not given, we cannot say anything about A’s share. 10. d LCM(15, 25) = 75 min is the time when both of them would meet at the starting point for the first time. In this time A takes a lead of two rounds over B. If they have to meet for the first time, then A has to take a lead of one round, which happens after 37.5 min. 11. a 13.a Ratio of savings of A and B is 5 : 3. So it is 2 obvious that A’s savings is 66 % more than that 3 of B. 14.b Here Sujith earns more but spends less so definitely his savings will be more than that of Pratima. 15.d 1 : 5 and 3 : 5 solutions Since equal quantities of both are mixed, we can take LCM of (1 + 5 = 6) and (3 + 5 = 8), i.e. 24. Assuming that 24 L of both solutions are mixed together in the resultant solution. Milk = 4 + 9 = 13; Water = 20 + 15 = 35 Hence, the ratio of water and milk = 35 : 13. Alternative method: Amount 1 1 of milk in 1 : 5 solution = . 6 6 Amount 3 3 of milk in 3 : 5 solution = . 8 8 Let the incomes are 3x and 2x and the expenditures are 5y and 3y. Therefore, 3x – 5y = 1000 and 2x – 3y = 1000. y = 1000, x = 2000. Therefore, incomes are Rs. 6000 and Rs. 4000. Alternative method: In first step see which option gives a ratio 3 : 2 in incomes. Unfortunately, all three are in the ratio 3 : 2. Now subtract Rs. 1,000 (savings) from each option to get the ratio of expenditures. We see that only option (a) gives a ratio 5 : 3. 12.b Plastic lenses are four times costlier than glass lenses. ∴ If cost of glass lenses = x, cost of plastic lenses = 4x. Cost of age examination and frames = y. ∴ y + x = 40 and y + 4x = 52 ⇒ 3x = 12 ⇒ x = $4 Alternative method: The price differential is due to plastic lenses which cost four times as much as the glass lenses. ∴ Price differential = Cost of plastic lenses over and above glass lenses = 3 (Cost of glass lenses). Explanations: Fundamentals of Ratio & Proportion 1 6 =1 3 –x 1 8 x– ⇒ 2x = 1 3 4 + 9 13 + = = 6 8 24 24 13 48 ∴ Water : Milk = (48 – 13) : 13 = 35 : 13 ⇒x= MBA Test Prep Page: 81 16. c Mileage when petrol is adulterated = 3 of (16) = 12 km/L 4 4 Hence, the cost increases to a factor   of the 3 original cost of maintenance. So it increases by 1 × 100 = 33.33% . 3 Questions 20 and 21: Half of the volumes of the containers A and B are emptied into the third container. Hence, the volumes emptied are 108 cm3 and 32 cm3 respectively. 20. c 21. e Hence, the ratio of water to milk is 108 : 32 or 216 : 64. 140 × 100 = 14% 1000 22. a Alternative method: We can safely assume that the car runs for the same number of kilometres. 3 of its original value, Now mileage is reduced to 4 which is, say, x. ∴ Distance = Mileage × Number of litres ⇒ x × y1 = Elder son Younger son now 20x 5x 4x After 10x years, 30x 15x 14x So 30x = 2 × 14x + 3 ⇒ x = 1.5 ∴ 20x = 30 years. 23. a 3 y 4 x × y2 ⇒ 2 = = 1.33 4 y1 3 The final amount of milk after two operations = Therefore, we see that the consumption of fuel 24. c increases by 33.3% . Father 4 4 3  × ×  × 50  = 19.2 L 5 5  5  2xy The harmonic mean of 2 numbers x and y is (x + y ) and the geometric mean is xy . 17.a 5 5 If the number is x, x − x = 125 or x = 360. 8 18 ∴ 18. e 19. a If A’s age is 5x, B’s age = 7x. Case I: 7x – (5x + 6) = 2 So x = 4. Case II: (5x + 6) – 7x = 2 So x = 2 Hence, sum of their present ages = 12 × 4 = 48 years or 12 × 2 = 24 years. ⇒ or This can be solved using alligation. 2xy x+y xy = 12 , 13 2 xy 12 xy 6 = = or x + y 13 x + y 13 x + xy y 13 x or = + 6 y xy y 13 = x 6 From here, you can put options in the above equation and check. Only option (c) satisfies. 25. c Let X ml be added to the solution. ⇒ [(15% of [400 400) + X] + X] = 32 100 ⇒ X = 100 ml So the amounts invested are in the ratio 2.5 : 1.5 = 5 : 3. Since the amounts invested at the two rates are in the ratio 5 : 3, and Rs. 680 is invested at 6%, amount invested at 10% = Page: 82 680 × 3 = Rs. 408. 5 Alternative Method: 15% of 400 ml = 60 ml Rest is water = 400 – 60 = 340 ml Since, water is constant ⇒ 340 ml will be 68% (in the new mixture) ⇒ 340 is 68% 340 × 100 = 500 ml ; Extra alcohol 68 required 500 – 400 = 100 ml ⇒ 100 is MBA Test Prep Solution Book-2 Solutions Exercise 3 (Level 2) 1. d 2. a 3. e The question uses the same concept as in Q 17. Number of mugs = 12 The sum of broken and unbroken mugs must be equal to 12. Sum of ratio = 3, 12, 5, 6, (5 does not divide 12) ∴ 3 : 2 cannot be the ratio. A’s effective investment = 16000 × 6 = B’s effective investment = 12000 × 8 = C’s effective investment = 1000 × 12 = Profit sharing ratio = 96 : 96 : 12 or 8 : Rs. 96,000 Rs. 96,000 Rs. 12,000 8:1 Bs’ share = 5. d For every 100 paise that A gets, B gets 65 paise and C gets 35 paise. 35 C' s share = 560 = sum × 200 ∴ Sum = Rs. 3,200 9. b Let the amount of second and third quality of sugar in the mixture be ‘x’ and ‘y’ kg respectively. ∴ 15 × 4 + 18x + 22y = 22 × 10. a r= 11. e ∴ using the rule of alligation, 11 9 1 0.5 0 .5 Let the quantity of second and third variety be x and y kg respectively. 100 ∴ 20 × 1 + 30x + 24y = 30 × [1 + x + y ] 120 ⇒ 5x − y = 5 …(i) All the four options (a), (b), (c) and (d) satisfy equation (i). 12. e Let’s take 12 litre of 5 : 1 sample 24 litre of 2 : 1 sample and 36 litre of 3 : 1 sample Amount of milk in the final mixture 5 2 3 × 12 + × 24 + × 36 = 53 L 6 3 4 Amount of water in the final mixture = 72 – 53 = 19 L ⇒ milk : water = 53 : 19. = 84 × 26500 = 10,500 212 2100 × 100 = 10.5% per annum 20,000 100 = Rs. 16 / kg 50 Let the quantity of second and third variety be ‘a’ and ‘b’ kg. ∴ 15 × 6 + 18a + 20b = 16[6 + a + b] Mean rice of the mixture is 8 × ⇒ a + 2b = 3 a = 1 and b = 1 is the only integral solution. Let A’s, B’s and C’s investments be 8x, 7x, 5x A’s effective investment = (8x) × 5 + (4x) × 7 = 40x + 28x = 68 x B’s effective investment = 7x × 12 = 84x C’s effective investment = 5x × 12 = 60x The net rate of interest paid is, 13. e Using alligation we find the amount of copper (average amount of copper in the final mixture is x) Thus ratio of loan from first to second bank is 0.5 : 1.5 = 1:3 Explanations: Fundamentals of Ratio & Proportion 7 10 2 5 x 2 1 .5 100 [4 + x + y ] 110 ⇒ y − x = 10 L:R:O=5:7:3 Let Labour cost be 5x, Raw Material Cost = 7x Overheads = 3x, Total cost = 15x Profits = 20% of 15x = 3x B’s share = 7. a Let the required quantity be x kg. ∴ 20 × 2 + 24x + 30 × 3 = 25 (3 + x + 2) ⇒x=5 3 × 5200 = 2600 6 Material cos t 7x 7 = = Pr ofit 3x 3 6. c 8.b Ratio of their profits = Ratio of their investments = 4000 × 12 : 8000 × 9 : 12000 × 2 48 : 72 : 24 2 : 3 : 1 4. c 1 ∴ Loan from 1st bank = × 20,000 = 5,000. 4 and Loan from 2nd bank = 15000. 1  7  – x  2 ⇒x=1 ⇒  10 = 2  x – 2  1 5   1 of the new alloy the final ratio 2 of copper to nickel will be 1 : 1. Since, copper is MBA Test Prep Page: 83 14. b The ratio between the distance and the speed is the same. Since the ratios between the two are the same, the time has to be constant and hence the ratio needs to be 1 : 1 : 1 or mathematically, Time = Dis tan ce Speed Hence, Ratio of time = 15. e 20. c Ratio of dis tan ce = 1: 1: 1 . Ratio of speed Selling at Rs. 11 per kilogram, profit percentage = 25%. Suppose the person buys A apples and M mangoes and cost price of an apple is Rs. x. Therefore, cost price of a mango will be 2x. Total cost price = Ax + 2Mx. Now selling price of an apple is 2x. ∴ SP of a mango will be 6x. Total SP = 2Ax + 6Mx. Now we have 2Ax + 6Mx = 21. a 11 = Rs. 8.8 1.25 Applying the rule of alligation, Therefore, CP = 5 M 1 (Ax + 2Mx) or = . 2 A 2 Let us assume that out of 100, he obtained 72 marks. If he had attempted four more questions, he would have made one more mistake. Hence, three of them are correct. The three correct questions secure him 12 marks more. This 12 = 4 marks and 3 hence, number of questions is 25. means, each question carries Alternative method: We can clearly see that three correct answers increase the score by 12%. [Three correct questions as four new questions and one mistake] 100% score corresponds to 3 × 100% = 25 questions. 12% The required ratio is 2 : 3. 16. c of the examination = 17. b 18. d 22. a Let the maximum marks in the quizzes be 1 and that in the examination is 3. Hence, the weightage 3 ×1 1 = . 6 × 1+ 3 × 1 3 Alternative method: In these type of questions it is always easier to work with man-hours (or man-days, etc). 50 men working 10 hr a day for 8 days is 50 × 10 × 8 = 4,000 manhours. Now 35 (boys and girls) working 15 hr a day for If he spends ‘t’ minutes on the second half, then he spends 2 t on the first half. 3 Therefore, 2 t + t = 90 or t = 54. 3 1  (say) x days is  × 35 × 15 × x  man-hours 2  If there were 2, 5, 1 coins of the denominations Rs. 1.50, one-rupee and 25-paisa respectively, then value of the coins would be Rs. 8.25. Hence, if the total value has to be Rs. 330, then the number of coins would have to be multiplied by a factor of 40. There would be in all 80 of Rs. 1.50 coins, 200 one-rupee coins and 40 25-paisa coins. ⇒ Let the value of the turban be Rs. x. Then Page: 84 100 + x 12 = ⇒ or x = Rs. 40. 65 + x 9 1 × 35 × 15 × x = 4,000 2 ⇒x= 23.c 19. c 20 boys = 10 men. 15 girls = 7.5 men. Therefore, total number of men = 17.5 Now 50 × 10 × 8 = (17.5 × 15)x, where x is the number of days. Therefore, x = 15.24 (approximately). 8000 = 15.24 days. 35 × 15 Suppose numbers of students who pass and fail are 3x and x respectively. ∴ Total number of students appearing = 3x + x = 4x. MBA Test Prep Solution Book-2 In second case, total number of students who appear = 4x + 8; total number of students who pass = 3x – 6. ∴ Total number of students who fail = (4x + 8) – (3x – 6) = x + 14. 3x − 6 = 1 ⇒ x = 10. x + 14 ∴ Total number of students is 40. 30 × 100 = 30% 100 of water is added to pure milk. Therefore, 30 L is added or 28. e Now we have 24.c 25. c Ratio of white to yellow balls = 6 : 5. Difference in the number of white and yellow balls = 6x – 5x = x = 45. Therefore, number of white balls now available = 45 × 6. Number of white balls ordered = (45 × 6) – 45 = 225. 29.c Ratio of white mice to total mice 30. c = 1 1 2 : 1 1 8 = 1: 4 = 1 1 3 : 1 1 : 1. 4 1 9 = 3 : 9 = 1: 3 = ⇒ 235 + x + 10 8 7 1 x = (235 ) + 10 ⇒ x = 45 8 8 ∴ Total collection = 235 + 45 = Rs. 280. 1 1 : = 3:4 . 4 3 27. a 4 1 − 140 × = 80 − 35 = 45 L. 7 4 Six people contribute a total of Rs. 180. Let the seventh person contributes Rs. x. Eighth person contributes Rs. 55. Total contributions of these eight persons = 235 + x. Now x = 1 : 1. 3 ∴ Ratio of white mice to gray mice = 26. e 4 1 v − x = v ⇒ 9v = 28x 7 4 Since by adding 35 L the level rises from a quarter to a half, the volume of the vessel = 35 × 4 = 140 L. Therefore, x = 140 × Ratio of gray mice to total mice = No price values (either of profit amount or of cost price) is given. Obviously, SP cannot be determined. When 5 L brandy is transferred, volume of brandy left in vessel I = 15 L. Volume of brandy in vessel II = 5 L. Volume of water in vessel II = 20 L. Now, 4 L of mixture containing brandy and water in the ratio 1 : 4 is poured into pure brandy of volume 15 L. 4 L of mixture contains 3.2 L water and 0.8 L brandy. ∴ Volume of brandy left in vessel II = 5 – 0.8 = 4.2 L. Ratio of water in vessel I to brandy in vessel II = 3.2 : 4.2 = 16 : 21. Solutions Exercise 4 (Level 2) 1. c Percentage of whisky in mixture = 75%. Percentage of whisky in water = 0%. In 100 L pure milk, let him add ‘a’ litres water. Let CP of 100 L pure milk = Rs. 100. CP of (100 + x) L adulterated milk = 100.  10000  CP of 100 L adulterated milk =   .  100 + x  SP of 100 L adulterated milk = 100. ( ( 100 Profit percentage = 100 1 − 100 +x 10000 100 + x ) ) × 100 So mixture and water should be mixed in the 1 ratio 2 : 1 or we need to replace 3 of the mixture with water. or x = 30. Explanations: Fundamentals of Ratio & Proportion MBA Test Prep Page: 85 2. a Let x be the distance travelled by bus. Quantity of milk after third draw x 1000 − x ∴ 80 + 36 = 16 ⇒ x = 770 km (approximately) 3. d 9 9 9 × × × 80 = 58.32 10 10 10 ∴ Ratio = 100 : 81. = Applying the rule of alligation: Alternative method: We know that New amount of liquid X = Original amount of liquid X n  Amount of liquid taken out and replaced  1 −   Total amount of liquid   where n = Number of times the process is repeated 2  8   9  = 1 –   =  10   80    ∴ 4. e ∴ Ratio = 102 : 92 = 100 : 81 Ratio of volumes of two solutions = 1 : 2. Tin content in the alloy = 100 – 77.78% = 22.22%. Using the principle of alligation, 2 6. b Initial respective amount of milk and water is 28 L and 7 L. 7 L of water added means now we have 14 L of water. To keep the ratio M : W same, amount of milk must be doubled, i.e. 28 L of milk must be added. Alternative method: In order to keep the solution of the same ratio, we need to add milk in the same ratio as already there in the solution. ∴ We need to add 7. a 4 × 7 = 28 L of milk. 1 Rule of alligation for fractional content of water:  50  × 18  : 18 = 32.4 : 18 . Ratio of volumes =  27 . 78   ∴ Total tin content in the new alloy = (22.22% of 32.4) + 18 = 25.2 kg. Total copper = (77.78% of 32.4) = 25.2 kg. Hence, none of these is the correct option. By the way, is there any need to do any calculations? In the question it is given that the new alloy has 50%, i.e. equal amount of copper and tin. 5.b Quantity of milk after first draw 90 × 80 = 72 kg . 100 Quantity of milk after second draw = = Page: 86 or 11 : 1 or 33 : 3 ∴ 3 L of water must be added to 33 L of solution of milk. 90  90  × 80  = 64 kg . 100  100  MBA Test Prep Solution Book-2 8.b 9.b In fact, she takes out 5% of the existing volume of wine in every operation. After the first operation concentration of wine is 95%. After the second operation concentration of wine is 95 × 0.95 = 90.25%. After the third operation concentration of wine is90.25 × (0.95) = 85.7375%. Obviously, after the fourth operation the concentration of wine will be less than 85%. 13. b 450 = 3.75 120 Using the alligation method, Hence, CP = The ratio of incomes of Rashmi and Divya is 3 : 5. Ratio of their expenditures is 2 : 3, i.e. 3 : 4.5. Had the ratio of expenditures been 3 : 5, ratio of savings also would have been 3 : 5; but since ratio of their expenditures is 3 : 4.5 only, obviously savings of Divya will be something 0 .25 5 of savings of Rashmi and thus 3 Divya will save more. more than 10.c Income Rohit Sunil 5x 7x Expenditure 5y 7y Saving 5x – 5y 7x – 7y Ratio of savings is 11.b SP = 4.5 and profit = 20%. i.e. 1 : 1 Hence, in a 40 kg mixture, there are 20 kg of the first variety and 20 kg of the second. 14. a Here we calculate the number of birds in terms of number of trees. We know that there are 630 birds. Let the number of trees be x. ∴ Number of sparrows = 5( x − y ) =5:7 . 7( x − y ) Other birds = n(n + 1)(2n + 1) . 6 Square of sum of first n natural numbers is Thus, x + n2 (n + 1)2 = 17 : 325 4 x 3 × ×4 . 2 4 x 3 x 21 x = 630 ⇒ x = 240 + = 8 2 8 Hence, other birds are there in 3x = 90 trees. 8 On solving, we get n = 25. 6a2 = 2(lb + bh + lh) l : b : h = 1: 2 : 4. Therefore, b = 2l; h = 4l Hence, 6a2 = 2[2l2 + 8l2 + 4l2] 6a2 = 28l2 If IBM initially quoted Rs. 7x lakh, SGI quoted 4x lakh. IBM’s final quote = (4x – 1) lakh  14  2  14  2 3 a =   l ∴ a3 =   l 3    3  Now = 17 : 325 . 4 15.a 1 12.c x ×2 . 2  x 1 Also number of pigeons =  ×  × 1 . 2 4 Sum of squares of first n natural number is n(n +1) ( 2n +1) 6 n 2 (n +1)2 0 .25 4x − 1 3 = or x = 1 4x 4 IBM’s bid winning price = Rs. 3 lakh. So IBM wins the bid at 4x – 1 = Rs. 3 lakh. Note: Whoever quotes the minimum price, wins the bid. Explanations: Fundamentals of Ratio & Proportion 3 Ratio of the volumes of the cube and cuboid 3  14  2 = a3 : lbh =   : 8  3  MBA Test Prep Page: 87 16. a If three kinds of birds are taken to be 3x, 7x and 5x respectively, then 7x – 3x = 63y (where y is any positive integer). As the number is a multiple of both 9 and 7, it has to be a multiple of 63. 20. d A, B and C are to share profit in the ratio 4 : 3 : 7. But, since A is to receive 10% of profit, only 90% is to be shared. A’s total share = 10% of profit + 63 y 4 Minimum value of y for which x is a natural number is 4. ⇒ x = 63 Hence, the number of birds = 15x = 945. ⇒x= 4 × 90% of profit 14 = 6000 or (10 + 180 )% of profit = 6000 7 250 % P = 6000 ⇒ P = Rs. 16,800 7 B and C’s share combined or 17. e 18. d Price = k × Weight, where k is any constant. Original weight of the stone = 28x. Original price = 28kx. New price = k(15x + 13x) = 28kx. Hence, there is no profit, no loss. Concept: Please note that since we have a linear function, there is neither profit nor loss. 10 10 90 of 90% profit = × × 16800 14 14 100 = 10,800 = 21. a In a mixture of 1,000 ml, milk : water = 3 : 1. Hence, milk = 750 ml, water 250 ml. A 250 ml of 3 : 2 solution contains 150 ml milk and 100 ml water. Total milk = 900 ml, total water = 350 ml. After using 250 ml to make curd milk used 250 × 900 = 180 ml. 1250 Pure milk left = 900 – 180 = 720 ml. 30 5 = or x = 2 (10 + x ) 2 = 19. d Let income and expenditure of the first person be 5x and 9y respectively. Then income and expenditure of the second person are 4x and 7y respectively. 5x – 9y = 4x – 7y = 500 or x = 2y. Therefore, y = 500, and x = 1000. Incomes are Rs. 5,000 and Rs. 4,000 respectively. OR By checking the choices you will find that only for choice (d) is the ratio of incomes 5 : 4. Alternative method: In the given choices, the choices (a) and (c) are not in the ratio 5 : 4. Now we can subtract Rs. 500 from incomes and check the ratio of expenditures in choice (b). Ratio of expenditures = (3750 – 500) : (3000 – 500) = 3250 : 2500 = 13 : 10 Hence, the solution is (d). Please note that we need not calculate the ratio of expenditures in (d). In the original mixture, 3 spirit = × 40 = 30 L, 4 1 water = × 40 = 10 L. 4 To get the ratio 5 : 2, let’s assume that we add x litres of water. Hence, 2 L of water has to be added. 22.d i.e. 2 : 1 Hence x litres must be 12 L, i.e. (6 × 2). 23. c This is a classic case for approximations. We see that the ratio 1075 : 1000, when converted to percentage terms, is approximately 52% : 48%. Thus, we can calculate the literacy percentage as 24 8 × 52 + × 48 ≈ 16.3% 100 100 Now as 100 – 16.3 = 83.7%, only choices (c) and (e) are left. We calculate the number of literate people as 16.3 × 311250 = 50,700 . 100 Page: 88 MBA Test Prep Solution Book-2 Thus, assuming we have x students initially. x(26 – 27) + 3(29 – 27) = 0 We could have approximated the ratio of men to women as 50% : 50% also. 24. b Let x be the number of children employed. Then numbers of men and women are 8x and 5x respectively. Let y be the common factor for wages. Then wages of men, women and children are Rs. 5y, 2y and 3y respectively. Hence, total wages of different categories will be 40xy, 10xy and 3xy respectively. Total wages = (40 + 10 + 3)xy = 53xy = 318. So xy = 6. Thus, they will be getting 40 × 6, 10 × 6 and 3 × 6 respectively. Alternative method: ⇒ –x+6=0 x=6 Please note that always subtract the average from the number and not vice versa. 28. a Assuming that there is 1 L of spirit and 3 L of water, the total volume of the mixture is 4 L. After increasing its volume by 25%, volume of the mixture is 4 + 25% of 4 = 5 L. So the ratio of volumes of spirit to water = 2 : 3. 29.c A ratio Sum of all three wages have to be equal to Rs. 318. Only (b) displays this property. 25. a 26. a Speed in upstream = Speed of the boat in still water – Speed of the river = b – r. Speed in downstream = Speed of the boat in still water + Speed of the river = b + r. Therefore, b + r = 3(b – r) or 2b = 4r or b = 2r Since r = 4 km/hr, b = 8 km/hr. Alternative method: 1 1 1 : : = 0.5 : 0.33 : 0.25 2 3 4 Thus, C, who was getting the least, finally got the maximum amount. This eliminates (a) and (b). Thus, gain of Milk in final/Milk in original = [(a – b)/a]n, where a = Quantity of mixture, b = Amount taken out during each operation, n = Number of times the operation is repeated. 3 4 0.25   –  C = Rs.117   4 + 3 + 2 0.25 + 1/ 3 + 0.5  = Rs. 25 Hence, the answer is (c). 3  40 − 10  3 Milk in final/Milk in original =   =  4  .  40    1 1 1 : : is equivalent to 6 : 4 : 3. 2 3 4 So in this case, A, B and C would have got Rs. 54, Rs. 36 and Rs. 27 respectively. But actually the money was divided in the ratio 2 : 3 : 4 and shares of A, B and C in this case was 26, 39 and 52 respectively. Hence, the answer is (c). 30. d 27 = 16.875 L. Milk in final solution = 40 × 64 Water = 23.125 L. Required ratio = 16.875 : 23.125 = 27 : 37. 27. b This question can either be done by substituting choices, or by using weighted averages. If x be the number of members in club initially, then sum of ages of (x + 3) members now = 26x + 87 = 27(x + 3) Hence, x = 6. So number of members now = 9. Alternative method: This method is called sum of deviations, as follows. “The sum of all weighted deviations from the average is zero.” Explanations: Fundamentals of Ratio & Proportion i.e. 7 : 1 Hence, milk and water are in the ratio 7 : 1 since there is 28 L milk. Hence, water should be 4 L. MBA Test Prep Page: 89 Solutions Exercise 5 (Level 2) 1. a If A, B and C represent the number of three denominations in increasing order of their values, then A = 7B, C = 4B. Since C = 12, B = 3 and A = 21, the total money in the purse = 21 × 1 + 3 × 10 + 12 × 20 = Rs. 291. 2.c If there are x number of one-rupee coins in the bag, then there are 8x and 16x, 50-paisa and 25-paise coins respectively. Therefore, money in the bag = x + 4x + 4x = 9x = 495. So x = 55. Hence, there are 8 × 55 = 440 50-paisa coins in the bag. 3. a A B 1 B + = (2C – A) = 2 3 2 2 ∴ B = 3A, C = 2A So amount spent 3A = (A + 3A + 2A) – = 4.5A 2 Percentage of amount spent = 75%. ∴ 10.e We cannot determine the final ratio unless we know the volume in 3 jars. 11. b Mixing two lumps, we have 18 g from each metal. So price of the second metal (87 + 78.60 − 6.7 × 18) = Rs. 2.50 per gram 18 ∴ 6.7x + 2.5(18 – x) = 87 ∴ x = 10 g and 18 – x = 8 g = There are 880 25-paisa coins. Value of the money = 0.25 × 880 = Rs. 220. 4.c 2 12.c V1 V2 V3 Initial volume: 0L 40 L milk 80 L W water After operation 1: 0 20 L M + 20 L W 60 L w + 20 L milk After operation 2: 0 15 L M + 25 L W 55 L w + 25 L milk 5.a Check from the above data. 6.a Suppose he invested Rs. x and Rs. y in the matches of South Africa and India respectively. From the first match he will get 3 x, but from the second he will get nothing. Now 3x = (x + y) + (x + y) ⇒ 1− 8.c When 25 L is withdrawn, half the solution is removed. Hence, 9.6 L of milk is left. 14. c Let Tina’s share be T, Issan’s be I, Abhishek’s be A and Fatima’s be F. I 80A =F−4 Given that T + 3 = I + = 3 100 You get T=F–7 ... (i) x 150 5 or y = . 100 1 Suppose he invested 3X on the first match and 4X on the second match. He will get back 3X × 3 = 9X . So gain = 9X – 7X = 2X = 200000 or X = 100000 :. Total investment = 7X = 7 × 100000 = 700000 9. c 8x × 100 = 266.66%. 3x A +B+C ⇒ Originally B had an amount C= 3 = 2C – A Now A′ = Page: 90 A 1 , B′ = B , C′ = 0 2 3 3 (F – 4) 4 ... (ii) 5 (F – 4) ... (iii) 4 Also given that T + I + F + A = 80 ... (iv) Substituting the values from (i), (ii) and (iii) in (iv), we get A= 3F 5F   F + 4 + 4 + F  – 7 – 3 – 5 = 80 or F = 23.75   Investment = x + 2x = 3x He gets back 3x + 8x = 11x. So gain = 8x. Gain percentage = x 2 = ⇒ x = 18 54 3 13. a I= 7.a 2 24 4 x  x    54  1 − =  = 24 ⇒  1 − 54  = 54 9  54    15. a Let the price of one tea = x ⇒ the price of other tea= x 2 Price of 1 kg = 2x + 3 × x = 7x 5 5 2 10 But CP = MBA Test Prep 17.50 × 100 = 14 125 Solution Book-2 (Only the first option satisfies the first condition.) 22. d 10 kg copper is from alloy II. Hence, 28 kg copper is from alloy I. In alloy I, tin : copper = 3 : 4 Since, 4 units = 28 kg Alloy = 7 units = 49 kg 23 c Use alligations. 7x ⇒ = 14 ⇒ x = 20 10 So the price of the tea’s are 20 and 10. 16. c 1 of the whole gets thrice 4 of what the others get on an average, each one will get If the person who gets So lution 1 Averag e percentage of acid in so lutions 1 1 1 × = of the whole. 3 4 12 Therefore, if there are x persons other than the person who gets one-fourth of the whole, then = 90% 75% Resu ltant average 1 x + =1 4 12 So x = 9 Hence, total number of people = 10 17. c So lution 2 Ratio of q uantities of acid in so lutions ∴ 6p + 8q = 7.5(p + q); 6q + 8p = n(p + q) 14(p + q) = (7.5 + n) p + q ⇒ n = 6.5 X Y 90 − 78 Y = 78 − 75 X 12 Y X 1 = ⇒ = 3 X Y 4 So ratio of quantities of solutions = 1 : 4 Total quantity = 30 L ∴ Quantity of 90% solution of concentrated acid ⇒ 18. b Amount of 16% iodine in the mixture 16 gm 100 Now the amount of iodine becomes 20% of the mixture. Amount  16   100  evaporated = 735 − 735 ×   ×  20  = 147 g  100    = 735 × 19. b Whether you add them in the ratio 2 : 3 or 1 : 1, we get the same concentration. Hence, both of them must have a 35% concentration. So (b) is the answer. 20. c Only 5 women were college graduates while 20 women were employed. Therefore, maximum number of ‘employed graduate women’ can be 5. Since number of ‘unemployed men’ was (55 – 15) = 40. Answer cannot be greater than 21. b 1 . 8 Only possibility is if 650 is divided by a multiple of 13, because the other jars have a capacity that is not a multiple of 13. 403 is the only choice possible. Alternative method: As the beakers filled by same fractions, a b c a+b+c = = = 250 650 200 250 + 650 + 200 3 a + b + c = 682 ∴ = 30 × 24. d 1 =6L 5 s−a s−b s−c = = =k 1 7 4 Simplifying this, we get a : b : c = 11 : 5 : 8 Solutions Exercise 6 (Level 3) 1.c After first amount of amount of amount of amount of transfer, wine in first vessel = 9 L, water in first vessel = 3 L, wine in second vessel = 3 L, water in second vessel = 1 L. 3 L drawn from first vessel contains of wine and 3 9 ×3 = L 4 4 1 3 ×3 = L of water. 4 4 3 L drawn from second vessel contains wine and b 682 = 650 1100 9 L of 4 3 L of water. 4 b = 403 ml Explanations: Fundamentals of Ratio & Proportion MBA Test Prep Page: 91 ∴ Amount of wine in first vessel after second 3. c 9 9 + =9. 4 4 Amount of water in first vessel after second transfer = 9 − 3 3 + =3. 4 4 for second From (ii), a = transfer = 3 − Similarly, 5a + b > 51 3a – b = 21 8a > 72 or a > 9. … (i) … (ii) 21 + b 3 21 + b >9⇒b>6 3 Combining a > 9, b > 6. ⇒ vessel, wine = 9 9 + =3 4 4 3 3 Water = 1 − + = 1 4 4 ∴ Ratio is same in both the vessels. 3− Alternative method: Ratio of wine to water in first vessel after first transfer is (12 – 3) : 3 = 3 : 1. Ratio of wine to water in second vessel after first transfer is 3 : (4 – 3) = 3 : 1. Since the ratio of wine to water in both the vessels is equal. Any amount of such exchange any number of times will not alter the ratio of wine to water. 2.e 4. b Average age = 13. Number of students = 50 W ∝ H and H ∝ A ⇒ W ∝ A ∴ W = K1 H and H = K2A ⇒ W = K1K2A For A = 11, H = 165 and W = 33 33 = 3 ∴ W = 3A 11 Since W ∝ A, the averages should also be in direct proportion. ∴ Wavg = K1K2 A avg = 3 × 13 = 39 kg K1K2 = Consider two cases: i. Princy Kunjumol Income 100 200 Expenditure 1 5 Saving 100 – 1 = 99 200 – 5 = 195 Thus, Kunjumol is saving more. ii. Princy Kunjumol Income 100 200 Expenditure 39 195 Saving 100 – 39 = 61 200 – 195 =5 Thus, Princy is saving more. Hence, their savings depends upon their respective expenditures. Page: 92 MBA Test Prep Solution Book-2 Explanations: Time, Speed & Distance Solutions (Non MCQ) 6. 2. Speed of second train = Distance to be covered = 100 m. Speed of the person = 36 km/hr = 10 m/s. 40 100 = 3 3 Time taken by the two trains to completely pass 100 m = 10 s. Therefore, time taken = 10 m / s = 20 + 2 × 75 × 50 = 60 km / hr Average speed = [75 + 50] each other = 7. 3. One revolution means circumference of the wheel = 250 cm = 2.5 m. In 10 revolutions 25 m is covered by the wheel. Speed of the wheel = 25 25 18 × m/s = km/hr = 4 4 5 22.5 km/hr 4. Method 1: The man takes 2 hr to travel 14 km, where he stops for 10 min. Therefore, time between the start of journey and start after first stoppage = 2 hr 10 min. This pattern of his travel repeats. Now, 98 km = 14 × 7. But when he completes 98 km, the case of his stopping for 10 min does not arise. Therefore, total time taken = 6 (2 hrs 10 min.) + 2 hr = 15 hr Method 2: If the man had travelled non-stop he would have covered the distance in 14 hr. But since he stops 6 times on the way he would take an extra 6 × 10 = 60 min to cover the same distance. Hence, he would reach the destination after 15 hr. 5. Total relative distance travelled by the car = [40 + 60] m = 100 m. The relative speed = 200 m / s = 20 m/s. 10 40 200 m/s = m/s. 3 15 Relative speed of the two trains (when they move in opposite directions) Level – I 1. Speed of first train = 100 m = 5 m / s. 20 sec The relative speed is also equal to (30 – v) km/hr  5  = (30 – v)×   m/s, where ‘v’ is the speed of the  18  200 + 200 = 12 s 100 / 3 As distance covered by faster one is 4 times the distance covered by slower one while their speed ratio is 3 : 2. So faster one is moving downstream and slower one is moving upstream.  6 +u 4 =  So   4 – u 1 They will meet first time in (6 + u) t + (4 – u) t = 100; t = 10 s. For second time meeting, faster is coming back to its starting point after reaching starting point of slower one. 20 80 S low e r B Fa ster A 20 m will cover by A to reach slower’s starting 20 = 2.5 s to reach B. In 8 this time slower one has cover 2 × 2.5 = 5 m. Now, A has to cover (20+ 5 = 25 m) with respect to B to catch B. Now A is also moving upstream so his speed is now 6 – 2 = 4 m/s. So relative speed of A with respect to B is 4 – 2 = 2 m/s. point. So he will take 25 = 12.5 s . 2 So in all = 10 + 2.5 + 12.5 = 25 s. They will meet second time after 25 s. So A will take bus. Therefore, (30 – v) × 5 = 5 or, v = 12 km/hr. 18 Explanations: Fundamentals of Time, Speed & Distance MBA Test Prep Page: 93 8. Speed downstream = v + u (where, v = Speed of the boat in still water, and u = Speed of the stream). Speed upstream = v – u. Distance downstream = Distance upstream (v + u) 45 = (v – u)75 or, u : v = 1 : 4. 9. a. Ratio of distances = Ratio of speeds = 20 : 23 :27. b. Let us calculate the time of meeting of A and B for the first time. A will meet B after 13. At 10 a.m. a train from station B is just coming to station A. That train must have started at 5 a.m. from station B. The train that starts at 10 a.m.. will reach at 3 p.m. at station B. So from 5 am to 3 p.m. every train started from station B will cross the train. So total number of train = 11. 14. From A a train is starting at 10 a.m. So from B every 1 train will start at an (integer + ) hr. interval i.e. 5 : 2 30, 6 : 30, 7: 30 .... and so on. 200 200 = s. 23 − 20 3 This means that A and B meet every Similarly, B meets C every So the 10 a.m train from A will meet first train which is suppose to reach at 10 : 30 starting from B at 5 : 30. Also, 10 a.m. train from A will reach station B at 3 p.m. So the last train it will cross is the one started at 2 : 30 p.m. from station B. So from 5 : 30 to 2 : 30, we have a total of 10 trains. 200 s. 3 200 200 = 27 − 23 4 = 50 s. Therefore, LCM of these time periods will give us the time when all the three meet the first time 15. So at 10 a.m. a train is just reaching station A that started from B at 7 a.m. So, the train that left at 10 a.m. train will reach at 3 p.m. Thus, from 7 a.m. to 3 p.m. each train will cross the train from B that started at 10 a.m. So total number of trains = 8 + 1 = 9 trains. 16. In this case we cannot find the exact number of trains crossing because we do not know the starting time of any train started from station B. May be possible it will reach station A at 10 a.m. or at 11 a.m. If it reaches at 10 a.m. to station A, then it starts at 5 a.m. and 10 a.m. train will reach at 3 p.m. So total number of trains crossing. = 5 a.m., 7 a.m., 9 a.m., 11 a.m., 1 p.m., 3 p.m. = 6 Or if train started from B reach A at 11 am should start from Bat 6 a.m. So number of crossing trains: = 6 a.m., 8 a.m., 10 a.m., 12 a.m., 2 a.m. = 5 trains. 17. They will finish 100% of the work in  200  , 50  which is LCM   3  = 200 s or (3 min. 20 s). 10. A’s speed is 2 m/s with respect to B. (i.e. relative speed) So to catch B he has to overcome 250 m because B is 250 m ahead of A. So A will take = 250 = 125 s . 2 x 11. B P Q A 12. Let the train increase its speed after ‘t’ hours. Total time taken by the train = 1 hr 15 min = 1.25 hr. Therefore, 40t + 50(1.25 – t) = 55 km. Or t = 45 min. Page: 94 = 12 days. Therefore, 75% of the work will be done in 75% of 12 = 9 days. –x 500 At the first time they will meet at point P. After that they meet for the second time at point Q. Up to point P distance travelled by A and B is add up to 250 m. After that when they meet at Q together they have travelled one circle. So total distance travelled by them is 750 m [250 + 500]. So time taken by them to cover this much distance is (5 + 3)t = 750 750 = 93.75 t= 8 20 × 30 20 + 30 18. Method 1: Let a, b and c be the times taken by A, B and C to do the work alone. Also let x = 1 1 1 ,y= ,z= . a b c 1 1 1 , y+z= ,z+x= . 20 15 18 From the last two equations, y – x Therefore, x + y = = 1 1 1 – = . 15 18 90 MBA Test Prep Solution Book-2 Therefore, 2x = = 1 1 7 – = , which means x 20 90 180 21. Ratio of distance = Ratio of speed = 32 : 41. Therefore, distance travelled by both the trains = 32x and 41x. According to given condition, 41x – 32x = 45 km Therefore, x = 5. Hence, distance between A and B = 41x + 32x = 365 km 22. Let the trains meet at some place M between A and B. Time taken from A to M = time taken from B to M. Therefore, ratio of speed = ratio of distance = 4 : 3. 23. A takes a lead of (3.5 km/hr) (2.5 hr) = 8.75 km over B. Hence, when B starts he has to travel a relative distance of 8.75 km and at a relative speed of (4.5 – 3.5) km/hr = 1 km/hr, which means B will take 8.75 hr to catch up and then overtake A. 24. In one day, the time gap between the two faulty watches is 6 + 4 = 10 min. The two watches will show the same time next when the faster watch takes a lead of exactly 12 hr over the slower. This would happen after 7 360 or a = days. 360 7 Method 2: Assume that the number of units of work is 180 units. A and B put in 9 units per day B and C put in 12 units per day; and A and C put in 10 units per day. (9 + 10 + 12) 2 units per day, i.e. 15.5 units per day. So A alone puts So A, B and C working together put in in 15.5 – 12 = takes 180 × 7 2 units per day. So he 2 360 days to complete the job = 7 7 working alone. Level – II 19. 1× 12 × 60 = 72 days. In 72 days the faster watch 10 would gain Let the distance travelled by the man be x km. Therefore, time taken at a speed of 10 km/hr = 72 × 4 = 4.8 hr = 4 hr and 48 min. 60 So the time according to the faster watch would be 7:48 p.m. and the slower would be 12 hr behind but would also show the same time. x hr. 10 Time taken at a speed of 20 km/hr = x hr. 20 x x 45 − = 10 20 60 Solving, we get x = 15 km. Therefore, Hence, 25. On that particular day it completes x x 15 15 – – = = 0.225 hr = 13.5 min 25 40 25 40 20. But in half time due to engine problem it takes 2 6 60 When he moves at 12 km/hr, time of travel = t − 9 60 Difference in time taken in the two cases 6   9  1 3 1  + = hr = t + =  − t −  60   60  10 20 4 If x km is the distance between school and house, difference in time taken = 1 the journey in 2 same time (12.5 hr). Let us say ‘t’ hr is the usual time. When Sanju moves at 10 km/hr, time of travel = t + Normal day it travels a distance = 25 (50 + u) 1 hr 2 extra. So it take 15 hr to complete the remaining half journey. 1 × 25(50 + u) 2 30(40+ u) = 25 × 50 + 25 u 30u – 25u = 1250 – 1200 5 u = 50; u = 10 km/hr So, 15(40 + u) = x x x − = 10 12 60 Therefore, x 1 = hr . 60 4 or x = 15 km. Explanations: Fundamentals of Time, Speed & Distance MBA Test Prep Page: 95 26. Ratio of speed of Bhim and Arjun = 7 : 4. (Since the ratio of the times = 4 : 7) a. If the length of circular track = 28m, the speeds of Bhim and Arjun are 7 and 4 m/min. The time when they are together the first time will be when Bhim (the faster one) has taken one round more than Arjun (the slower one). Therefore, if time when they meet is ‘t’, then 7t 28 – 4t = 28, which means t = min. 3 b. They will meet at the starting place the first time at a time which is the LCM of the times each one of them takes to reach the starting place. Therefore, LCM of 4 and 7 is 28 min. 29. c. 30. Diametrically opposite point is at a circular distance of 14 m. Bhim reaches this point in Method 2:  (49 + 25)  x=  ×145 = 429.2 km  25  As A starts puffing, the length of burnt cigarette in  → B →  one cycle is  A  gap gap   14 14 = 2 min and Arjun reaches this point in = 7 4 3.5 min. Bhim reaches this point in the 2nd min, 2 + 4 = 6th min, 6 + 4 = 10th min ... so on. Arjun reaches after 3.5 min, 10.5 min, 17.5 min. so on. The time after the start when Bhim reaches the point is a natural number, whereas the time when Arjun reaches this point will always be a non-natural number. So they will never meet. Method 1: Let the total distance between A and B be x. Since B had to travel 145 km to reach A, this means they met at a distance of 145 km from A. Hence, distance between the meeting place and B = (x – 145) km. Ratio of speed = Ratios of distance 25 : 49 : : 145 : (x – 145). Therefore, x = 429.2 km. =2×3+3+3×3+3 = 6 + 3 + 9 + 3 = 21 mm.  63  = 3 , As there are three such cycles Q  21  time taken in one cycle = 2 + 3 + 3 + 3 = 11s. So total time = 3 × 11 = 33 s. 1 31. 5 G oing o ut C o m ing in So the hour hand and minute hand move 360° in this time (together). 1° per minute and 2 speed of minute hand = 6° per minute. Let t is the required time in minutes. As the speed of the hour hand = When Bhim gives Arjun a lead of 4 min, in 4 min Arjun travels 16 m. Bhim now starts running towards Arjun at a relative speed of 3 m/min. To catch up So 6t + with a distance of 16 m, Bhim takes Page: 96 4 5 the time when they would meet for the first time. 28. 3 4  28   min to take a lead of one round. This is take   3  16 min. 3 Let x be the distance between A and B. The train which leaves A travels a total distance of x + (x – 200) = 2x – 200. The train which leaves B travels a total distance of x + 200. The ratio of distances travelled by the trains = Ratio of the speeds. Therefore, (2x – 200) : (x + 200) : : 40 : 60 Or x = 250 km. 2 3 Alternatively for (a) and (b): If the time when they would meet for the first time at the starting point = LCM(4, 7) = 28 min, in this time Bhim does 7 rounds and Arjun completes 4 rounds. So Bhim takes a lead of 3 rounds. Hence, he would 27. 1 2 1 t = 360° 2 13 t = 360° 2 32. t= 720 min. 13 Let us say total work is of 20 × 20 units. Each woman does 1 unit per day. Each man does 2 units per day while each boy 1 unit per day. 2 So on first day 20 units are completed. On second day 21 units, on third day 20.5 units and on fourth day 21.5 units are completed. does MBA Test Prep Solution Book-2 33. 34. Combining two terms, we get 41 + 42 + 43 + ... + 48 + 49 = 405 > 400. So there are 9 such pairs. So on the 18th day work will be complete. Let the inlet pipe take ‘t’ minutes to fill the tank alone. Let the total work be of 4,00 units. So a woman does 1 unit per day, while a man does 2 units per day. So work done on first day = 20 units and work done on second day = 21 units. Thus, the pattern goes like 20 + 21 + 22 + ... + 30 = 275 275 + 31 + 32 + 33 = 371 up to 13th day, 371 units has been completed. On 14th day job will be complete. Method 2: The net inflow is 50 l per minute. Hence, the actual inflow = 80 l per min (50 + 30). So the inlet pipe will Then  2000   = 25 min to fill the tank independently. take   80  39. C can start the work on first day but there is nothing to demolish then. So this day is wasted. The work effectively starts from second day onwards. Let the total work be of LCM(10, 15, 20) = 60 units. Now A can do 6 units in a day. B can do 4 units in a day. C can demolish 3 units in a day. So in 3 days (starting from A) 6 + 4 – 3 = 7 units will be complete. So in 8 × 3 days, 8 × 7 = 56 units of work will be done by A, B and C. The rest 4 units 36. Let the total job be of LCM(10,15) = 30 units. A + B can do 3 units per day. B + C can do 2 units per day. A + B + C can do 5 units a day. A + 2B + C can do 5 units. Here it can be clearly observed that B’s share in the total work is null. Hence, he cannot complete the job by himself. So, 1 1 2 1 + + = t 2t 3t 3 1  1 2 1 × 1 + +  = t  2 3 3 1  13  1 ×  = t  6  3 t = 6.5 hr. Level – III Let the leakage take ‘t’ minutes to empty the tank completely. When all the three pipes (two inlets and 1 1 1 1 – = + 10 20 t 25 or, t = 100/11 = 9.09 minutes (approximately). 1 m/s per second. 2 Thus, reduction in the speed of the slower truck = Method 1: The outlet pipe will discharge the water in 1 m/s per second. 2 As A alone can do the job in 10 days and A and B together also taking 10 days to complete the job. So 100% of A = 60% of A + B’s efficiency. So B will take one leakage) are open, then 38. t + 2t 3 = t hr . 2 2 36 km/hr ⇔ 10 m/s 18 km/hr ⇔ 5 m/s It is mentioned that the reduction in speed of the faster truck = 1 m/s per second. Note that the brakes are applied successively. Thus, the faster truck travels 9 m in the 1st second, 8 m in the 2nd second, 7 m in the 3rd second and so on. Final speed of the faster truck = 0 m/s ⇒ Truck stops at the 10th second. Since both the trucks stop simultaneously, speed of slower at 10th second = 0 m/s If the reduction in speed of the slower truck = ‘a’ m/s per second, (5 – 10a) = 0 10 = 25 days . 0.4 37. A → First pipe B → Second pipe C → Third pipe A can fill in t hr. B can fill in 2t hr C can fill in 2 will be done by A in day . So total number of days 3 2 2 = 1 + 8 × 3 + = 25 days. 3 3 35. 1 3 1 = – . Therefore, t = 25 min. t 200 40 40. ⇒a= 2000 200 = minutes alone. 30 3 Explanations: Fundamentals of Time, Speed & Distance MBA Test Prep Page: 97 Evaporation which is similar to a leakage Alternative Method: Ratio of speeds = 18 : 36 = 1 : 2 Since both the trucks are equally efficient, the reduction in the speed of the slower truck must be 1 m/s per second 2 Now, distance travelled by the first truck in 10 seconds = (9 + 8 + 7 + 6 + … + 1 + 0) = 45 m Distance travelled by the second truck in 10 seconds = (4.5 + 4 + 3.5 + 3 + … + 0.5 + 0) = 22.5 m Thus, net separation between the trucks when they notice each other = 45 + 22.5 = 67.5 m 41. 42. 45. The rate of burning of first candle = 1 cm/hr. The rate of burning of second candle = 1.5 cm/hr. They will be of the same length when the longer candle melts by a relative length of (10 – 8) = 2 cm. Therefore, time after which they will be of the same After 4 hr, the candle of length 8 cm is now 8 – 4 = 4 cm long which would be of the same length as the length of the other candle. Rate of flow for the first pipe = λ(1)2 = λ. Rate of flow for the second pipe = λ(2)2 = 4λ. Rate of flow for the third pipe = λ(3)2 = 9λ. Since rates are inversely proportional to time taken by each to fill the tank, time taken by the 3 pipes are 9 min and 1 min respectively. Therefore, 4 when all the three pipes are opened, the fraction of the tank filled in 1 min 9 min, 3t = 6 Hence, t = 2 Distance travelled by bus is equal to area under 1 1 × Vm1 × 2 + Vm1 × 2 + × Vm1 × 2 2 2 = 4Vm1 [Vm1 = Bus (maximum speed)] Distance travelled by jeep. the curve, i.e. The thief travels for 2 hr (7.00 p.m. to 9.00 p.m.) and takes a lead of (4.5 km/hr)(2hr) = 9 km. The policeman is required to cover up a distance of 9 km at a relative speed of 1.5 km/hr. Therefore, the policeman will take 6 hr after 9 p.m. to catch up with the thief, i.e at 3 a.m. 2 cm length = [1.5 cm / hr – 1 cm / hr ] = 4 hr. 43. 2.5 × 40 = 1 l / hr. 100 So total leakage is 3 l/hr. So added water in an hour is 4 – 3 = 1 l So it will take 40 hr. = 1 1 V × 2 + Vm2 × 2 + × Vm2 × 2 2 m2 2 = 4Vm2 [ Vm2 = jeep’s maximum speed] So 4Vm1 + 4Vm2 = 280 Vm1 + Vm2 = 70 Vm1 – Vm2 = 10 Vm1 = 40 km/hr Vm2 = 30 km/hr 46. Average speed by bus = Dis tance travelled by bus Time taken by bus = 4Vm1 6 2 80 km / hr × 40 = 3 3 Average speed by jeep = = Dis tance travelled by Jeep Time taken by jeep = 4Vm2 6 = 2 × 30 = 20 km / hr 3 1 4 1 14 + + = 9 9 1 9 The time taken by all the pipes to fill the tank is = 9 min. (λ is the constant of proportionality) 14 44. Tap 1 can fill the tank in 10 hr. So tap 1 gives 40 = 4 lit / hr . 10 Similarly, tap 2 leaks = Page: 98 40 = 2 l / hr . 20 MBA Test Prep Solution Book-2 Solutions Exercise 1 (Level 1) 1. a 5. e A takes 7 × 9 = 63 hr. ∴ in 1 hour A does = 1 of work 63 Assume that units to fill up = LCM of (20, 15, 12) = 60. Units filled up by A, B, C in 1 min = 3, 4, 5 respectively. Time taken by (A + B + C) to fill up = B takes 6 × 7 = 42 hr. ∴ in 1 hr B does = A + B in 1 hr = A + B in 1 1 + 63 42 1 =3 0.33 1 part 80 ∴10 days’ work = 10 1 = part 80 8 Work to be done by B = (1 – = 6, 5 and ( −3) units respectively. ∴ A + B = 11 units, C = –3 units ∴ Per hr intake = 11 – 3 = 8 units Time to fill up = 60/8 = 7.5 hr. 7. d 1 7 )= 8 8 Number of days required by B = 3. b 8. a 4. b 60 60 + =9 12 15 3 min total intake in units = 9 × 3 = 27 Units to be filled = 60 – 27 = 33 Time taken by B = 33/4 = 8.25 (8 min 15 sec) 336 = 48 days 7 9. c Pipes A + B both are there, 1 1 1 + = , say A takes x hr to fill then A B 12 B will take x – 10 hr to fill the tank. ∴ 80× 48 = 30 days 128 Let us assume that work done by A and B in 1 day are 2 and 1 unit respectively. Work done in 1 day by (A + B) = 3 units ∴ Work done in 14 days by (A + B)= 14 × 3 = 42 42 = 21 days 2 Assume total wages = 21 × 28 A’s wages for 1 day = 21 × 28/21 = 28 B’s wages for 1 day = 28 × 21/28 = 21 A + B’s wages of 1 day = 49. A + B’s wages will last for 21 × 28/49 = 12 days Explanations: Fundamentals of Time, Speed & Distance Say, capacity of tank = LCM (12, 15) = 60 units 1 min total intake in units = 1 1 128 + = 80 48 80× 48 ∴ A finishes the work in = 90 = 10 units per hr. 9 90 = 9 units per hr. 10 ∴ Resultant outflow = 10 – 9 = 1 unit per hr. Total outflow = 90 units, Min. = 90/1 = 90 hr. 7 7 7 , x= = 8 42×8 336 ∴ Number of days = Capacity in units = LCM (9 ,10) = 90. Inflow, when there is a leak = Let ‘B’ do ‘x’ part of the work in 1 day A+B= 60 60  -60  , ,   10 12  20  Inflow, when there is no leak In 1 day, A does = x × 42 = Assume that capacity of the tank = LCM (10, 12, 20) = 60 units. Rate of A, B, C per hr = 1  42 42  1 + hr =  = 0.33 × 5  63 42  5 ∴ Number of days = 2. e 6. c 1 of work 42 60 = 5 min. 12 1 1 1 x –10 + x + = ⇒ x x – 10 12 x ( x –10 ) = 1 2x − 10 1 ⇒ = 12 x ( x − 10 ) 12 24x – 120 = x2 – 10x x2 – 34x + 120 = 0 x2 – 30x – 4x + 120 = 0 x(x – 30) – 4 (x – 30) = 0 (x – 4) (x – 30) = 0 x = 4, 30 ∴ 30 hr ( Q x cannot be 4) MBA Test Prep Page: 99 10. d (a) Distance = Speed × Time = 48 × 10 = 480 km (b) To cover the same distance in 8 hr. 17. d the time = d 480 = 60 km/hr Speed = = t 8 ∴Speed must be increased by 60 – 48 = 12 km/hr 11. a Let his usual time be t hr and his usual speed be s km/hr. Distance, d = st = 18. b = SA 3 = SB 4 19. b 110 =11 : 9 90 To cross the platform the train has to cover 165 + 110 = 275 m. 5 110 = m/sec 132 km/hr = 132 × 18 3 t= 13. a Thief starts at 2.30 p.m. Policeman starts at 3.00 p.m. In 30 min (= 1/2 hour), the thief covers 1 60 × = 30 km 2 At this instance, the distance between them = 30 km difference of their speeds = 75 – 60 = 15 km/hr ∴ Policeman will overtake the thief in 14. d 15. b Since ratio of speeds of A : B = 3 : 4. So, ratio of time taken would be 4 : 3. If A takes 30 min more, then 4x – 3x = 30 min ⇒ x = 30 min. A takes 4x min, or 4 × 30 = 120 min = 2 hr. Average speed = = 16. d = 2xy 2 × 24 × 36 = x+y 24 + 36 144 = 28.8 km/hr 5 Average speed = Total distance Total time 2500 + 1200 + 500 4200 = = 420 kmph 2500 1200 500 10 + + 500 400 250 Page: 100 20. a 275 × 3 = 7.5 sec 110 t = 10 sec, 5 = 25 m/s 18 Length = s × t = 25 × 10 = 250 m Speed = 90 km/hr = 90 × 21. d 30 hr 15 = 2 hr, i.e. at 5.00 p.m. 27 min = 13.5 min. 2 Speed of A = 3x km/min Speed of B = 4x km/min distance is same. d = SA × t A = S B × tB 3x × tA = 4x × 36 tA = 48 min 3 s × (t + 2.5) 4 By the time the trains crossed each other, one of them had covered 110 km and the other had covered 90 km. Ratio of speeds = Ratios of the distances covered 54 = 27 min. 2 So, time taken be A = Or, 4t = 3t + 7.5 t = 7.5 hr 12. c A = 2B, B = 2C. C takes 54 min. So, B would cover the same distance in half of Total length to be covered = Length of train + Length of platform = 900 + 300 = 1200 m Total time = 60 + 12 = 72 sec Speed = 22. b 1200 50 50 18 m/s = × = = 60 km/hr 72 3 3 5 A + B = 72 days, B + C = 120 days, A + C = 90 days. Let total units of work = 360 … ( = LCM (72, 120, 90 )) ∴ (A + B) do units of work in 1 day = (B + C) do units of work in 1 day = 360 = 5. 72 360 = 3. 120 360 = 4. 90 ∴2(A + B + C) units in 1 day = 5 + 3 + 4 = 12. (A + C) do units of work in 1 day = 12 =6 2 ∴ A units in 1 day = 6 – 3 = 3 A + B + C units in 1 day = A will finish 360 units in MBA Test Prep 360 = 120 days. 3 Solution Book-2 Solutions Exercise 2 (Level 1) 4t – 1. d Let total units of work = LCM of (12, 16) = 48. Work done by: (A + B) in 1 day = 48/12 = 4 units. (B + C) in 1 day = 48/16 = 3 units. (A + B) worked for 5 days ⇒ 4 × 5 = 20 units done. (B + C) worked for 2 days ⇒ 3 × 2 = 6 units done. Work remains = 48 – 26 = 22 C finishes in (13 – 2) days = 11 days. ∴ Units of work in 1 day by C = 2. c 1 1   1  40 + 30    8. c At 5 km/hr t 2 = 1/ 3d 1 d hrs = 5 15 Let his normal speed be s km/hr. Let his normal time be t hr 1  d = st ⇒ 4  t −  = 3  t + 1  6  6  Explanations: Fundamentals of Time, Speed & Distance Length of train= 110 m, Speed of train = 58 × 5 m/s = 16.11 m/s 18 Speed of man = 4 km/hr = 4 × days Time = 10. a 110 m 110 = = 7.33 sec 16.11 − 1.11 15 108 + 112 220 × 18 = 5 5 (50 + x ) (50 + x ) × 18 250 + 5x = 660, x = 82 km/hr 11. b 5x = 410 Length of the faster train = (36 + 45) × 12. d 5 = 1.11 m/s 18 Length of first train = 108 m. Its speed = 50 km/hr Length of second train = 112 m. Let its speed = x time to cross = 6 sec ⇒ 6= Total time = 60 + 24 = 84/60 hr 2 / 3d 1 = d hr 4 6 d=s×t 1 3 journey ( = 40 km ) in of time ( = 6 hrs ) 2 5 Total distance = 80 km Total time = 10 hr 80 = 40 + S2 × 4, S2 = 10 km/hr Let units of work = 18 units. In one hour, man completes work = 9 units (= 6 units + 3 units) ∴ Time taken = 2 hr. d d 84 = Total time = + 6 15 60 Or, d = 6 km 6. d 44 25 = 5t + 60 60 t = 19/60 hr = 19 min 4t + 9. d 1 1200 days = 17 days. 7 70 At 4 km/hr t1 = Let normal speed be s km/hr. Let normal time be t hr. 11  5   d = st ⇒ 40  t +  = 50  t + 60  60    10 men in 20 days or 20 women in 15 days. ∴ 5 men in 40 days and 10 women in 30 days. = 5. c 48 = 24 2 1 = 1.5 days 2 ⇒ 5 men + 10 women in 4. b 7. d Let total work = 12 × 8 = 96. In 6 days, 12 men complete work = 12 × 6 = 72 ∴ Work left after 6 days = 96 – 72 = 24 Now, number of men = 12 + 4 = 16 ∴ Time = 24/16 = 1 3. b  7 1 Distance = 4 ×  −  = 4 km 6 6 22 =2 11 ∴ Number of days required for C = 4 3 7 = 3t + , t= hr 6 6 6 5 5 × 8 m = 81 × × 8 = 180 m 18 18 Let x be the rowing speed of the man in still waters Speed of the river = 2 km/hr = ‘y’ Speed upstream = x – y km/hr Speed downstream = x + y km/hr (x – y) 2t = (x + y) t 2 (x – 2) = x + 2 x = 6 km/hr MBA Test Prep Page: 101 13. b Let length of train be l m and its speed be s m/s. It crosses a pole in 15 sec 17. a … (i) ∴ l = 15 s It crosses a platform 100-m long in 25 sec 100 + l = 25 s Solving (i) and (ii) 100 + 15s = 25 s ∴ ∴ s= 14. a 15. c … (ii) or 1 1 1 1 + + = 22 33 44 x or x = 100 =10 m/s, ⇒ l = 150 m. 10 18. b The time taken to cover 240 km without any stops = 6 hr. Since he stops every 80 km, he would stop twice before reaching the destination. Hence, total time taken = 6 hr 40 min. 11× 12 132 2 days = = 10 6 + 4 + 3 13 13 Here the total distance = 180 km. Total time = 30 60 90 2 2 6 38 hr . + + = + + = 45 90 75 3 3 5 15 Average speed = 180 ÷ 38 1 = 71 km / hr. 15 19 Let ‘l’ = Length of train in metres. Then speed of the train = 19. d l l + 100 m/s = m/s . 15 30 ∴ l = 100 m 16. c 1 1 1 1 + + = 22 33 44 x when x = Number of days in which they together can finish the work. 1 1 1 1 + + = a b c 15 Alternative method: The ratio of speeds of Sujit and Rishi = 100 : 95 = 20 : 19. Similarly, the ratio of speeds of Rishi and Parveen = 20 : 19. ∴ The ratio of speeds of Sujit and Parveen = 202 : 102. ⇒ When Sujit goes 100 m, Parveen goes … (i) 5 25 25 + + =1 a b c 1 1 1 1 + + = … (ii) 5a b c 25 Subtracting (ii) from (i), 4 2 = ⇒ a = 30 days 5a 75 ⇒ Alternative method: Let the total work be of 15 units. They complete this work in 15 days. It means one day work is 1 unit. In 5 days, they will complete 5 units of work. Remaining 10 unit of work, B and C complete in 20 days. It means one day work of (B + C) = 361 × 100 = 90.25 m . 400 ∴ The lead that can be given is 100 – 90.25 = 9.75 m. 20. c 1 unit 2 1 1 = unit. 2 2 So total days taken by A alone to complete 15 units of work = 30 days. Suppose Pallavi takes t seconds to finish the race. It means Anuva and Richa would be taking (t + 50)s and (t + 90)s respectively to finish the race. When Pallavi runs 1,000 m, Richa will run 550 m ⇒ Richa runs 1,000 m in (t + 90) s. t + 90 1000 = of t = 110 s t 550 = 110 + 90 = 200 s. So we have ⇒ A’s one day work is 1 − Page: 102 When Sujit runs 100 m, Rishi runs 95 m. When Rishi runs 100 m, Praveen runs 95 m. ∴ When Rishi runs 95 m, Praveen runs 90.25 m. When Sujit runs 100 m, Praveen runs 90.25 m and is beaten by 9.75 m. 21. a Take (+) for the person walking down and (–) for walking up. Therefore, his relative position from the starting point = +4 – 3 + 6 – 2 – 9 + 2 = – 2. Hence, the answer is 2 steps above the starting step. MBA Test Prep Solution Book-2 22. a Since they move at the same time on different days, the situation is equivalent to the that of two persons where one moves uphill and the other downhill, both starting at the same time, viz. 7 a.m. Ratio of speeds of uphill to downhill = Ratio of distances = 10 : 15 = 2 : 3. Therefore, 1   1 +   work is done by (A + B) in 1 day  25 20  3 100 3 × = 6.66 work is done by them in = 5 9 5 days Number of days = 10 + 6.66 = 16.66 ⇒ 3 of 25 = 15 km from the top of the hill 5 2 of 25 = 10 km from the bottom of 5 the hill is the point where they will meet. 6. b they will meet or Solutions Exercise 3 (Level 2) 1. a  9  × 60  min Time taken to cover 9 km =   54  = 10 min 7. d Let speed of boat be x km/hr. Let speed of stream be y km/hr. 13 =x–y 5 Rowing upstream = … (i) 28 =x+y 5 Solving for y, y = 1.5 km/hr Rowing downstream = Let speed of boat in still waters be ’x’ = 6 km/hr Let speed of stream be ‘y’. x – y = 4.5 km/hr ⇒ y = 1.5 km/hr ∴ Rate along the stream = 1.5 + 6 = 7.5 km/hr 3. c Let the total work be (LCM of 45,40) = 360 units. In one day, A does 360 = 8 units and 45 8. d 4. d Let r lt/hr be the rate at which the hole empties the tank and let the tank has a capacity of V liters. The inlet pipe fills the tank at 4 liters/min or at 240 liters/hour, we have two equations, r×6=V …(i) & V + 8 × 240 = r × 8 …(ii) V = 5760 liters 10. b Length of bridge = 1 km Length of train = 0.5 km 5. d Time to clear the bridge = 2 min = Speed = 12 = 7.5 days 1.60 In 10 day’s A completes work = 1 2 ×10 = of the 25 5 11. a 2 hr 60 1 + 0.5 1.5 = × 60 = 45 kmph 2 / 60 2 Speed of stream = 1 km/hr Let speed of boat in still waters = x km/hr Total time = 12 hr 35 35 + x −1 x +1 Now put the value of x and check the options, only (a) satisfies. 12 = total work. Remaining work = 60 = 13.33 4.5 9. e A takes 12 days, B is 60% more efficient ⇒ B will take time = A:B=1:2 C : (A + B) = 1.5 : 3 If C does 1.5 units of work per day, A does 1 and B does 2. Total units done by C = 40 × 1.5 = 60 units In a day work done by (A + B + C) = 4.5 ⇒ Number of days taken by (A + B + C) to complete work = 360 = 9 units 40 Both A and B together do 8 + 9 = 17 units of work. In last 23 days, B does 9 × 23 = 207 units of work. So work done by both A and B together = 360 – 207 units = 153 units. 153 = 9 days. 17 1333.33 = 0.66 2000 ⇒ Total number of hr = 4.66 B does Number of days = A in 1 hr = 2000 B in 1 hr = 1333.33 ⇒ (A + B) in 2 hr = 3333.33 ⇒ (A + B) in 4 hr = 6666.66 In 5th hr work to be done = 8000 – 6666.66 = 1333.33. Number of hr required by A = … (ii) 2. d Due to stoppage, it covers 9 km less. 3 of the total work. 5 Explanations: Fundamentals of Time, Speed & Distance MBA Test Prep Page: 103 12. a Ratio of distances travelled by the trains when they meet = 9 : 10. Suppose first train travels 9x. Then the second would travel 10x km. ∴ Total distance = 10x + 9x = 19x km. We have 10x – 9x = 120 or x = 120 km. ∴ 19x = 19 × 120 = 2,280 km. Alternative method: It is given that speed is 45 and 50 km/hr respectively. So in 1 hr, the faster train moves 5 km extra than the slower one. It is given that the faster one moves 120 km extra than the slower one. Alternative method: Solve using the relative velocity concept, V is the speed of train. 10(V + 330) = 330 × 12 V = v 17. c 37 = 7.4 hr, which is same as 5 the time taken at the original rate. 18. e 1 = 30 km, 2 which is the relative distance to be covered by the owner moving at relative speed of (70 – 60) km/hr = 10 km/hr. ∴ Time taken by the owner to catch up with the Let ‘w’ is the time of walking and ‘r’ is the time of riding ∴ w + r = 7 hr. Riding both ways he gains 2 hr. Therefore, he takes 5 hr to ride both ways or 2.5 hr to ride one way. ∴ r = 2.5 hr ⇒ w = 4.5 hr. Time taken to walk both ways = 2 × 4.5 hr = 9 hr. 19. b From 7 a.m. to 1 p.m., i.e. in 6 hr, the first train travels a distance of 60 × 6 = 360 km. ∴ Distance between the trains at 1 p.m. = 1200 – 360 = 840 km. ∴ Time taken to cover this relative distance In half an hour, the thief travels 60 × Suppose the distance between the college and residence is d. Then 2  d d 2  40 min = hr  ⇒ d = 20 km − = 3  10 15 3  15. d = 20. c Average speed = 16. e When he travels the entire journey at 5 km/hr, total distance travelled = 35 + 2 = 37 km. ∴ Time of travel = 30 km = 3 hr which is at 5 p.m. thief = 10 km / hr 14. c 3 30 120 = 24 hr. 5 There relative speed is 95 km/hr. So distance travelled by them = 95 × 24 = 2,280 km. So they travel for 13. e 330 = 66 m / s 5 (50)(1) + (48)(2) + (52)(3) 1 = 50 km / hr. 6 3 If the person had not moved towards the source of the gunfire, he would have heard the second shot 12 min after the first shot. Since the person is actually moving towards the source, the shot is now heard after 10 min. It means the sound would have taken 2 min more to reach the initial position of the person; but this very distance was travelled by the train in 10 min. It means it is the speed of the 1 train in 10 min. It means speed of the train is of 5 the speed of the sound, i.e. = 66 m/s = Page: 104 330 m/s 5 66 × 3600 = 237.6 km / hr 1000 21. d 840 = 6 hr, i.e. 7 p.m. 140 In 6 hr, the second train travels 6 × 80 = 480 km. ∴ They meet at 480 km from Q. 3 4 Speed in upstream = 1 × 60 km/hr. 11 4 3 4 Speed in downstream = 1 × 60 km/hr. 7 2 Speed of man in still water 1 (Speed in downstream + Speed in upstream) 2 =  1  1  3  1 +   × 60    2  4   11 1 7 1  4 2  =  MBA Test Prep    = 5 km/hr.   Solution Book-2 22. b Let x be the distance per litre of petrol and v = Speed at which it is driven. ⇒ v′ = 30 km/hr Alternative method 2: Let the total work be 180 units. 10 men complete this in 18 days. So one day work of 10 men = 10 units. (All men are equally efficient, so one man’s work in one day is 1 unit.) In 6 days, 10 men will complete 60 units work. Remaining 120 unit is to be done by (10 + 5) men in Let a = Speed in upstream and b = Speed in downstream 120 days = 8 days. 15 Then x ∝ 1 v2 or xv2 = k = Constant. 25(36)2 = 36 ( v′ )2 ( v′ is the required speed) 23. b 40 90 60 60 + = 5 and + =5 a b a b ∴ a = 20 km/hr and b = 30 km/hr. ∴ Speed of the water (current) = Solutions Exercise 4 (Level 2) 1. c 1 (b − a) = 5 km/hr 2 3 men = 4 women. 4 women complete the job in 12 days. Hence, 5 women and 3 men, i.e. 9 women can do the job in Q {b = VW + VB and a = VB − VW } 2. c 24. a 25. c Let the job be completed in x days. Then the young man worked for 2 days. His father worked for x – 1 days and the grandfather worked for x days. ∴ 2 x −1 x + + =1 10 15 20 or 3 12 + 4x − 4 + 3 x = 1 or x = 7 days 7 60 To complete the given work, 10 × 18 = 180 man-days will be required. In 6 days, amount of work done = 60 man-days. Number of men now available = 10 + 5 = 15. ∴ Number of days in which the job can be done = 6 × 500 = Rs. 375 . 8 Earning of 4 women = Rs. 375. Earning of 3 men = Rs. 375. Earning of 1 man = Rs. 125. Alternative method: 5  3   4  The man earns 50 ×       = Rs. 125 . 4 2 3  3. b 4. a 3 of people becomes   times the earlier number, 2 2 the job would be done in   times the earlier 3 number of days, i.e. 8 days. Earning of 1 girl = Rs. 50. Earning of 10 girls = Rs. 500. Earning of 8 boys = Rs. 500. Earning of 6 boys = Rs. 120 = 8 days more. 15 Alternative method 1: After 6 days, 12 days of the job is left. If the number 12 × 4 1 = 5 days . 9 3 Amount of work to be done = 10n, where n = Number of workers originally available. Now 10n = 12(n – 5) ⇒ 2n = 60. Therefore, n = 30. 1 1 of a man = of a woman = 1 child. 3 2 ⇒ 2 men = 3 women = 6 children. 20m + 30w + 36c = 60c + 60c + 36c = 156c 78 . 156 Now 15m + 21w + 30c = 45c + 42c + 30c = 117c. If 156 children get Rs. 78, 1 child gets  78  ×117  per day. ∴ 117 children should get Rs.   156  ∴ For (18 × 7) days, they should get  78  ×117  = Rs. 7371. = Rs. (18 × 7)  156   Explanations: Fundamentals of Time, Speed & Distance MBA Test Prep Page: 105 Alternative method: Let a man, a woman, a child put in 3, 2, 1 units per day. Then they are paid Rs. 78 for putting in (20 × 3 + 30 × 2 + 36 × 1) units, i.e. 156 units. Hence, if (15 × 3 + 21 × 2 + 30 × 1), i.e. 117 units is 6. a 12n = 18(n – 5) ⇒ n = 15 7. c 4 of work is done by them in 7 days. 5 Hence, they take 117 per day or Rs. put in, then the payment = Rs. 2 7,371 in 18 weeks. 5. b 1 1 1 1 + + = a b c 18 job. ∴ … (i) … (ii) 1 1 3 + = a c b … (iii) 2 10 1.25 1 = −1= = a 8.75 8.75 7 a = 14 days Alternative method 1: A and B work together for 8 days. 1 1 2 − = or c = 54 days 18 c c WA + B in 7 days is 4 W [W is total work] 5 1 1 3 − = or b = 72 days 18 b b ∴ a = 43.2 days 4 8 32 × W = W ⇒ WA + B in 8 days is 5 7 35 (i) and (iii) ⇒ Now A leaves and B does Alternative method 1: Let the number of units of work = 18. ⇒ B will do W in 2 ×  1 C puts in   of that in 18 days = 6 units. 3 1 3 5 1 – = = 10 70 70 14 So A will do the work in 14 days.  18   × 18 = 43.2 days. In order to finish it he takes   7.5  Alternative method 2: Let the total work be 35 units (LCM of 7 and 5). 4 of work = 28 units, is done by A and B in 6 7 days. ∴ 1 day’s work of A + B = 4 units. B completes (7 – 4) units in 2 days. (Out of a total of 10 days, 7 + 1 = 8 days’ work = 8 × 4 = 32 units) ∴ 1 day’s work of B = 1.5 units. ∴ A’s 1 day’s work = 2.5 units. Total time taken by A alone to complete 35 units of Alternative method 2: Let the total work be 18 × 12 = 216 units. Ratio of (A and B) : C = 2 : 1. 1 × 12 = 4 units . 3 Ratio of (A and C) : B = 3 : 1. ∴ C does 1 × 12 = 3 units . 4 ∴ A does = 12 – (4 + 3) = 5 units. ∴ D does = 216 = 43.2 days. 5 35 70 = days. 3 3 = Hence, A does 7.5 units in 18 days. Total work will be done in 3 W in 2 days. 35 WA = WA + B – WB  1 B puts in   of that in 18 days = 4.5 units. 4 Page: 106 1 1 1 8 10 =1 + = and + a b a b 8.75 ⇒ 1 1 2 + = a b c (i) and (ii) ⇒ 7 = 8.75 days to complete the 4 5 work = 8. d 35 = 14 days . 2 .5 In 30 days, only half the work could be done. Therefore, number of men must be doubled to get the work done in half the time (i.e. half of 30 = 15 days, because only 15 days are remaining). MBA Test Prep Solution Book-2 9. c Alternative method: In 4 hr, lead taken by Shivku is (13 – 8) × 4 = 20 km. Now speed of Sujeet = 16 km/hr and of Shivku = 12 km/hr. Sujeet has to cover 20 km with relative speed 4 km/ hr in 5 hr. So total time = 4 + 5 = 9 hr. 15m = 24w = 36b x men must be associated.  36  x + 18 + 6  boys. ∴ (x)m + 12w + 6b =  15    36 x  30 × 6  12 × 8  Therefore,  = (36 ) + 24    15  21  1  4 10. a M aya nk A Suppose the work is of 150 units. So in a day A and B together do 15 units; B and C together do 10 units; and A and C together do 6 units. We have A + B = 15, B + C = 10 and C + A = 6. ⇒ A = 5.5, B = 9.5 and C = 0.5 In first 4 days, A, B and C together will do 4(5.5 + 9.5 + 0.5) = 62 units. In next 5 days B and C will do 5(9.5 + 0.5) = 50 units. So C alone has to do 150 – (62 + 50) = 38 units, that Let X be the point where they meet on the way. 38 = 76 days . 0.5 Let us assume that the distance between the two places be 60 km. This means that to go 120 km at 120 12 km/hr, she can use = 10 hr. 12 Also, as she goes at 6 km/hr, the time taken for this 60 = 10 hr. 6 Thus, there is no time to come back. ⇒ Alternative method: If distance from Kashipur to Bareily is D, Nupur takes D hr to travel distance D. 6 Since average speed of journey is 12, total time 2D D = . 12 6 Hence, there is no time to return. X AX 6 = [As their speeds are in this ratio] XB 5 t m dm / Sm dm SG Now t = d / S = d × S G G G G m dm AX 6 t m 6 6 36 ⇒ Since d = XB = 5 ⇒ t = 5 × 5 = 25 G G tm = 2.5 × 36 = 3.6 hr = 3 hr 36 min. 25 Alternative method 1: Conventional method of solving D 13. a Suppose the total time was x hr. Since distances travelled by both would be same, we have (8 × 4) + (x – 4)16 = (13 × 4) + (x – 4)12 or x = 9 Explanations: Fundamentals of Time, Speed & Distance B G M Let Golu and Mayank met at point C which is x kilometres from A, and A and B are D kilometres apart. VG 6 The ratio of speeds = V = 5 . M They take same time to reach point C. ⇒ x D – x = 6 5 taken is Suppose they meet after (100 + x) m. Shivku starts running when Pawan has already run 100 m. Now if Pawan runs x km, Shivku will run 2x km. So we have 100 + x = 2x or x = 100. Thus, they will meet at 100 + 100 = 200 m from the starting point. If the length of the track is 150 m, they will meet at a distance of 50 m from the starting point. C x A journey is 12. c B ⇒ x = 8 men he will do in 11. e G ollu 14. d ⇒x = 6 D 11 Now Golu covers So VG = 5D 5 11 × 2 5 5 D in hr. 11 2 = 2D km/hr 11 5 5 2 5D D= ⇒ VM = × VG = × 6 6 11 33 Mayank covers 5D in 33 hr. So he will cover MBA Test Prep 6 33 6 D in = 3.6 hr × 11 5 11 = 3 hr 36 min Page: 107 Alternative method 2: G 15. a 3 t t − t = = 45 min 2 2 or t = 90 min. Had the accident occurred 60 km away, the train would have been delayed by 1 hr only. Normal time M 2 VG = VM tM t 6 ⇒   = M tG 5 tG   ∴ tM = 36 5 × = 3 hr 36 min 25 2 2 of the speed, the delay will be 15 min only 3 (1 hr – 45 min stoppage) ∴ Normal time to cover the remaining distance ⇒ t = 2 × 15 = 30 min. It means the train covers 60 km in 60 min (90 – 30 min). Speed of the train = 60 km/hr. Total journey = 30 km + 90 km = 120 km. Due to Suppose the winning point is x metres far if the girls finish the race at the same time. Then Pallavi will have to cover x metres when Richa covers (x – 350) m. Ratio of their speeds is 20 : 13. 20 x = 13 x − 350 or x = 1,000 m = 1 km. So we have 18. a  200  used 8 gallons   for the journey.  25  Alternative method: Pallavi has to cover 350 m with a relative speed of 7x. 350 . 7x In this time, the actual distance covered by Pallavi 19. c Required time will be is 16. a 17. b 350 × 20x = 1,000 m, i.e. length of the track. 7x It happens when both of them meet for first time at the starting point. Ratios of speeds = 4 : 12. Hence, ratios of distances covered = 1 : 3. Hence, when A makes one round, B makes 3 rounds, so the answer is 1. ∴ x = 16 90 4 min . = 16 5.5 11 [Converting minutes to hours] Solving for x, we get x = 120 km and v = 60 km/hr. Alternative method: Overall, the train was late by 1 hr 30 min. Out of this for 45 min, the train was stopped. So due to 4 min 11 Alternative method: At 3 o’ clock, the angle between the hour hand and the minute hand is 90°. The minute hand will cover the distance with a relative speed of (6 – 0.5) = 5.5° per minute, i.e. v x− 1 3 2 − x = 11 + + 2v 2 4 v 2 3 v v x − − 60 + 60 3 x 2 2 + + − =1 Also 2v v 4 v 3 We know that in 60 min, the minute hand gains 55 min over the hour hand. At 3 o’ clock, the minute hand and the hour hand are 15 min apart. To cross the latter, the minute hand must gain 15 min. Since 55 min is gained by the minute hand in 60 min, 15 min is gained by the minute hand in 60 4 × 15 = 16 min. 55 11 Let length of the journey = x km and speed of the train = v km/hr. Then Vibhor had lost 4 gallons. Since he travelled at 50 mph for 4 hr = 200 miles, he 20. b Let’s assume the distance to be 24 km. (LCM of 6, 4, 3) Then time taken to cover that distance will be in the 24 24 24 : : = 4 : 6 : 8 = 2 : 3 : 4. 6 4 3 Hence, the ratio of their speed will also be 2 : 3 : 4. ratio 2 of the speed, the train delayed by 45 min 3 only. Page: 108 MBA Test Prep Solution Book-2 Alternative method: Since the distance travelled by the three is same, the ratio of speeds will be inverse of the ratio of time taken. 1 1 1 : : Hence, the ratio of speeds = 6 4 3 = 4 : 6 : 8 = 2 : 3 : 4. 21. c 23. a Fraction of the tank filled in 2 hr = 1 1 9 + = . 4 5 20 So in 4 hr, fraction of the tank filled = The remaining Ratio of speeds = 3 : 4. Distance remaining constant, the ratio of time taken = 4 : 3. A takes 0.5 hr more than B. Hence, time taken by A = 4 × 0.5 = 2 hr 18 . 20 2 1 or of the tank will be filled by 20 10 pipe A. Pipe A fills 1 of the tank in 1 hr. 4 1 1 of the tank in 4 × = 0.4 10 10 = 0.4 hr = 24 min. Hence, the total time taken to fill the tank = 4 hr 24 min. So it will fill 22. a 2 1 = . 6 3 Work done by B and C in 2 days Work done by A in 2 days = 1 1  5 + 4 9 = 2 +  = 2  40  = 20 .  8 10    Remaining work = 1− 24. c 1 9 60 − 20 − 27 13 . − = = 3 20 60 60 Hence, 13 part of the job will be done by C in 60 1000 = 200 s . 25 − 20 In 200 s, A will have made 13 60 = 13 1 = 2 days. 1 6 6 10 200 ÷ Total time taken to complete the job =2+2+2 Alternative method: Let the total work be LCM (6, 8, 10) = 120 units. A’s 1 day’s work = 20 units. B’s 1 day’s work = 15 units. C’s 1 day’s work = 12 units. In 2 days A will work = 40 units. (B + C) will work in 2 days = 54 units. Work remaining after 4 days = 26 units. ∴ Time taken by C = 26 13 = . 12 6 Total time taken = 4 + 13 days . 6 1000 200 × 25 = = 5 rounds. 25 1000 Alternative method: From the given data, in 1,000 m race A can give B a start of 200 m. It means when A has covered 1,000 m, B can cover 800 m. The ratio of their speeds is 5 : 4. From this ratio of speed, it can be easily found that they will meet only at the starting point. So A has covered 5 rounds. 1 1 = 6 days. 6 6 Answer is 6 A can give B a lead of 200 m for every 1,000 m, i.e. every one kilometre. This means that when A covers 1,000 m, B covers 800 m. Suppose the speed of A is 25 m/s and of B is 20 m/ s. A will meet B for the first time after 25. a When B covers 800 m A has covered 1,000 m. So when B has covered 3 rounds = 3,000 m, A will cover 3,750 m. But to meet at some point, the overall lead should be of one round = 1,000 m. So there must be an initial lead of 250 m. 1 days . 6 Explanations: Fundamentals of Time, Speed & Distance MBA Test Prep Page: 109 Alternative method: We see that the car reduces its travelling time by 20 min. So it reduces its one-way travelling time by Solutions Exercise 5 (Level 2) 1. d The hands of the clock are 35 min (= 35 × 6 = 210º) apart at 7 o’ clock. For the hands to be together, the minute hand has to gain 210° min over the hour hand. Speed of the minute hand relative to the hours hand = 6º – 1/2º = 5.5º per minute. Hence, the hands will meet after 20 = 10 min. 2 Thus, it would have met the children 10 min earlier, i.e. 3.50 p.m. Now this means that the children walked from 3 p.m. to 3.50 p.m., i.e. 50 min. 4. d 210 420 382 = = min. 5.5 11 11 OR Between x and (x + 1) o’ clock, the two hands will 12 = 11.11 hr. 27 Hence, the total time = 600 + 11.11 = 611.11 hr. 25 ×  12  be together at 5x ×   min past x.  11   12  2 min past 7. i.e. 5 × 7 ×   min past 7 = 38 11  11  5. d Alternative method: Since the hour hand is between 7 and 8, the minute hand has to show the time between 35 min and 40 min. Hence, there is only one choice in between them. 2. a If original speed and time are s and t, and new speed and time are t1 and t1 respectively, then s1 = 3. c 6. b 7 10t s ⇒ t1 = −t =1 10 7 or t = 7 hr = 2 hr 20 min. 3 Let X minutes be the time required for going to school from home. Usually the car leaves home by (4 p.m. – X) min and the children are back home by (4 p.m. + X) min. On Saturday, the children reached home 20 min earlier, i.e. by (4 p.m. + X – 20) min. ⇒ The total time for which the car was in use = 4 p.m. + X – 20 – [4 p.m. – X] = (2X – 20) min. Hence, the car was used for (X – 10) min to go and come back from school. If the children were walking for Y minutes, then ⇒ 3 p.m. + Y + (X – 10) = 4 p.m. + (X – 20) min ⇒ Y – 10 = 4 p.m. – 3 p.m. – 20. Hence, Y = 60 – 20 + 10 = 50 min. In 24 hr, i.e. 1 day, the snail goes up by only 11 inch. height of the pole = 12 × 25 = 300 inches. In 25 days, i.e. 600 hr it will go up (effectively) by 25 × 11 = 275 inches. Now it can creep up 27 inches in 12 hr. So to climb up 25 inches it will take Two of the dog’s leaps cover 4 m, three of the fox’s leaps cover 3 m, i.e. each time the dog runs 4 m. The distance between them is reduced by 4 m – 3 m = 1 m. The initial distance between them is 30 m. Hence, the dog will catch up with the fox when it covers 4 × 30 = 120 m. 148 = 8 hr, i.e. it reaches 8 hr 18.5 after 7 a.m., i.e. 3 p.m. So the second train reaches at 3.15 p.m. Ratio of speeds of two trains is 8 : 5. Hence, the ratio of time taken (distance being constant) is 5 : 8. Therefore, time taken by the second train The first train takes 5 =   × 8 = 5 hr, i.e. it starts 5 hr before 3.15 p.m. 8 ⇒ 10.15 a.m. 7. c Since Ajay is faster than Mallu and they start together, to meet at 10 m from B, Ajay would have covered a distance from A to B and would meet Mallu on his way back to A. Mallu would be on his way from A to B. So Ajay covers 200 + 10 = 210 m in 10 s. Hence, Ajay’s speed = 21 m/s. So he will take 190 s to cover the remaining 190 21 m. The time required for Ajay to reach A will be 10 + Page: 110 190 400 s. = 21 21 MBA Test Prep Solution Book-2 8. c In the first 15 min, the thief will cover Alternative method: 1 × 60 = 15 km. 4 Hence, to cover this 15 km, a policeman will take = 384 = 16 days for New York-Delhi travel. 24 Since the speed gets added up, when two people come from opposite sides, anybody going from New York to Delhi will meet two people going from Delhi It takes Dis tance 15 15 = = = 3 hr. Relative speed 65 − 60 5 to New York every 9. a When Zigma-S leaves Jaipur, the thief has covered a distance of 15 km. Distance between them = 300 – 15 = 285 km. Relative speed of Zigma-S with respect to the thief = 60 + 60 = 120 km. So time taken to catch the thief 285 = 2.375 hr = 2 hr 22.5 min. 120 According to question 8, Sigma-Z takes 3 hr to catch the thief. Hence, Sigma-Z takes 37.5 min more than Zigma-S in catching the thief. ∴ A person will take 11. a 13. b 16 hr to complete the race. 7x Since they move in the same direction, relative speed = 4x. 4 1 = . 4x x Therefore, number of times they meet in the entire Time when they first meet = 1 of the 8 16 x 7 = 16 race = = 2.3. 1 7 x work. Similarly, the second photocopier will complete 1 12 of the work in 1 hr. If both work together, then fraction of the work completed in 1 hr 1 1 3+2 5 . + = = 8 12 24 24 Hence, both the photocopiers Therefore, they meet twice before the race finishes. 14. d working 384 = 16 days. 24 Thus, the person starting from Delhi (at 3 p.m.) is bound to meet the 17 persons already enrouted, and 16 more (who will have started after he starts) during his journey. Therefore, the total number of people he meets = 17 + 16 = 33. 4 2 = . 10 x 5 x 16 x 7 = 80 Therefore, they meet = 5.7 times, 2 14 x 5 24 = 4.8 hr. simultaneously will take 5 i.e. 5 times. Duration of the journey = Explanations: Fundamentals of Time, Speed & Distance When they move in opposite directions, the time when they meet the first time = = 12. d Let the speeds of two persons are 3x and 7x. Y takes Let the total distance covered by them when they first meet is d kilometres. When they meet for the second time, the total distance covered by them is 3d. Hence, they will meet after 12 × 3 = 36 min. In 1 hr the first photocopier will complete 384 = 32 intervals to complete 12 his journey. ⇒ He will meet (32 + 1) = 33 people during the journey (As he will meet the first person as soon as he leaves New York and last person as soon as he reaches Delhi.) = 10. d 24 = 12 hr. 2 15. c 1 of the 6 tank. Hence, if the capacity of the tank = V m3, 60 In 1 hr, pipe A fills 60 m3, and pipe B fills v v = (Portion of the tank filled in 1 hr when both 6 4 A and B are open) 720 + 2V = 3V ⇒ V = 720 m3 + MBA Test Prep Page: 111 16. d 720 = 12 hr. 60 Therefore, 1 hr net task of all three pipes, when opened simultaneously, is Pipe A can fill the tank in and d =3 u−v or u−v 2 = u+v 3 Each maid takes 7 hr to clean a floor. Hence, 3 maids would also take 7 hr. Here the concept of chain rule can also be used, but the mop and the person should be taken as one entity. Since no work can be done if there are only mops or only persons without mops. 24. c Area per revolution = (2πR) × L = 1.32 sq. m ⇒ Total area = 1.32 × 400 = 528 sq. m ⇒ Total cost = Rs. 5280 25. d In a mile race Akshay can be given a start of 128m by of Bhairav. Or in 100m race Bhairav can give a start off 8 m to Akshay. So, for 100 m race: u 5 = . v 1 Thus, we see that we can get the ratio of the speeds and not the exact speeds. Let the job consists of 120 units [i.e. LCM (20, 24, 15)] ∴ A + B do 6 units per day. B + C do 5 units per day. A + C do 8 units per day. A + B + C do 9.5 units per day. So B + C in 8 days do 40 units. A in 4 days does 18 units. Hence, 19. e 10 190 min × 19 = = 4 4 Hence, total distance covered 180 190 1 × × ⇒ 7.5 km 19 4 60 PQ = 3.75 km = Let the speed of the boat in still water is u Let the speed of the flowing river is v when boat goes upstream (against the flow of river) speed of the boat = u – v for downstream (with the flow of the river) =u+v 4 : 1 = (u + v) : (u – v) ⇒ u + v = 4u – 4v ⇒ 3u = 5v Page: 112 Bhairav 100 Chinmay 96 So Chinmay is faster than Akshay. Now Chinmay can give a start off 4 m in 96 m race to Akshay. So in 2,400 m race start given to Akshay 2 × 10 × 9 180 = km / hr 19 19 180 : 12 ⇒ 15 : 19 Ratio of times of B and A = 19 Hence, if A takes 10 min more than Bibek 20. a Akshay 92 58 29 = part of the job is completed. 120 60 Average speed of A = Assume speed of the river and the boat and make an equation by equating their time. Let M be the speed of the motor boat and R be the speed of the raft/river. Then 23. c or 18. c 2. 4 2. 4 + =1 5−x 5+x 40 28 4 M 17 + = ⇒ = 3(M + R) 3(M − R) R R 3 Let the speed of the boat in still water be u km/hr and that of the river be v km/hr. If the distance from A to B is d, then we have d =2 u+v Time taken = ⇒ x = 1 km/hr 22. c 1 1 1 2+4−3 1 + − = = . 12 6 8 24 8 Hence, the tank will be filled in 8 hr. 17. e 21. a = 4 1 × 2400 = 100 m = mile 96 16 Solutions Exercise 6 (Level 3) 1. a Let the speed of the goods train be X m/s, and that of the passenger train be Y m/s. In 28 s the goods train covered 28X (m), and the passenger train, 28Y (m). Therefore, 28X + 28Y = 700. The goods train passes the signal lights in and the passenger train in 490 s X 210 s. Y 490 210 – = 35 . X Y Solving the two equations, we get X = 36 km/hr and Y = 54 km/hr. Therefore, MBA Test Prep Solution Book-2 Short cut: Going from the given answer choices, we see that option (a): 36 km/hr = 10 m/s and 54 km/hr = 15 m/s. So the goods train takes 49 s  490   10  and the    210   . passenger train takes 14 s   15  Hence, the goods train takes 35 s longer than the passenger train. 2. c  16  + 11 hr or The two trains would meet after =   3  Let’s assume that A takes x days to finish the job. Then B will take 3x days. 3x – x = 60 ⇒ x = 30 days 1 . Work done by A in one day = 30 1 . Work done by B in one day = 90 at 4 : 20 p.m. 6. d As in question 5, speeds of the first train before and after the accident would be 1 km/hr and 0.5 km/hr respectively. Since they meet 7 hr after they leave, the distance covered by the first train till 4 p.m. = 12 – 7 = 5 km. Average speed of the first train till 4 p.m. = Using alligations, 1  2  1 + of the Both combined would do   =  30 90  45 job in one day. Hence, they together will do the job 45 days = 22.5 days. in 2 3. a Since the two trains have the same speed, they would take same time to cover the same distance (between A and B). Hence, they would meet exactly 6 hr after they start, i.e. at 3 p.m. 4. b If the distance between A and B is assumed to be 12 km, then relative speed of the two trains = 1 km/ hr. Therefore, by 1 p.m., they would have both covered 4 km each. Distance left between them = 4 km. Speed of the first train = 0.5 km/hr; speed of the second train = 1 km/hr. (Relative speed = 1 + 0.5 = 1.5 kmhr) Hence, time taken to meet, after the accident 1 2 5 7 Hence, the time for which the train travels at two speeds is in the ratio 3 : 4, i.e. after 3 hr of departure, the accident takes place. Hence, the accident takes place at 12 noon. Alternative method: Assume that the total distance d = 12 km. Speed of the train from A to B and B to A is 1 km/hr. 4 2 = 2 hr = 2 hr 40 min. 1.5 3 So they meet at 3.40 p.m. d B A Now if the accident occurred after t hours from start, the distance covered by the first train in t hours = t km. The train is late by 10 hr due to reduction in speed after the accident as the speed is half. So time taken now will be twice that of the original. Hence, the accident must have occurred 10 hr before the actual arrival time at the destination, i.e. at 11 o’ clock. Now till 11 o’ clock, if we assume the speed of each train to be 1 km/hr and total distance = 12 km, in 2 hr they together will cover 4 km. The remaining distance of 8 km will be covered with 1 km/hr. relative speed of 1 km/hr and 2 8 16 ∴ Time = = hrs 3 3 2 1 2 (Because after the accident the speed is half) (9 to 4 → 7 hr) Distance covered by the second train in 7 hr = 7 km. To meet at 4 a.m., the total distance = 12 km. = 5. c 5 km/hr. 7 Explanations: Fundamentals of Time, Speed & Distance And distance covered in (7 – t) hr = (7 − t ) × ∴ 7 + t + (7 − t ) × 1 = 12 2 ⇒ t = 3 hr after start, i.e. at 12 o’ clock. MBA Test Prep Page: 113 7. b Let the length of the wall be 30 m. Bamdev can demolish 1 unit per day and his son can demolish 0.5 unit per day. Manas can construct 3 units per day. If they work simultaneously with Manas starting they would do effectively 1.5 units per day. So in 36 days, 18 × 1.5 units is completed, i.e. 27 units is completed. Hence, on the 37th day, Manas would complete the job. Manas would have worked for 19 days and constructed 19 × 3 m. Hence, the fraction of the job completed 9. d = 57 19 38 = = = . 30 10 20 10. a 1 work in 1 day. 10 Also Bamdev and his son can demolish ∴ He does 23 min + 55 s ~ 51 s. 28 Let us calculate the tonne-hours done. 3 tonnes × 30 trucks × 8 hr = 720 tonne-hours. 5 tonnes × 9 trucks × 6 hr = 270 tonne-hours. ∴ Total tonne-hours = 720 + 270 = 990 tonne-hours ∴ Hours required for a single three-tonne truck 990 = 330 hr 3 Hours required for five-tonne truck 1 1 1 + = of the wall in next day. 30 60 20 Thus, in 2 days Manas effectively constructs = 1 1 1 – = of the wall. 10 20 20 = 20 × 9 9 ×2 of the wall in 10 10 = 36 days and on 37th day, he finishes the construction. Since he has worked for 19 days, he ∴ He constructs constructed 11. b 19 38 = portion of the wall. 10 20 If Manas started when there were 24 units to be completed, then working at 1.5 units per 2 days, 21 units would be completed in Let the rate of growth of the grass be x straws per day and the initial amount of grass be y straws. Assuming that one cow grazes on one straw of grass in a day, (40 cows) × (40 days) = 1600 = y + 40x. (30 cows) × (60 days) = 1800 = y + 60x. ⇒ y = 1200 and x = 10. So for 20 cows, (20 cows) × (n days) = y + nx = 1200 + 10n Alternative method: Let G be the amount of grass in the field and g be the growth rate of grass per day. ⇒ G + 40g = 40 × 40 and G + 60g = 60 × 30 Alternative method 1: ⇒ g = 10, G = 1200 2 of the 10 wall, i.e. 8 days of work is already done. ∴ So 37 – 8 = 29 days are required. This is identical to question 22, only that Alternative method 2: From choices it is evident that there is only one odd choice. We know that the answer has to be odd since on the last day Manas has to complete the job. 990 = 198 hr. 5 ⇒ n = 120 days.  21     1. 5  = 14 pairs of days or 28 days. Hence, on 29th day Manas would complete the job. Page: 114 4 min 18 s + 7 min 13 s + 12 min 24 s 6 + 9 + 13 = Alternative method: Manas can construct a wall in 10 days. 8. b In his first shift, he uses the container six times. (45 glasses in 5 rounds and the remaining 6 glasses in the 6th round) Similarly, he prepared 73 glasses in 9 rounds, and 112 glasses in 13 rounds. Hence, the required average Total time = Total number of rounds ⇒ 1200 + N × 10 = 20 N ⇒ N = 120 days 12. d t1 t2 Let t1 be the time at which B switches the speed and t1 + t 2 be the total time between start and finish. Let x be the speed of B initially. So A’s speed = 1.2x and B’s final speed = 1.44x. MBA Test Prep Solution Book-2 13. d Now lag of B in time t1 = (1.2 x – x )t1 = 0.2 × t1 20 = 4 days. 5 So time taken by 4 men to complete the job = 4 × 2 = 8 days. And time taken by 4 women = 8 × 2 = 16 days. Therefore, 12 men + 10 children + 8 women … (i) Also gain of B in time t 2 … (ii) = (1.44 x – 1.2 x )t 2 = 0.24 × t 2 Since both reach at the same time, ∴ Lag = lead ⇒ t1 0.24 6 = = t 2 0.20 5 complete = 1760 × 6 = 960 m. 11 14. a x 20 × 20 = 44% more 100 VB + t1 + 1.44 VB t 2 = 1760 6 t2 5 Now distance covered by A in t1 time = 1.2 VBt1 ⇒ t1 = 5   1.2 VB  t1 + t1  = 1760 6   1.2 VB t1 = 1760 × 6 = 960 m . 11 = 9 ⇒ x = 9y – 5 ⇒ 90y – 9x = 100 Solving (i) and (ii) x = 50 m Since race ended in dry heat, equating time of A and B ∴ 1.2 VB ( t1 + t 2 ) = 1760 5 m/s ) 9 ... (i) x Similarly, for second 9 y − 10 = 10 9 1.44 u Alternative method 2: Assume that A runs 20% faster than B for t1 time and for t2 time B runs 20% faster than A. Speed of B initially is VB for t1 time. So VA = 1.2 VB for (t1 + t2) time VB’ = 1.44 VB for t2 time (As 2 km/hr = 9 x1 x1 1760 − x1 1760 + = ⇒ x1 = 797.819 m . u 1.44u 1.2u 5 9y − 5 = 9 9 9y − 5 = 1.44 u u x = Length of train, y = Speed of train (m/s) Relative speed with respect to first one = y− Alternative method 1: Let the speed of B → u. Speed of A → 1.2u New speed of B = 20 + 20 + 3 1 1 (3 + 4 + 1) 8 + + = = 1 of the = 8 2 8 8 8 work. Hence, time taken = 1 day. 1760 × t1 ∴ A covers of the distance ( t1 + t2 ) = The time taken by 5 children to complete the job = y= ... (ii) 55 m/s 9 15. b When Ram completes second round, they pat each other once. In the 20th round, Ram will finish the race and the total number of pats = 20 – 1 = 19 This solution is only valid when greater ones speed is less than twice the first one speed. 16. c They reverse directions on reaching A. Ram with 12 km/hr speed will reach A first and immediately reverse directions. Suraj will reverse direction only after he reaches A. Time taken by Ram to complete the circle = 1.2 km/12 km/hr = 0.1 hr = 6 min Time taken by Suraj to complete the circle = 1.2 km/8 km/hr = 0.15 hr = 9 min In those extra (9 – 6) = 3 min, Ram would have already reversed direction and done another half circle in the same direction as Suraj. 1.2 The effective distance between them = 2 = 0.6 km Time taken to meet after Suraj reverses direction 0 .6 = 0.03 hr = 1.8 min (12 + 8) Therefore, time taken to meet second time = 9 + 1.8 min = 10.8 min = Explanations: Fundamentals of Time, Speed & Distance MBA Test Prep Page: 115 17. b Time taken will be double the time taken to meet the first time = 3.6 × 2 = 7.2 min Since each time the effective distance between them will be 1,200 m. 18. c The time taken by P and Q to meet after their start is 2. e For every day's work, he can afford to miss 3 days. Hence, to break even it has to be 7 days out of 28 days. 3. e In 1 day A and B together produce 1 1 3 + = units 15 12 20 ∴ In 20 days, 400 400 = = 4 hr 40 + 60 100 Hence, they must have started at 11 a.m. A and B together will produce Alternative method: which will fetch them Rs. 270. 3 Q covers of the distance, i.e. 240 km. 5 It takes 4 hr to cover this distance. 19. e R would take 3 × 20 = 3 units , 20 Now, 60 × 4 ⇒ 12 hr to meet Q. (80 − 60) Efficiency of B 15 5 = = Efficiency of A 12 4 ⇒ Share of B is 5 × 270 = Rs. 150 9 By that time, Q would have already reached Guntur. 4. a 20. a Normally P and Q meet after 4 hr of journey. Now they are meeting after 5 hr. Hence, the distance travelled by Q = 400 – (40 × 5) = 200 km If the accident occurred after x hours, then 60(x) + 20(5 – x) = 200 or 40x = 100 or x = 2 1. a 2 1 = of the remaining is to be scarified by 6 3 each. ∴ 1 hr 2 Solutions Exercise 7 (Level 3) 5. b Let ship sends the radiowave from point A and receives it at point B A 5 00 m M 12 bottles have to be shared by has 10 people, i.e. a man consumes 1.2 bottle and 50% has been consumed ⇒ 6 bottles or (0.6 bottle by each). 2 bottles out of 6 have to be scarified to give them their original share. V = Speed, N = Wagons ∴ The quantity by which speed is diminished = 40 –V ⇒ 40 – V = K N ; V = 28 and N = 16 ⇒ K=3 B ∴ 40 – V = 3 N When V ≥ 10, the least value of V = 10 km/hr ⇒ 40 – 10 = 3 N ⇒ N = 100 O O cea n b e d 0.5 1 time taken = = = 1 min 30 60 Distance travelled by radiowave in 1 min. = 200 × 60 = 12 km 6. d 1  28 x  1 x  +  =  12 16  192 Left capacity = 1 – 28x 192 ∴ OA = OB = 6km In ∆AOM OM2 = OA2 – AM2  1 = 62 −   2 ⇒ OM = Page: 116 2 This is filled by P in 5 min and fills ⇒ 1 in 1 min 12 192 − 28 x 5 ⇒ x = 4 min = 192 12 143 km 2 MBA Test Prep Solution Book-2 L 7. e Shallow end 12. b S1 S2 10.5m O 2 × 33 5 D eep end O 18.5m Trained labourer Days First they meet at point O, S2 L − 18.5 for which S = 18.5 1 y ...(i) as both stay at the ends for equal time, so time taken from the starting up to the second meeting(at O’) will be same. S L + (L − 10.5) ⇒ 2 = ...(ii) S1 L + 10.5 L − 18.5 2L − 10.5 = 18.5 L + 10.5 apply componendo & dividendo- Thus, y = 13. d Hours Work 15 12 1 11 9 1.5 2 15 3 12 × 33 × × × = 36 5 11 2 9 6 33 7 8 36 + = + = 2 and u v 4 u v Solving these, we get u = 16 and v = 24. Hence, rate of stream is 4 km/hr. from (i) and (ii) 14. b L 3L = ⇒ L = 45m L − 37 L − 21 8. b The two trains start at the same time. If they meet after x hr, then 21x – 16x = 60 ⇒ x = 12 ⇒ Distance = (16 × 12 + 21 × 12) = 444 km 9. c Ratio of time taken is 3 : 2. If difference is 1 min, A takes 3 min. If difference is 10 min, A takes 30 min. ⇒ At double speed A takes 15 min. 10. c 3 men = 4 boys Hence, 27 boys can reap a field in 15 days. 3 of the So 20 boys + 16 boys = 36 boys will take 4 time, i.e. 11.25 days. LCM of 224, i.e. (25 × 7) and 364, i.e. (22 × 7 × 13) = 25 × 7 × 13 25 × 7 divides this 13 times. So A does 13 rounds before meeting again. 22 × 7 × 13 divides this 8 times. So B does 8 rounds before meeting again. Thus, the difference in the number of laps is (13 – 8) = 5 15. b 11. c If A takes x seconds and B takes y seconds to run 1 km, then 960 960x + 30 = y y and 1000 1000 ⇒ y = 150 s and x = 125 s X's rate = Y's rate or 45 = Y ' s rate 150 × 5000 = 750 s 1000 16. d If the cross-sectional circumference of pipe A is 1 unit, that of B would be 2 units and that of C would be 3 units. So areas would be in the ratio 1 : 4 : 9 or the areas of B and C together is 13 times that of A. So it takes 1 3 × 16 min or 1 min to fill the 13 13 original tank. But now we have to fill the second tank. Thus, time required will be Explanations: Fundamentals of Time, Speed & Distance 200 5 = 288 6 ⇒ Y’s rate = 54 km/hr x + 19 = ⇒ answer = Time taken by Y to reach Jodhpur Time taken by X to reach Jaipur MBA Test Prep 16 × 2 32 = min . 13 13 Page: 117 17. c 18. c Alternative method: Let x be the initial speed. Equating time Let him walk at 4 km/hr for t hr and at 3 km for h hr Now 4 × t + 3 × h = 36 Also 4 × h + 3 × t = 34 Solving, we get h = 4, t = 6 Therefore, h + t = 6 + 4 = 10 hr 420 60 9 480 6 – + + = x x + 20 60 x 60 420 60 1 480 + + = x x + 20 4 x 1 man = 2.5 boys; 1 woman = 1.5 boys; convert all work in terms of boys. Thus, group 1 = 43 boys, group 2 = 107.5 boys. Hence, group 2 would take 25 × ⇒ 43 ×3 107.5 60 1 60 + = x + 20 4 x Check from options, x = 60. = 30 days 20. e 19. d It is given that 7 of the distance = 420 km or total 8 8 = 480 km. 7 It is given that he stopped for 9 min and when he increased his speed by 20 km/hr, he reached his destination by 6 min earlier. Hence, there is a difference of 9 + 6 = 15 min. Now take options into work. Taking option (d), i.e. 60 km/hr, distance = 420 × Average speed = = Total dis tance Total time taken 480 480 480 = = 7 hr + 9 min + 45 min 7 hr 54 min 7.9 = 60.75 km/hr. 420 60 + = 7 hr 45 min. 60 60 + 20 If he travelled the entire 480 km at the rate 60 km/ hr, then time taken = 8 hr. Since there is a difference of 15 min, (d) is the answer. time taken = Page: 118 MBA Test Prep Solution Book-2 Explanations: Fundamentals of Grammar Unit – 1 Chapter 1 6. who baked the winning pie modifies woman 7. when I was unable to answer modifies time 8. for whom you are looking modifies one 9. who are willing to serve others modifies those 10. to whom much is given modifies one Exercise 1 1. time - subject, may be - verb Exercise 4 2. mail - subject, was - verb 1. How the prisoner escaped = subject 3. letter- subject, has been - verb 2. that the robbery was an inside job = predicate nominative 4. men - subject, were - verb 3. how he could just disappear = direct object 5. tracks - subject, were – verb 4. that he had escaped = appositive 5. whoever finds the stolen diamonds = indirect object 6. where he might be = object of the preposition 7. That we were ready to go = subject 8. whoever wants to go = indirect object Exercise 2 1. compound verb 2. compound sentence 3. compound sentence 9. That you are losing ground = subject 4. compound subject 10. Whoever injured the handicapped woman = subject 5. compound verb Exercise 5 6. compound sentence 1. that we have had = adjective clause modifying the predicate nominative year 7. compound object of the preposition 2. 8. compound sentence until we received word of his rescue = adverb clause modifying the verb waited 9. compound verb 3. whom I saw on Mount Kilimanjaroo = adjective clause modifying the subject hiker 10. compound sentence 4. that he will win the lottery = noun clause used as the direct object 5. Who lost this wallet = noun clause used as the subject Exercise 3 1. who listens to his men modifies leader 2. which I loved dearly modifies dog 3. who takes responsibility well modifies person 4. who purchased tickets modifies individuals 5. that you bought for me modifies shirt Exercise 6 Explanations: Fundamentals of Grammar 1. If the manager is unable to help = adverb clause modifying the verb try 2. whom you should write the letter = noun clause used as the object of the preposition MBA Test Prep Page: 119 3. whose neck was broken = adjective clause modifying the subject man 5. She said that she did not know where her shoes were. 4. that the ozone levels were dangerous = noun clause used as the direct object 6. She asked, “Why are you studying English?” 7. She said, “May I open a new browser?” 5. when the governor changed his mind = adverb clause modifying the verb objected 8. She said she had to have a computer to teach English online. 6. that Sarika will not return = adverb clause modifying the predicate adjective unfortunate 7. Why you don’t do your work = noun clause used as the subject. 8. where your Aunt is buried = noun clause used as the predicate nominative. 9. that the island is under water = noun clause used as the appositive. 10. whoever told the truth = noun clause used as the indirect object. Unit – 2 Chapter 5 Test 1 1. She wants an orange from that tree. (there can be many oranges on the tree and the noun is starting with a vowel sound). 2. The edifice on the corner is huge. (some specific building, which is huge). 3. The Chinese that Mrs. Sarla speaks is very easy to learn. (that specific Chinese, which Mrs Sarla speaks). 4. I borrowed a pencil from your sister’s pile of pens and pencils. (She borrowed one of the many pencils that her sister has). 5. One of the officers said, “ The chairman is late today.” (The officers are talking about someone specific, their chairman) 6 Ankur likes to play volleyball. (No article is required here.) 7. I bought an umbrella recently. (the modified noun is beginning with a vowel; u) 8. Kirti is learning to play the violin at her school. (Definite article comes before a) Please give me the book that is on the counter. (The speaker is asking for a particular book, which is lying on the table.) Chapter 2 Exercise 1 1. Mistakes are made by us. 2. Extensive research is done to determine by which gene autism is caused in the body. 3. People all over the world speak English. 4. The dog bit Latika. 5. Characters are revealed by manners. 6. Why did Sheena say such a thing? 7. He was laughed at by everyone. 8. I will be obliged to go by the circumstances. 9. Chapter 3 Exercise 1 10. We lived on Pali Street when I first came to your town. (No article is required before names of cities, states , countries or streets) Julia Roberts asked him about his plans for that day. 11. Delhi is the capital of India. (No article is required before names of cities, states , countries or streets) 3. Richa said that all of us would go there in the morning tomorrow. 12. My colleague’s family speaks Polish. (No article is required before names of languages.) 4. She said, “How long have you worked here?” 1. Vandana says that her husband loves her a lot. 2. Page: 120 MBA Test Prep Solution Book-2 13. The apples in my basket are red. (Those particular apples, which are in my basket) 14. Our friends have a cat and a dog. (our friend has both, one dog and one cat) 15. Go to a university near your house. (The noun is starting with a vowel but is pronounced with a ‘u’ sound, therefore, a instead of an is used.) 16. a lot 17. several of 18. a lot of 19. some 20. only, a few 21. a majority of, enough 22. many 23. much of Test 3 1. d A is incorrect use an 'a' before store, B is incorrect use 'the' before commodities, E is incorrect, use 'an' before enormous. 2. c Options A, C & D are incorrect, all need the article 'a' before 'far' in A, 'spirit' in C and 'watch tower' in D. 3. d B is incorrect, needs 'the' before hero, E is incorrect, needs 'A' before crush. 4. b Option A is incorrect because it should be 'an' epigram not 'the' as it embodies the generic, C is incorrect since it uses an before earliest which should be 'the' instead. 5. d A is incorrect, use 'the' before 'West', C is incorrect, use 'the' before the word 'roots'. 6. a Option B is incorrect, use 'a ' instead of 'the', E is incorrect, use 'The' instead of 'A' to describe the definite. 7. b Ans. b: A is incorrect as the article 'a' should be used before certain, C is incorrect 'the' should be used to emphasize 'nutritional'. Test 2 1. a In 'A' it should be 'a tedious loser' and in 'C' it should be 'a similar level…'. 8. d A is incorrect; use 'the' before Olympics, B is incorrect, use the article 'the' before 'big contract'. 2. c In 'E' it should be 'in …the sunshine". 9. b 3. c In 'B' 'enough' is not required. The correct expression is 'large enough' not 'larger enough'. In 'D' 'various' is inappropriate. It should be replaced by 'several'. A is incorrect, use definite article 'the' before 'dangers', D is ambiguous, needs a determiner before 'people' use 'some' to make the sentence correct. 4. c 'A' should have 'a friend'. 'C' should have 'gardening is a joy....'and 'D' should have 'pick a better.." 5. a In 'B' it should be 'ever more....' and not 'even..'. In 'C' it should be 'many' not 'more'. 6. e 'E' should have '..the fleece'. 7. a In A it should be 'a prudent board..' because it can be any board. 8. e In E it should b 'a mistake'. 10. e C is incorrect the definite article (the) should precede 'two' and E is incorrect as 'the' should precede the word 'pleasure'. Unit – 3 Chapter 6 Exercise 1 1. petrol - mass; class - collective 2. group - collective; bus - count 3. orchestra - collective; arena and evening - count 4. water and oil - mass; beach - count 9. c Money is quantified in terms of ‘little’. 10. d ‘People’ are numbered. ‘Few’ is used for numbers. Explanations: Fundamentals of Grammar MBA Test Prep Page: 121 Exercise 2 7. a It should be ‘Sumeet, Rekha and I’ because in such cases the sequence should be third person, second person and finally first person. 8. e In A the right word is ‘baggage’; in C the right word is ‘equipment’ and in E the right word is ‘data’. 9. b In B the correct word is ‘strength’ and not ‘strengths’, and in E it will not be ‘shore’ but ‘shores’. 1. Objective case 2. Objective case 3. Objective case 4. Subjective case 5. Subjective case 10. e ‘Suspicion’ will not take ‘s’. 6. Objective case Test 2 7. Possessive case 1. c 8. Possessive case 9. Objective case 10. Possessive case C is the correct option. In A it should be ‘Soviet dictator’ (singular). In B the correct word should be ‘they’ since it refers to ‘people’ and ‘people’ is plural. In D ‘often’ should come after ‘Stalin’ and in E it should not be ‘Soviet’s’; rather it should be ‘Soviet’ because it is telling about the nationality of the concerned person and thus qualifying the noun. 2. c C is the correct option because ‘our house’ is being compared to ‘that of’ our neighbours. B & D suggest that ‘our houses or house’ is bigger than ‘our neighbours’. 3. a B is incorrect because the ‘apostrophe’ should come with the ‘novelist’ and not ‘Charles Dickens’. In case of two nouns coming together the apostrophe is always placed with the second noun. D is incorrect because ‘respect’ is an abstract noun and it can never be in plural. In E ‘emotions’ will be followed by ‘has’ and not ‘have’ because it is referring to the ‘portrayal’ and not to ‘emotions’. 4. e In A ‘boomers’ should have an apostrophe. In B ‘civil right’ is a wrong expression because it’s always ‘rights’ in the given context. In C ‘credits’ is incorrect because here ‘credit’ means ‘trustworthiness; credibility’ and it cannot be used as plural. In D ‘too much’ is not appropriate in the given context because it changes the positive tone of the sentence into a negative one. 5. b In ‘A’ ‘coverages’ is incorrect. The right word is ‘coverage’. Certain words remain same in both singular and plural states. For example, ‘aircraft’, information’, ‘sheep’ etc. ‘C’ has improper structure. In D it should be ‘coverage makes..’ because ‘coverage’ is a collective noun here. In E it should be ‘days’ since the context is referring to a general kind of atmosphere for which the author yearns therefore ‘days’ will be more appropriate. Exercise 3 1. People and wall are concrete nouns. Home run is a compound noun. 2. Son is a concrete noun while post office and Udaipur Lake city are compound nouns. 3. Mumbai Island is compound; success is abstract; Singapore is concrete. 4. Respect, honesty and policy are abstract nouns. Compound nouns can also be concrete or abstract. Test 1 1. e All the parts of the sentence are grammatically correct. 2. d In D it should be ‘another’ and not other since it is talking about one more. 3. c In C ‘barren century’ should be preceded by ‘some’ or ‘a’. 4. b In B the right way of writing will be ‘each called a scissor’. 5. a In A the correct way of writing is ‘information does not..’ since ‘information’ is always treated as a singular. 6. c In A the correct word is ‘damage’ and not damages and in E the right word is ‘illness’ since it is ‘a chronic’. Page: 122 MBA Test Prep Solution Book-2 6. c A is incorrect because ‘hairs’ is incorrect. B is incorrect because it says ‘not the…’. A process can be any process therefore it will be ‘an ..extremely…’. D is incorrect because ‘growing’ will not take ‘of’ here. E is incorrect because ‘are’ and ‘processes’ are incorrect out here, since the noun (gerund) referred to, in this is ‘growing’. 7. a ‘Tourists’ and not tourism should be used. 8. c The climate of Delhi can be compared to the climate of Jaipur and not to the city of Jaipur. 9. a ‘alike’ means resembling each other. Similar would mean not exactly the same. Chapter 7 Exercise 1 1. She, him, his 2. I, you, your 3. He, himself, our 4. It, me, you 5. They, her, me Exercise 2 10. c When the sentence is positive, the tag has to be negative. 1. ‘He’ is the antecedent for ‘his’. Test 3 2. ‘Madonna’ is the antecedent for ‘her’. 1. c 3. ‘Rabbit’ is the antecedent for ‘its’. 4. ‘They’ is the antecedent for themselves, and you is the antecedent for ‘your’. 5. ‘Teacher’ is the antecedent for she, and us is the antecedent for ‘our’. In A ‘politician’ should be in plural because the sentence talks about ‘other’ which is an undefined number. In case it were a singular then it should have been ‘..the another’. In B ‘a time’ is incorrect; it should have been ‘some..’ 2. c In C ‘colonial’ is an incorrect word to be used because it is an adjective and not a noun. 3. d In A instead of ‘enough’ it should be ‘some’. In D an article ‘the’ should precede ‘Atlantic’. Exercise 3 1. his In A it should be ‘a degree of showmanship’ because degree has to be somewhat quantified and showmanship does not require a ‘the’ before it. 2. yours 3. theirs In A it should be ‘a rebuke’. In D it should be ‘indiscretion’ and not ‘indiscrete’, because the previous word is a noun and the latter is an adjective. 4. his, hers In A it should be ‘surge’ and not ‘surging’ since the context requires a noun. In E logging will not take an ‘s’. 1. myself 2. herself 7. c In C prediction should be in plural form and in D it will be ‘effects’ and not ‘effect’. 3. themselves 4. himself 8. b In B ‘experience’ is the right word and not ‘experiences’. 9. e In E, it will be ‘imminence’ and not ‘imminent..’ since in the context the emphasis is on the very fact of ‘imminence’ which is a noun. If we put ‘imminent’ the meaning will be entirely out of context. 4. a 5. d 6. a 10. e In D it will be ‘tank bombardment’; since ‘bombarding’ is a verb. Explanations: Fundamentals of Grammar Exercise 4 Exercise 5 1. ourselves 2. himself 3. herself 4. themselves MBA Test Prep Page: 123 Exercise 6 4. what - interrogative, this - demonstrative, me - personal I - personal, that - relative, you - personal 1. ‘Which’ is the relative pronoun. 2. ‘That’ is the relative pronoun. 5. 3. ‘Who’ is the relative pronoun. Exercise 11 4. ‘Whom’ is the relative pronoun. 1. 5. ‘Whose’ is the relative pronoun. he - personal, himself - personal, intensive, my personal, possessive. He is the antecedent for himself. (something is a noun) Exercise 7 2. Which - interrogative, this - demonstrative 1. that 3. These - demonstrative, mine - personal, possessive, Whose - interrogative, these - demonstrative 2. those 4. 3. this this - demonstrative, that - relative, I - personal, you personal 4. these 5. everyone, some, many, no one, none - all are indefinite 6. he - personal, himself - personal, reflexive, his personal. He is the antecedent for himself and his. 7. I - personal, myself - personal, intensive, him personal, himself - personal, reflexive, everybody indefinite. I is the antecedent for myself, and him is the antecedent for himself. 8. neither - indefinite, them - personal, anyone - indefinite, who - relative, us - personal 9. who - interrogative, that - relative, that - demonstrative Exercise 8 1. everybody, someone 2. both, everything 3. anyone 4. no one, others 5. somebody, one, neither Exercise 9 1. what 2. who 3. which 4. whose 5. whom Test 1 1. Here the subjective case of pronoun will be used. 2. ‘She’ is also a subject in this case along with ‘Payal’. 3. In this sentence ‘They’ is the subject and the action is directed towards ‘him’ and ‘me’ therefore these two words are in objective cases. 4. The ‘call’ is directed towards ‘him and her’ therefore these two are the objects of the call in the sentence thus in objective case. 5. Same explanation as in ‘4’. 6. Here both ‘I’ and ‘he’ are the subjective cases of pronouns. 7. ‘Rohit’ and ‘she’ both are subjects in this sentence. 8. In this sentence ‘we’ will be used because the action of ‘decide’ is being done by them in the sentence; thus the subjective case. Exercise 10 1. 2. 3. whom - interrogative, you - personal, that - demonstrative neither - indefinite, my - personal, me - personal you - personal, someone - indefinite, who - relative, others - indefinite Page: 124 MBA Test Prep Solution Book-2 9. Here ‘I’ will be used because ‘you’ is the subject and the condition referred to is ‘if you were ..’. 10. ‘One’ is the pronoun antecedent therefore it should be followed by ‘oneself’. 3. b A is incorrect, use Everybody / Everyone instead of ‘Everyman’, B is incorrect use ‘who’ instead of ‘whom’ – whom is used as the object of a verb or preposition. D is incorrect- use ‘they’ instead of them. 4. e B is incorrect, use ‘what’ instead of which, ‘what’ here is used relatively to indicate that which; D is incorrect, use ‘This’ instead of the plural ‘these’, ‘this’ is used to indicate a person, thing, idea, state, event. 5. b B is incorrect, should be the possessive pronoun ‘its’, C is incorrect , the pronoun should agree in number with the noun that is the object of the preposition, use ‘is’ instead of ‘are’. 6. e D is incorrect, use singular pronoun instead of ‘their’, E is incorrect use ‘your’ instead of the possessive yours. 7. c E is incorrect, should be ‘her speech’- with compound antecedents joined by or, nor, either...or, neither...nor, use pronouns that agree with the nearest antecedent. 8. a A is incorrect- When a pronoun is used along with a noun, choose the pronoun case that matches the noun’s function, should be ‘us’, B ‘They’- When a pronoun is part of a compound element, choose the pronoun case that would be correct if the pronoun were not part of a compound element, C- Should be ‘he’ instead of himself - when a personal pronoun is used in a comparison, choose the correct pronoun case by carrying the sentence out to its logical conclusion. 9. e B is incorrect uses the contraction of it is should be the possessive ‘its’, D is incorrect should use ‘which’ instead of ‘who’. Test 2 1. 2. ‘whom‘ is the correct pronoun. In this sentence a relative pronoun that is in the objective case because the woman is the object of the action and ‘we’ is the subject. Similarly in the second sentence the subject is ‘you’ and the object has to be ‘whom’ and not ‘who’ because the latter is a subjective case of pronoun. 3. Same explanation as ‘2’. 4. ‘Whomever’ is the objective case. 5. Same explanation as ‘4’ 6. Same explanation as ‘4’. 7. Same explanation as ‘4’. 8. Politicians are the subjects out here, therefore the relative pronoun should be in the objective case. 9. In this sentence the action is ‘write’, therefore the doer has to be in the subjective case. And in this case ‘whoever’ is the doer. 10. Similar explanation as ‘9’. 11. In this sentence a particular and definite set of book is being referred to, therefore ‘that’ has been used. 12. The sentence is referring to one of the books, therefore ‘which’ is being used. 13. Similar explanation. 14. Here the reference is to a definite law therefore ‘that’. Chapter 8 15. The law referred to is one of the laws thus ‘which’. Exercise 1 10. e B is incorrect use the reflexive form ‘herself’ instead of her, C is incorrect use ‘they’ instead of which. Unit – 4 Test 3 1. are going, 1. e C is incorrect, uses the pronoun ‘myself’ , ‘me’ is the correct pronoun to be used here, E is incorrect use ‘I’ instead of ‘me’. 2. have been resting 3. must be B is incorrect; use singular verb with ‘each one’ should be works, E is incorrect there should be a pronoun before ‘young ones’, use ‘their’ to make it correct. 4. will be finished. 2. e Explanations: Fundamentals of Grammar MBA Test Prep Page: 125 Exercise 2 Exercise 6 1. has 1. have been driving 2. do 2. was parked 3. was 3. can be. 4. are being Exercise 7 ‘Do’ is action verb, and was, has and are being are state of being verbs. 1. have helped 2. will be 3. had choked. Exercise 3 1. plays - action 2. will return - action 3. is - linking or state of being Exercise 8 4. have been - state of being 1. is reading 5. should have been playing - action 2. can be altered 6. go - action 3. is being planned Exercise 4 4. should be 1. Can understand 5. ‘ve (have) run. 2. must have told 3. shall go 4. was howling The first and third sentences are action verbs, and the second sentence a state of being verb. There are twenty-three helping verbs viz. is, am, are, was, were, be, being, been, have, has, had, do, does, did, shall, will, should, would, may, might, must, can, and could. Exercise 9 The use of helping verbs causes certain changes in verb phrases. One change is the use of contractions. 1. will be finished Exercise 5 2. should have been worked 1. ‘ve (have) done 3. would give. Not and n’t are not part of the verb phrase. 2. are going, 3. ‘s (is) staying. Have and is are in contracted form. Are is connected to the contracted form of not. All three-verb phrases are action verbs. Verb phrases can have one, two or three helping verbs in them Verb phrases with two or more helping verbs always keep a definite order. Most helping verbs can combine with other helping verbs but will not combine with all of them. e.g., is being said, has been said, will be said, could have been said, may have said, had been said. We can change the form of a verb: A verb can have an s added to it as in eat, eats or run, runs. Other changes could be eating, ate, or eaten for the verb eat. Run could be changed to running, or ran. Some Irregular verbs have several confusing changes. Page: 126 MBA Test Prep Solution Book-2 Exercise 10 Exercise 14 1. am coming 1. farming = predicate nominative 2. came 2. traveling = appositive 3. comes 3. writing = subject 4. send 4. saving = object of the preposition / traveling = direct object 5. was sent 5. gossiping = indirect object 6. am sending Exercise 15 Exercise 11 1. to skate = subject 1. can understand - action 2. to enjoy = direct object 2. is going - action 3. to win = predicate nominative 3. can be held - action 4. have seen - action 4. to marry = direct object 5. to kill = appositive Exercise 16 Exercise 12 1. to sit in judgment = subject 1. roared - intransitive complete (no receiver of the action) 2. to waste time in class = subject 3. to party/to sleep in = subjects 2. was - intransitive linking (captain is a predicate nominative) 4. to build a home/(to) raise a family = direct objects 5. to be home soon = direct object 6. to play cricket/to visit friends = predicate nominatives 7. to save money for a rainy day = subject 8. to go to college/to study management = predicate nominatives Exercise 13 9. to be rewarded for one’s efforts = direct object 1. failing 10. to live happily = object of the preposition 2. bordering Exercise 17 3. to consult 1. broken modifying spoke 4. searching 2. smiling modifying face 5. to match 3. frightened modifying child 4. growling modifying dog 5. squeaking modifying wheel 3. will be needed - transitive passive (be is the helping verb and dress receives the action) 4. did forget - transitive active (title receives the action and is the direct object) 5. has - transitive active (camera receives the action and is the direct object) Explanations: Fundamentals of Grammar MBA Test Prep Page: 127 Exercise 18 1. are = verb, you = subject, important = predicate adjective modifying you, too = adverb modifying important, to help the poor = adverb infinitive phrase modifying important, poor = direct object to the verbal to help, the = adjective modifying poor 2. had upset = verb, child = subject, the = adjective modifying child, crying = participle modifying child, everyone = direct object, in the room = prepositional phrase modifying everyone, in = preposition, room = object of the preposition, the = adjective modifying room 3. 4. 5. 6. 7. 8. 9. jumped = verb, he = subject, across the gap = prepositional phrase modifying jumped, across = preposition, gap = object of the preposition, the = adjective modifying gap, without knowing the distance = prepositional phrase modifying jumped, without = preposition, knowing = gerund used as the object of the preposition, distance = direct object of the verbal knowing, the = adjective modifying distance is = verb, exercising = gerund used as the subject, good = predicate adjective modifying exercising, for everyone = prepositional phrase modifying good, for = preposition, everyone = object of the preposition loves = verb, Jasmine = subject, to dance constantly = noun infinitive phrase used as a direct object, constantly = adverb modifying to dance is = verb, teasing by your friends = gerund phrase used as the subject, by your friends = prepositional phrase modifying teasing, by = preposition, friends = object of the preposition, your = adjective modifying friends, hard = predicate adjective modifying teasing, to take = adverb infinitive modifying hard fled = verb, people = subject, the = adjective modifying people, fearing reprisal = participial phrase modifying people, reprisal = direct object to the verbal fearing, from the city = prepositional phrase modifying fled, from = preposition, city = object of the preposition, the = adjective modifying city is = verb, eating out = gerund phrase used as a subject, out = adverb modifying the verbal eating, thing = predicate nominative, the = adjective modifying thing, to do tonight = adjective infinitive phrase modifying thing, tonight = adverb modifying to do do know = verb, I = subject, n’t = adverb modifying do know, whether/or = correlative conjunction, to tell him/to keep quiet = noun infinitive phrases used as direct objects, him = direct object to the verbal to tell, quiet = adverb modifying to keep Page: 128 10. should be done = verb, job = subject, our/next = adjectives modifying job, to run to the shop = noun infinitive phrase used as an appositive, to the shop = prepositional phrase modifying to run, to = preposition, shop = object of the preposition, the = adjective modifying shop, quickly = adverb modifying should be done. Test 1 1. Both “to spend” and “spending” could be correct 2. to have gone 3. Both “to call” and “calling” are correct. 4. Answer E. (the participial phrases in A and B correctly modify Rahul.) 5. The overloaded car gathered speed slowly. 6. The opening participial phrase is misplaced because it is intended to modify him, not the proposal. A possible revision would be: Espousing a conservative point of view, he was bothered by the proposal for more spending on federal social programs. 7. When I return to Mumbai next year, I will be very happy. 8. Rahul goes to school every day. 9. Sapna is visiting her family right now. 10. I studied/was studying Economics in 1994. Test 2 1. He has spoken/has been speaking French since he was a child. 2. Raj had visited many places before he came here. 3. We saw terrible things back then. 4. Sometimes I still have dreams like I did twenty years ago 5. Japan had never had democracy until 1945 6. The father will call the family together if he thinks there is disharmony. 7. When I was young, I never cooked because my parents had two servants. 8. d ‘hearing’ is not used in present continuous. It is ‘hear’ even in continuous tense — ‘I hear her’. MBA Test Prep Solution Book-2 9. b The verb with amounts of money and time periods is singular. Therefore, ‘twenty thousand’ is required. 5. a I go. Simple present is correct here rather than continuous tense to indicate habit. 6. e ‘ate … left’ the parallelism in the sentence is correct. 7. a Present perfect continuous is correct here. Test 3 8. e The sentence in indirect speech is correct. 1. a ‘a’ is the right option. In ‘A’ it should be ‘off’ not ‘of’. In ‘B’ the correct word will be ‘seemed’ not ‘seemingly’. And in ‘D’ it should be ‘at’ not ‘in’. 9. a ‘with aplomb’ is correct. ‘e’ is the correct option. B and C are correct because in both cases ‘its’ does not refer to Alan Davey, but to the company that is not mentioned. 11. a ‘Reach’ doesn’t take a preposition. 10. e ‘ordered for three cups of tea’ is correct. 2. e 3. b ‘b’ is the right option. In ‘A’ it should ‘was’ and not ‘were’; in ‘B’ it should ‘than’ and not ‘then’; and in ‘D’ it should be ‘conservatives’’ and not ‘conservative’s’. 4. a In ‘B’ the correct verb should be began because ‘..as I nosed..’ happens simultaneously with the rest, therefore past perfect is not appropriate in this case. 5. d It should be ‘changing ..’ 6. e In ‘C’ it should be ‘attracts us..’ and in ‘E’ it should be ‘combine’ because it talks about a habitual action. 7. c In C it should be ‘you’ll bump..’; ‘have bump’ is incorrect. 8. a In ‘A’ it should be ‘helped’ because the sentence is referring to past action. 9. a 10. c (c) is the correct way of asking a question. Test 2 1. a In A the correct helping verb is ‘is’ and not ‘was’, because the whole sentence is in present tense. In E instead of ‘gives’ it should be ‘given’ because the act is in past tense. 2. b In B it should be ‘...will be’ and not ‘...have to be…’ because the sentence is indicating towards the future action, besides ‘have’ is used for a plural subject. 3. d In A it should be ‘rears’ since the sentence is talking about a routine action. 4. e In A, it should be only ‘happened’ and not ‘will have happened’; since the statement is referring to action that has already taken place. 5. c In A it should be ‘I have’ because ‘will be followed’ is an incorrect expression. In D it should be ‘gathered’ and not ‘gathers’, since the statement talks about past action. 6. a In A it should be ‘to encompass’. In E it should be ‘dampen’ and not ‘dampens’. 7. e In E ‘does’ is the incorrect word. It should be ‘do’. 8. d In ‘B’ it should be ‘..have been’, since the sentence is in present tense. 9. b In B the correct tense will be present, so it should be ‘They work…’. In ‘A’ it should be ‘of making’ and in ‘C’ ‘being’ is not required. 10. e In ‘A’ it should be ‘did..’ and not ‘do’ because the statement is in past tense. In ‘C’ it should be ‘like..’ and not ‘alike’. Chapter 9 Test 1 1. a In (a), instead of ‘using’ it should be ‘use’. 2. d Rather than ‘precipitate’ it should be ‘precipitating’ because the action talked about is in present continuous. 3. a Instead of ‘giving’, it should be ‘given’ because ‘the efforts’ have already been made therefore past tense should be used. 10. a In A, since ‘may’ is already there, ‘will’ is not required; and since the idea is related to probability, ‘will’ is too strong a word to be used here. 4 . d ‘Being’ is not required here. Explanations: Fundamentals of Grammar MBA Test Prep Page: 129 Test 3 9. Here is (change to are)the three books you wanted. 1. b Option B is the correct answer because ‘by’ should precede ‘involving’ to give a proper meaning to the sentence. 10. Five hundred rupees is/are all I am asking. 'Has been' is the correct expression here since the context suggests that the work has got over first now. Thus present perfect should be used here. 1 d. 2. d 3. e Option E is correct in all respects of the verb usage, ‘becomes’ is the correct usage. 4. c correct usage of the verb is ‘is’. 5. b Correct usage is ‘ motivated’ which means to provide with a motive. In ‘C’ the meaning of the sentence is entirely changed. 6. c Option C is correct with respect to the form of the verb as well the subject, since the sentence is in past tense. Test 2 2. The majority of the Parliament is/are Congressmen. (The majority of the Parliament… what follows the of is singular. Hence singular verb) 3. c (‘who’ refers to what is immediately before it ‘students’, hence ‘are’) 4. d 5. The original document, as well as subsequent copies, was/were lost. 6. c (what follows the ‘of’ is plural) ‘... are going to the polls.” 7. Almost all of the magazine is/are devoted to advertisements. 7. d Option D is correct because once the results come out then only analyzing discoveries can the made thus the use of past tense. 8. Here is/are Manish and Mandar. 8. e Only option is E is correct in its usage of the verbs ‘are’ and ‘explore’. 9. Taxes on interest is/are still deferrable. Correct usage is ‘have’. 10. b (five rupees is singular, hence is) 9. a 10. d Option D is the correct answer becaue have should precede ‘held and hold’ which is a phrase here. Chapter 10 Test 1 Test 3 1. d In D it should be ‘have’ since it is referring to both graffiti and its street art cousins. 2. b The correct verb is ‘come’ because the subject is ‘irreverence’ which is singular. 3. b In B ‘carries’ is incorrect. It will be ‘carry’ since the subject i.e. is in plural. 1. In the newspaper, an interesting article appeared. 2. Across the road lived her boyfriend. 3. Around every cloud is a silver lining. 4. d It says ‘Mr. Lott, along with Speaker …’, therefore in D it should be ‘was ..’ because the subject is ‘Mr. Lott’. 4. Neither he nor his brother are (change to is) capable of such a crime. 5. e In E it should be ‘that are ...’, since the subject referred to is ‘techniques’. The teacher or student is going to appear on stage first. (No change required) 6. b It should be ‘there is ...’ since it is referring to ‘good as well’. The mother duck, along with all her ducklings, swim (change to swims) so gracefully. 7. c In C it should be ‘a hacker could get into…’. 7. Each of those games is exciting. (No change required). 8. c In C the correct helping verb is ‘has’ and not ‘have’ since the subject referred to is ‘No one’. 8. The file, not the documents, were (change to was) misplaced. 5. 6. Page: 130 MBA Test Prep Solution Book-2 9. e In E it should be ‘is’ since the referred subject is ‘each’. 10. d In D it should be ‘nail scissors were..’ and in E it should be ‘channel was..’ because here the subject is ‘breach’; ‘some’ here has a different contextual meaning. Unit – 5 Exercise 5 1. Either and the first any are pronouns, this and the second any are adjectives. 2. The first each and their are adjectives, and the second each is a pronoun. 3. This and no one are pronouns. 4. Both and one are pronouns, and many and neither are adjectives. 5. What and neither are pronouns, and your is an adjective. Chapter 11 Exercise 1 1. Heavy, red, fifty. 2. My, two, my, graduation Exercise 6 3. That, small, Indian, the, next, fresh 1. 4. Little, black, the, well-dressed 2. Happy, three, frolicking, their, his 5. Old, wood, several, discarded, packets 3. Exciting, most 4. Flooded, terrible 5. Kaushik, hot, exhausting Exercise 2 1. red 2. Those, brown 3. Two, what, many, their 4. Third, city, another 5. That, good, this Exercise 3 Our, first, many, strong Exercise 7 1. jolly, jollier, jolliest 2. honest, more honest, most honest 3. dim, dimmer, dimmest 4. friendly, friendlier, friendliest 5. little, less or lesser or littler, least or littlest (Little when referring to amount uses less, lesser and least; when referring to size uses littler and littlest.) 1. New 2. Stormy, spring, flash 3. New, UK Exercise 8 4. December, dangerous Himachal 1. interesting, more interesting, most interesting 2. critical, more critical, most critical 3. splendid, more splendid, most splendid 4. delicious, more delicious, most delicious 5. outstanding, more outstanding, most outstanding Exercise 4 1. Soaking 2. Broken, crying 3. Great, giving 4. Laughing 5. Eager, torn Explanations: Fundamentals of Grammar MBA Test Prep Page: 131 Exercise 9 Test 1 1. many, more, most 1. a 2. ill, worse, worst 3. much, more, most 4. perfect - cannot be compared since there is no more perfect or most perfect. In B ‘dried’ is not appropriate, since it cannot be used as a predicative adjective here and it does not match ‘clean’ and ‘safe’. Similarly C is incorrect because ‘cleaned’ is not appropriate. In D ‘and’ is not required after ‘dry’ because when using adjectives in a string we can separate them by commas. Therefore correct way of writing will be ‘dry, clean and safe’. In E ‘effected’ is not the right adjective to be used. 5. bad, worse, worst 2. e In A ‘relenting’ is not the right adjective because it changes the meaning of the sentence. In B ‘relentlessly’ is incorrect because it is an adverb and not an adjective and contextually we require an adjective here to qualify the noun ‘increase’. In C ‘sophisticated’ should come before ‘systems’ since it is qualifying ‘systems’ and is used as an attributive adjective. In D ‘sophistication’ is a noun and not an adjective therefore incorrect. 3. c In A instead of ‘Kings eye’ it should be King’s eye’ because here possessive case is required. In B the apostrophe is missing from ‘family’s’. In D instead of ‘family’s’, ‘families’ is given. In E ‘king eye’ is incorrect. 4. a In B ‘pushy’ should be preceded by ‘and’ because only two adjectives have been used here. In C ‘which seems’ should be followed by ‘surprising’ which is an adjective and it qualifies the unwritten noun in the sentence that is ‘love’; whereas ‘surprisingly’ is an adverb which cannot qualify a noun. In D ‘velvety’ is not an appropriate word because ‘brocade’ is a noun here thus requires a noun ‘velvet’ as the other word in the pair, whereas ‘velvety’ is an adjective; though ‘Velvet and Brocade’ has been used here as a ‘Noun adjunct’ (refer to Practice Exercise 2). In E ‘velvetand-brocade’ should precede ‘backdrop’ rather than following it since the former is qualifying the latter. 5. c In A, ‘original’ is an appropriate word because the context requires an adjective. In B ‘in probing his opponent’s defences’ should follow ‘duke’s skill lay..’. In D it should be ‘persuasively’ and not ‘persuasive’ because the former qualifies the verb ‘argues’ thus it is an adverb whereas the latter is an adjective and cannot qualify a verb.. In E, ‘when a weakness opened up..’ should follow ‘quickly and committing troops’. 6. a In B ‘enjoyed fervently the support ..’ does not make any sense; rather it should be ‘the fervent support’ because ‘fervently’ is an adverb and ‘fervent’ is an adjective. In C ‘of Americans of all colours’ should come after ‘’fervent support’ or else it will not be clear as to preposition ‘of’ is referring to what. In D ‘billed as a contest’ is referring to ‘a fight’ therefore it should follow ‘a fight’. In E, ‘similar’ is not an appropriate Exercise 10 1. worst 2. hungrier 3. shortest 4. best 5. happiest Exercise 11 1. a wife and doctor 2. a girl and a boy 3. a green and red 4. a rock star or a lawyer 5. a bat and a ball Exercise 12 1. this or that 2. this or that 3. these or those 4. these or those 5. these or those Page: 132 MBA Test Prep Solution Book-2 adjective. Instead ‘same’ should be used. If at all ‘similar’ has to be used it should be preceded by ‘a’ because it can be any. 7. d In A rather than ‘irrelevantly’ it should be irrelevant, since the former is an adverb and an adverb cannot qualify a noun. In B ‘irrelevance’ is used incorrectly because it is a noun and the context requires an adjective. In C ‘publicly’ is incorrect. In E ‘nuisance’ should follow ‘irrelevant’. 8. a In B a comparative adjective is not required, therefore ‘crisper’ is incorrect. In C ‘admiring’ is incorrect because an adverb (admirably) is required here which is qualifying the adjective ‘self- deprecating’. In D ‘both emotional and professional’ is placed incorrectly. It should come after ‘..life’. In E ‘self-deprecatingly’ is an adverb which is inappropriate since ‘vignettes’ is a noun which requires an adjective. 9. d In A ‘populist’ is an incorrect adjective since the statement talks about ‘sovereignty’. In B ‘with the ideal of popular sovereignty’ is placed incorrectly; it should come in the end. In C ‘habitual’ is incorrect since it is an adjective and the sentence requires a noun that is ‘habit’. Again in E ‘popularly’ is an adverb and the context requires an adjective not an adverb. 10. c A doesn’t require an adverb ‘extraordinarily’; rather it requires an adjective ‘extraordinary’. In B ‘demographic’ is placed incorrectly before ‘extraordinary’ but the ‘growth’ in this case pertains to ‘demography’. In D ‘in which settler pressure’ should precede ‘boosted by extraordinary demographic growth’, because the second phrase qualifies the ‘..settler pressure..’. In E ‘demography’s’ is incorrect because it is a noun and an adjective is required out here since ‘growth’ is a noun here. 5. c In ‘C’ ‘unsightly’ should precede ‘beggars’ since it is modifying ‘beggar’. 6. e In ‘A’, ‘exuberance’ is a noun whereas the context demands an adjective, therefore it should ‘exuberant’. In ‘E’ rather than ‘huge’ it should be ‘hugely’ because ‘welcome’ is an adjective out here qualifying ‘start’. ‘Huge’ is also an adjective therefore it cannot qualify another adjective whereas ‘hugely’ can since it is an adverb. 7. e In ‘A’ ‘particularly’ is an adverb and therefore inappropriate. It should be ‘particular’ which is an adjective. In ‘E’ ‘extensive network’ is meaningless. Here ‘network’ will be treated as a verb rather than a noun and ‘extensive’ will become an adjective by adding ‘-ly’ to it. Thus it will be ‘network extensively’. 8. b In ‘B’ ‘most absolute’ is incorrect because ‘absolute’ in itself is an absolute term and thus cannot have any superlative form. In ‘D’ ‘red facedly’ is an adverb therefore inappropriate out here. It should be ‘red faced’. 9. d In ‘D’ the appropriate adjective will be ‘irrational’. Since in ‘E’the statement is talking about ‘all the new sitcoms’ it should be ‘least favorite’. If it were a comparison between ‘two’ then it would have been ‘less’. 10. d In D it should be impetuousness since the context demands a noun and not an adjective. In E, it should be ‘an Elvis Presley wig’. An apostrophe with Presley will change the meaning. It will mean that the wig actually belongs to Elvis Presley whereas the context implies that the wig should be of Elvis Presley style. Test 3 1. a In ‘A’ it should be ‘cleverer’ since here a comparison is made between what the Robots were and what they are now. 2. c ‘no more longer’ is incorrect because longer will not take ‘more’. ‘A’ is incorrect because it’s probably not a good idea to use this construction with an adjective that is already a negative: “He is less unlucky than his brother,” although that is not the same thing as saying he is luckier than his brother. In ‘C’ when we are already using ‘harder’, ‘more’ is not required. 3. e In ‘E’ ‘tempting’ should precede ‘special’ since ‘special’ describes the offer, and ‘tempting’ is the nature of the ‘special offer’. 4. d Rather than the adjective ‘apparent’ an adverb apparently should by used. 3. a In ‘B’ ‘more ardent’ is incorrect because ‘ardent’ cannot have any comparatives or superlatives. 5. a In ‘A’ ‘harder’ is the correct form of adjective since the sentence refers to ‘this time’, which implies a ‘comparison’ is being made. 4. a In C, ‘party’s preference’ is incorrect, rather it should be ‘party preference’ otherwise the meaning will be entirely changed. In d, ‘running-down’ is an incorrect phrase. In should be ‘run- down’. 6. d In ‘D’ rather ‘solid linking’, it should be ‘solidly linked’. Test 2 1. e 2. d In ‘A’ it should be ‘rural poor have..’ and not ‘has’. Similarly in ‘C’ ‘the rich..are..’; since collective adjectives are taken as plural. Explanations: Fundamentals of Grammar MBA Test Prep Page: 133 7. d In ‘D’ using more with ‘humility’ and ‘grace’ is incorrect, since these two words do not require to be used with comparatives. Exercise 4 1. not, correctly (both words modify the verb did do) 8. c Rather than professionalised it should be professional. 2. 9. a It should be ‘most reliable’ and not ‘reliable most’. never, so, deeply (never and deeply modify the verb was pleased; so modifies deeply telling how much) 10. b In ‘B’ ‘freer’ is an incorrect; ‘free’ is a more appropriate adjective. 3. actually, almost (actually modifies the verb likes; almost modifies every telling how much) Chapter 12 4. recently, n’t (recently modifies the verb found; n’t modifies the verb would help) Exercise 1 5. not, tomorrow, very (not and tomorrow modify the verb will go; very modifies scary telling how much) 1. quickly (how) 2. too (how much), softly (how) Exercise 5 3. soon (when), yesterday (when) 1. n’t, very, quickly, foolishly 4. then (when), there (where) 2. suddenly, quietly 5. why (why), so (how much), often (when) 3. laughingly, happily, together Exercise 2 4. rapidly, accurately 1. Daily, very 5. today, tomorrow 2. totally Exercise 6 3. hourly, rapidly 1. wisely 4. suddenly 2. seldom 5. unusually, outside 3. often 4. now, again 5. now Exercise 3 1. completely modifies the adjective exhausted telling how much, quickly modifies the verb was pulled telling how, aboard modifies the verb was pulled telling where 2. once/twice modify the verb has called telling when 3. usually/slowly modify the verb work telling how, rather modifies the adverb slowly telling how much 4. 5. surely/n’t modify the verb were telling how, very modifies the predicate adjective expensive telling how much Exercise 7 1. n’t (when/how), often (when), here, (where), before, (when). They all modify the verb have stopped. 2. faithfully (how), carefully (how). They both modify the verb does. 3. sometimes (when), always (when), highly (how much). Sometimes modifies the verb say. Always modifies the verb have been. Highly modifies the adjective critical. 4. yesterday (when), by, (where), once (when), twice (when). They all modify the verb came. greedily modifies the verb had telling how, too modifies the adjective much telling how much Page: 134 MBA Test Prep Solution Book-2 5. there (where), very (how much), safely (how). There and safely modify the verb lay. Very modifies the adverb safely. 5. 6. today (when), rather (how much). Today modifies the verb seemed. Rather modifies the adjective restless. Exercise 12 Exercise 8 often modifies the verb go, too modifies the adverb far, and far modifies the verb go 1. sure 2. surely 3. surely 4. sure 5. surely 1. not, now 2. yesterday, very 3. never, there, before 4. too Exercise 13 5. always 1. well 2. surely Exercise 9 3. very 1. today, tomorrow, technically 4. good 2. n’t, alone 5. really 3. where, yesterday 6. badly 4. too, again 7. really 5. finally, away Test 1 Exercise 10 1. any 2. can 3. ever 4. have 5. any Exercise 11 1. extremely modifies the adjective tired, and very modifies the adjective sore 2. yesterday and hardly modify the verb had completed, very modifies the adjective hard, and rudely modifies the verb was interrupted 3. gradually modifies the verb reached, and before modifies the verb had climbed 4. just modifies the adverb now, now modifies the verb remembered, and rather modifies the adjective important Explanations: Fundamentals of Grammar 1. e Since none of the sentence/s or part/s thereof have/ has any error. 2. b In 'A' the adverb to be used is 'doubly'. It doesn't require an adjective since 'irrational' is an adjective and 'doubly' as an adverb will qualify it. On the contrary option C will have an adjective i.e. 'huge' and not 'hugely', which qualifies the noun 'business opportunity'. 3. e 'B' is correct since 'downright' can be used as both adjective and adverb. 4. c In 'D' 'evenly' is incorrect because it changes the meaning of the sentence. The correct adverb will be 'evenly'. 5. d In B 'far' is an adverb that is describing the adverb 'worse' and structurally it should come before 'worse' and not after it. In c since the context requires an adjective and both 'obsessive' and 'obsessed' are adjectives. But the right choice here will be 'obsessed' that means 'having an obsession', whereas 'obsessive' means 'excessive in degree'. Here with achievement 'obsessed' parents will be appropriate MBA Test Prep Page: 135 6. e 7. d In D instead of 'humanly' it should be 'human'. There is no word such as 'humanly'.In E 'nonetheless' should come after 'robots', because 'nonetheless' is an adverb and not an adjective. Usually an adjective precedes the noun. 11. b 'Discounting' is a verb and the context requires a noun adjunct. Therefore 'discount' is a better option. In D 'anyways' is not appropriate since it is not used in standard English usage. The correct adverb will be 'anyway'. Test 3 8. a In 'A' 'deep' is an adjective and changes the meaning of the sentence therefore it should be replaced by an adverb 'deeply' which qualifies the verb 'held' here. 9. b In a 'very unique' is incorrect because 'unique' in itself is a superlative adjective and 'very' will not make it more so. In c the context demands an adverb since it describes the verb 'explaining' therefore 'originally' should replace 'original'. 12. c 'although' is not a required because 'yet' has already been used in the sentence. 1. b 'a' is incorrect because 'yet' is incorrect in this sentence because it means 'for the present'; whereas 'still' means 'nevertheless'. Option c is also incorrect because the placing of 'nevertheless' is incorrect. Option d inappropriate because 'albeit' means 'notwithstanding' and though is not required, it doesn't make any sense in the sentence. In 'e' the use of 'yet' is incorrect. 2. a In 'a' 'such' which is an adjective,, means 'of the kind'. In 'b' 'as' with 'such' changes the meaning. In 'c' 'still' is incorrect because it doesn't make any sense here. Again in 'd' 'yet' is inappropriately placed after 'but'. In 'e' 'as such' again changes the meaning of the sentence. 3. e In 'a' instead of 'more old' it should be 'older'. In 'b' a comparative form is required therefore 'older' is required. In c 'reassured' is incorrect because the context requires an adverb. In d rather than 'reassuring' an adverb is required thus 'reassuringly' is appropriate. 4. c In 'a' 'enormous' is used which implies that the meaning of the sentence will convey that markets have become enormous, though the stress in the sentence is on the 'growth'. 'Enormously' is apt because it is an adverb, describing the growth. In b 'possibly' is an adverb and inappropriate in the given context. In d 'vigorously' an adverb is incorrectly used. Since the word is talking about the 'capital markets'- a noun, it can be qualified only by an adjective. In e 'enormously' should follow 'grown'. 5. b In 'a' instead of 'poor' it should be 'poorly' because it is describing the verb 'enforced'. In 'c' 'wretched' will be correct because wretchedly is an adverb and thus inappropriate since the context is dealing with 'China disclosures' which is a noun. 10. b In C 'fully' is incorrect because the context requires an adjective since it has to describe a noun (account). Therefore 'full' is the right word and it should come before account. Test 2 1. c Correct adverb will be 'parallel'. 2. a It should be 'bump along..' because using 'alongside' here will imply that the surface is also bumping along. 3. e The use of an adverb 'deeply' is incorrect out here. The context requires the comparative form of an adjective. 4. d The correct adjective in this part will be 'worse' which is the comparative form of 'bad' and in this sentence the comparison is made with 'useless'. 5. d It should be 'moving quickly' rather than 'moving quick'. 'Quickly' an adverb will modify 'moving' which is a verb. 6. e Rather than 'relative' which is an adjective, 'relatively' (an adverb) will be more appropriate 7. b In part (b), 'helpful' (an adjective) should be preceded by 'politically'(an adverb). 8. c 'Rather' itself means 'somewhat', therefore 'somewhat' is not required here. 9. a It should be 'rapidly' because 'rapid' is an adjective and cannot qualify a verb i.e. 'developing'. 10. e Rather than an adverb the context requires an adjective; therefore it should be 'official' and not 'officially'; since it is qualifying the noun 'neglect'. Page: 136 In d 'poor' is incorrect because it changes the meaning of the sentence and it is also placed incorrectly. In e 'enforcing' is incorrect because the 'laws' talked about have already been in past tense. 6. a In b 'awful' is an adjective and changes the meaning of the statement. In c 'awfully' should be preceded by 'an', because the former is an adverb which is qualifying an adjective 'long'. In d 'worried' changes the meaning. It appears that the 'signs' are 'worried'. In e 'awfully' is not placed properly. MBA Test Prep Solution Book-2 7. e In 'a' 'urbane' is inappropriate because though an adjective its meaning is 'having the polish and suavity' whereas the correct word should be 'urban'. For similar reasons 'b' is incorrect. In 'c' 'costing' is inappropriate. In 'd' 'costlier' should precede 'fuel'. 8. d In 'a' 'fast' is inappropriate because here Yuan is competing against dollar. The requirement is that of the comparative form of adjective. In 'b' 'instead of' is inappropriate. Because instead (an adverb) means 'as a substitute' whereas 'instead of' means 'in lieu of'. The context here demands 'instead'. In c 'annually' an adverb is incorrect since it cannot qualify a noun i.e. rate. Therefore the correct word is 'annual'. In e 'despite' is an incorrect adverb to be used. 8. b Only option (b) is correct in all respects, the subject is the type of Indian architecture in central India, an example of misplaced modifier. 9. a Only option (a) correctly modifies the subject in question, an example of misplaced modifier. 10. c The sentence is correct in all respects, the subject is the new experimental device, a case of misplaced modifier. Test 2 1. 'Smaller' doesn't make any sense in option b because there is no comparison being made. In c 'opposing' does not make any sense. 'Opposite' will give a meaning to the sentence. In d 'complete' is an adjective, whereas the context requires an adverb. Similarly in e 'utter' is incorrect. It should be 'utterly'. Looking through the telescope, we could see Venus clearly in the night sky. 2. Flying out the window, the papers were grabbed by him. OR He grabbed the papers as they flew out the window. 10. c In 'a' 'seldom' is incorrectly placed. It should be before 'dissuade us'. Similarly in 'b' 'seldom' is placed between 'dissuade' and 'us'. In d also seldom is incorrectly placed. And in e 'seldomly' i s an incorrect word. 3. Dhas arrived with the keys as I was waiting outside. OR While I was waiting outside with the keys, Dhas arrived. OR While I was waiting outside, Dhas arrived with the keys. (The ambiguity in the original is removed in all these sentences. First and third sentences are preferred.) 4. While walking on the grass he was bitten by a snake. 5. I tried calling half a dozen times to tell you about the Career Launcher Seminar. OR I called half a dozen times to tell you about the Career Launcher Seminar. (The ambiguity in ‘tried calling’ is eliminated in the second sentence. Choose this over the first sentence if both are given as options) 6. Dhas manged to finish the soup although it was extremely spicy. 7. While walking across the street, she was surrounded by them and was robbed of her purse. OR She was surrounded by the and was robbed of her purse while walking across the street. 8. In her lunch box, she has some cake (that) she baked. 9. I really/very glad to be of help to you. 10. The baby smells very sweet. 9. a Chapter 13 Test 1 1. b Only option (b) is correct. Option (b) clearly defines the reason for going on trial and develops the consequences in the correct order, a misplaced modifier. 2. a Option (a) conveys the meaning of the sentence completely, defines the efficacy of houses 'with' central air conditioning. 3. c Option (c) is correct. The sentence describes the 'mirror with the brass inlaid figures', only option (c) conveys the meaning correctly. 4. b Only (b) is the correct option. All other options make it seem that Federer has blue stripes, a case of misplaced modifier. 5. d Only (d) conveys the complete meaning of the sentence. Here the subject is the rider who was thrown while jumping the obstacle, and example of dangling modifier. 6. b Only option (b) is meaningful and conveys the complete meaning, an example of misplaced modifier. 7. e Only option (e) is correct, place the adjectives after the words they modify, the air is the subject which is being described here. Explanations: Fundamentals of Grammar MBA Test Prep Page: 137 Test 3 Exercise 3 1. No change required. 1. of the new book modifies “title”/ about morals modifies “book” 2. No change required. 2. 3. Life in the city is exciting, but life in the countryside is better. on the planning commission modifies “work”/ of ideas and concepts modifies “kinds” 3. 4. Drive more slowly as work is in progress. on the west side modifies “houses”/ of town modifies “side” 5. No change required. 4. in the next room modifies “man” 6. Speak a little more slowly or you will not be understood. 5. of the citizens modifies “few” 7. When he spoke at a press conference on Saturday night, the Home Minister acknowledged the role played by the men who subdued the gunman. Exercise 4 1. with sharp thorns modifies “tree”/ beside the wall modifies “grew” 8. To improve company morale, the consultant recommended three things. 2. above the people modifies “soared”/ on the field modifies “people” 9. In reviewing the company’s policy, the board identified three areas of improvement. 3. of the village modifies “owner”/ past the house modifies “rode” 10. Baked, boiled, or fried, potatoes make a welcome addition to almost any meal. 4. by its tracks modifies “followed”/ in the snow modifies either “tracks”(telling which tracks) or “followed” (telling where we followed it) 5. over the fence / into some bushes modify “tumbled” 6. of wreckage modifies “tons”/ after the tsunami modifies “were left” 7. over a hill / through a beautiful valley modify “wound” Unit – 6 Chapter 14 Exercise 1 1. on the wall, of the house 2. in the shade, of the apple tree, of the jobs, for the day 3. over the mound, behind the farm, into the street. 4. but you, from home, with parental permission 5. around the yard, for miles, except junk Exercise 5 1. from her desk modifies “took” telling where / of one modifies “edition” telling which / of the classics modifies “one” telling what kind 2. in the display case modifies “was placed” telling where / in the corner modifies “case” telling which / of the library modifies “corner” telling which 3. of mysteries and detective stories modifies “books” telling what kind / in the library modifies “are found” telling where 4. about magic modifies “story” telling what kind / in our literature book modifies “appears” telling where 5. to the solution modifies “clues” telling which / of the mystery modifies “solution” telling which Exercise 2 1. in, with 2. up, on, through 3. by, at 4. with, near 5. from Page: 138 MBA Test Prep Solution Book-2 6. by my uncle modifies “stories” telling which / about Sherlock Holmes modifies “stories” telling what kind 5. a ‘Naval gazing’ is the correct expression. It means indulge in self-absorbed pursuits. 7. of ancient Pompeii modifies “wall” telling which / by an ordinary peasant modifies “was discovered” telling how 6. b ‘Crystal-ball gazing’ is the correct expression. It involves ‘the use of a crystal ball, a seer stone or other crystal as a divine tool through which one seeks to receive visions or information about the future. 7. d ‘Sell himself’ is the right expression. Other phrasal verbs used in the options have different meanings. ‘Sell off’ means to get rid of by selling, often at reduced prices. ‘Sell out’ means to betray one’s cause or colleagues. ‘Sell down’ means to betray the true trust or faith of. I cannot put up with this sort of English 8. e ‘Cast a shadow’ is the correct idiom. 2. I don’t know where she will end up. 9. a 3. It’s the most curious book I have ever run across. ‘Lo and behold’ is the correct idiom which means look and behold; an expression of surprise and amazement. 4. No change required. 5. India became free on 15th Aug. 1947. 6. India is independent for more than 50 years. Test 3 7. India is free since 1947. 1. Cut the pizza into six pieces. 8. Where did you get this? 2. No change required. 3. Where did he go? 9. If we split it evenly among the three of us, no one will be unhappy. 4. Where did you get this? 10. You can’t just walk into the class without permission. 5. I will go later. 6. Cut it into small pieces. 7. We will arrive on the fourth of next month. 8. No change required. 9. No change required. 10. No change required. 11. Tanya entered the room. 12. She dived into the pool. Test 1 1. This is the sort of English that I cannot put up with. OR Test 2 1. e 2. a 3. c 4. d The correct idiom is ‘light at the end of the tunnel’, which means seeing some hope after period of despair. ‘Step down’ means to resign from a high post or to reduce, especially in stages. Therefore it is correct contextually. Other options are incorrect. ‘Step aside’ means “to resign from a post, especially when being replaced.” ‘Chipped away at’ is the right way of writing. It means to reduce or make progress on something incrementally. ‘Chipped in’ means too contribute money or labor and doing that to ‘that legislation’ is somewhat illogical. ‘With a straight face’ is the right way of writing, which means ‘to do or say something confidently without a trace of guilt’. Explanations: Fundamentals of Grammar 10. a ‘Faced down’ is correct because it means to confront boldly or intimidate. Test 4 1. e It means ‘Criticism’. 2. a ‘keeping an eye’ is the correct idiom. 3. b ‘Let loose’ means without any restriction (free). 4. e ‘Shell-shocked’ is the correct idiom which means ‘stunned’. MBA Test Prep Page: 139 5. c ‘At a low ebb’ is the correct expression which means a bad state. 6. e ‘Wrapping up’ means to complete or stop doing something. 7. d ‘Middle of the road’ means having a balanced approach, between the two extremes. 8. a ‘Horse-trading’ takes place in parliament when members change sides. 9. c ‘Lame duck’ means a person or company that is in trouble and needs help. 4. b It means ‘to make an effort to understand and deal with a problem or situation’. come/get off your high horse- to stop talking as if you were better or more clever than other people. come/go along for the ride- to join in an activity without playing an important part in it. come/go cap in hand- to ask someone for money or help in a way which makes you feel ashamed. come/go down in the world- to have less money and a worse social position than you had before. 5. d Cut the Gordian knot means to deal with a difficult problem in a strong, simple and effective way. cut some slack- to allow someone to do something that is not usually allowed, or to treat someone less severely than is usual. cut the (umbilical) cord- to stop needing someone else to look after you and start acting independently cut the ground from under- to make someone or their ideas seem less good, especially by doing something before them or better than them. Cut the crap!- an impolite way of telling someone to stop saying things that are not true or not important. 6. b It implies something that you say which means that failing to do something when you almost succeeded is no better than failing very badly. miss a trick- to not fail to notice and use a good opportunity. miss out- to fail to use or enjoy an opportunity miss the boat- to be too late to get something that you wantmiss the point- to fail to understand what is important about something 7. a still and all- despite that. still waters run deepsomething that you say which means people who say very little often have very interesting and complicated personalities. sting in the tail- an unpleasant end to something that began pleasantly, especially a story or suggestion stink up- to make a place smell unpleasant; to do something very badly. stitch in time- something that you say which means it is better to deal with a problem early before it gets too bad 8. e Sticks and stones may break my bones (but words will never hurt me)- something that you say which means that people cannot hurt you with bad things they say or write about you. sink like a stone- to fail completely leave no stone unturned- to do everything that you can in order to achieve something or to find someone or something. set in stone- firmly established and very difficult to change get blood from a stone- to do something very difficult 9. a hang the cost/expense - if you say that you will do or have something and hang the cost, you mean that you will spend whatever is necessary. at all costs -if something must be done or avoided at 10. d ‘Out in the cold’ means exposed and vulnerable. In option A, ‘cord’ is the correct word and not ‘cords’. Test 5 1. a In A-Correct usage is 'in' business and in D it should be 'in' the share market values. 2. e A is incorrect, should be 'In' January, B is incorrect should be was found 'in' a street, C is incorrect should be 'in' his family's care. 3. d B is incorrect should be 'think of…' C is incorrect should be 'in a thing…' 4. c C is incorrect, should be 'outcome of…' , D is incorrect, should be 'us into'. 5. e A is incorrect should be 'in your neighbourhood…' and B is incorrect should be 'across the globe…'. 6. c A is incorrect, use the preposition 'by' instead of from, D is incorrect, use the preposition 'for' instead of through. 7. b B is incorrect, use 'of the health' instead of 'with…', C is incorrect use 'in time…' instead of 'from time…' 8. a A is incorrect, should be 'ultimate form of….', D is incorrect should be ' appeal as strongly to …' 9. c A is incorrect should be 'moved by', D is incorrect should be 'through this…" 10. d A is incorrect, should be 'of' instead of 'from', C is incorrect, should be 'in earthquakes…'. Test 6 1. e It should be ‘blink of ..’. 2. e It should be winning streak. 3. a It should be ‘pigeonholed..’. Page: 140 MBA Test Prep Solution Book-2 all costs, it must be done or avoided whatever happens. cost (someone) a pretty penny -to be very expensive. cost (someone) an arm and a leg (informal) to be very expensive. count the cost - to start to understand how badly something has affected you 10. b copper-bottomed- a copper-bottomed plan, agreement, or financial arrangement is completely safe. cordon bleu- cordon bleu cooking is food which is prepared to the highest standard and a cordon bleu cook is someone who cooks to a very high standard corner the market- to become so successful at selling or making a particular product that almost no one else sells or makes it. cotton-picking- something that you say before a noun to express anger corridors of power- the highest level of government where the most important decisions are made. Chapter 15 6. Either accept our conditions or leave. 7. We rested until the storm was over and we felt better. (until is an adverb) Test 1 1. c Rather than 'in lieu of', the appropriate conjunction will be 'because', since the sentence is talking about the reason that why 'pipeline for public offerings has dried up.' 2. a In A the correct conjunction should be 'however' rather than 'wherever' because the latter doesn't make any sense in the given context. 3. b 'Moreover' is an incorrect conjunction out here because the sentence talks about a contradiction. Therefore 'but ' should replace 'moreover'. 4. c 'Due to' should be replaced by 'since' because the latter gives the meaning that 'due to the tiny town', which has no meaning. 5. a The word 'for' is most often used as a preposition, of course, but it does serve, on rare occasions, as a coordinating conjunction. Other options change the meaning of their respective sentence. 6. e The correlative conjunction 'not only' is always coupled with 'but also'. 7. d The appropriate conjunction will be 'as long as..' 8. c In A 'as' is following 'though' which is incorrect. In B 'like' has been incorrectly used. 'Like' as a conjunction means 'as if', therefore doesn't make sense here. Similarly in D, 'though' and 'yet' are used together thus rendering the sentence incorrect. Same problem is with E. 9. a Other options change the meanings of their respective sentences. Exercise 1 1. and - joining Akbar/I 2. but - joining slow/strong 3. or - joining Jatin/Lalit 4. nor - joining like/appreciate 5. or - joining You/I Exercise 2 1. neither-nor 2. not only-but also 3. either-or 4. whether-or 5. both-and Exercise 3 10. e Other options change the meanings of their respective sentences. 1. Cox and Kings is open today so we’re going to buy our tickets to Australia. (so is an adverb) 2. As he read the letter he laughed. (There is no conjunction As is an adverb: He laughed as he read the letter) 1. c Instead of 'neither' it should be 'nor'. 2. d The correct conjunction will be 'otherwise'. 3. So he told me but I didn’t believe him. 3. a The appropriate conjunction will be 'despite'. With 'yet', 'its size' doesn't make any sense. 4. She did not reply, nor did she make any gesture. 4. d 5. We ran from the building when we noticed the time. (when is an adverb) In this part the conjunction 'nor' is incorrect. In this case with 'whether' 'or' should be coupled. Explanations: Fundamentals of Grammar Test 2 MBA Test Prep Page: 141 5. d Here 'such' should be followed by 'as' or else it does not make any sense. 6. a Rather than alongside the conjunction 'along with' will be more appropriate. 7. a Rather than 'even', the appropriate conjunction here will be 'although'. 8. e 'So' is an incorrect adverb here. It should be replaced by 'like'. 9. b 'Wherever' doesn't make any sense here. It should be replaced by 'however'. Unit – 7 Chapter 16 Test – 1 1 I like hiking, skiing, and snowboarding. (A verb has been mentioned with gerunds. So, ‘to snowboard’ should be changed to ‘snowboarding’.) 2. A low-fat diet, most experts agree, is the best way to reduce artery blockage and achieve weight loss. (The writer describes the second benefit of a low-fat diet as “the achievement of weight loss”—a noun phrase. Clearly, following the first infinitival phrase with another to create “to reduce artery blockage and to achieve weight loss” improves clarity.) 3. When one takes the CAT, it’s perfectly natural to be a little nervous, irritable and sweaty. (“Nervous” and “irritable” are adjectives. “Sweaty” is also an adjective, but, because it is followed by “palms,” the third element is a noun phrase modified by an adjective. Parallelism could be achieved simply by removing “palms.”) 4. Eating huge meals, snacking between meals, and exercising too little can lead to obesity. A noun here has been used with gerunds. So, exercise should be changed to exercising.) 5. Our coach is a former champion batsman, but now he is overpaid, overweight, and over forty. (The fact of being a champion batsman cannot be mentioned in the same breath with negative attributes like overpaid, obese and over fifty.) 6. Mustaine likes people who have integrity and character.(The verb ‘have’ should modify both nouns.) 7. I like editing books more than just reading them. (Here, an infinitive is paired with a gerund. A gerund should be paired with a gerund.) 8. Career Launcher needs teachers who are ambitious, self-motivated, and dedicated. (Here, a verb form has been given with one adjective and one noun. The parallelism can be achieved by converting all three words into adjectives) 9. As an artist, he drew, painted, and sculpted. (For the parallelism to be there, all three activities should be mentioned as verbs.) 10. Aggression and melancholy are behaviours that many steroid-users exhibit. (In this sentence the parallel elements should both be nouns that function as the subject of the sentence. So they have to be in the same grammatical form.) 10. c The appropriate conjunction here will be 'as much as'. Test 3 1. b In A 'like' is an inappropriate adverb. It should be replaced by 'as'. In D rather than 'therefore', 'and' will be more appropriate. 2. b In B, 'henceforth' will be replaced by 'while', since the sentence is talking about the ongoing mood of pessimism. 3. b In C, rather than 'then', 'than' should be used because comparison is being made here. In E 'unless' is inappropriate because it means 'except', whereas the sentence talks about 'till'. Therefore 'until' will be appropriate. 4. c In E, 'even' should be followed by 'before' or 'when', rather than 'if'. 5. d In C 'though even' is incorrect. Either it should be 'though' or 'even though'. In D 'while' is inappropriate. It should be replaced by 'and'. Usage notes: Among some conservatives there is a traditional objection to the use of though in place of although as a conjunction. However, the latter (earlier all though) was originally an emphatic form of the former, and there is nothing in contemporary English usage to justify such a distinction. 6. d In E, rather than 'nor' it should be 'or'. 7. e In B, 'despite' is incorrect because it is always followed by 'of'. It should be replaced by 'instead'. In E, rather than 'or' should be 'nor' because the previous sentence has already discussed something in negative therefore 'nor' can appropriately follow it; since the very first sentence says 'But this is not it'. 8. b In D 'whether' is inappropriate and should be replaced by 'if'. Page: 142 MBA Test Prep Solution Book-2 Test 2 1. e There is no error in the statement. The first part of the sentence before hyphen (-) is the main clause and rest are the ‘elements’ (subordinate clauses) that the main clause includes. All of them are parallel- ‘balance’, ‘age’ and ‘wearing’. ‘Wearing’ is correct because it is a noun (gerund) here and not a verb. 2. b Following the principles of parallelism, ‘to keep’ should be match by ‘to supply’. 3. c Again according to the rules of parallelism, in a string of nouns should have all singulars or all plurals. Since we have ‘whispers’, ‘tingles’ and ‘shocks’; ‘shout’ should also be in plural. 4. d Instead of using an infinitive ‘to herd’; a gerund should be used that is ‘herding’; because there is a string of gerunds used in the sentence, which are ‘singing’, ‘irritating’ and ‘barking’. Secondly ‘at’ cannot be followed by ‘to’. 5. d It should be ‘selling’ and not ‘to sell’. 6. d To make the structure parallel ‘who will benefit’ should be matched with ‘to what extent’ because ‘and the extent’ does not have the component of questioning though the context requires that. 7. d. In D rather than ‘consolidating’ it should be ‘consolidate’ keeping the parallel structure in view because the verbs should match when in a string. 8. e 9. b Both should be followed by ‘manufacturers and dealers’ to keep the parallel structure at its best. All the verbs in the sentence are in simple past; thus it doesn’t make any sense to use past continuous in the middle of the sentence. (Opossums- a prehensile-tailed marsupial, of the eastern U.S., the female having an abdominal pouch in which its young are carried: noted for the habit of feigning death when in danger.) 10. e ‘Running’ is a gerund here whereas the requirement is that of an infinitive to match ‘to have’ and ‘to buy’. 3. b In C ‘to accept’ is incorrect. It should be replaced by ‘accepting’ 4. a In E ‘the crucible …’ should be preceded by ‘as’, so that the parallel structure of the statement is intact. The correlative ideas here should be joined by ‘either as the …or as the’. 5. b This tradition began in ancient Rome and continues into modern times. (‘Begun’ is a participial adjective while ‘continues’ is an active verb.) 6. b I acquired my wealth by investing carefully, working hard and finding a rich father-in-law. (Here, a noun has been mixed with a pair of gerunds.) 7. e We can either drive to Shimla or fly to Chennai. (Correlative conjunctions, here ‘either . . . or’, introduce clauses that must be parallel.) 8. c I do not like hot water or cold milk. (Here, unequal structures are being used to explain equal ideas. Converting ‘milk that is cold’ to ‘cold milk’ rectifies the problem.) 9. e Please write a long story, a short novel and an epic poem. (Again, unequal structures are being used to explain equal ideas. Converting ‘a poem that is epic’ to ‘an epic poem’ rectifies the problem.). Chapter 17 Exercise 1 1. Most graciously, 2. Dear Madam: (a business letter) 3. B-52, Okhla Industrial Estate, New Delhi-110020? 4. 1 March, 2006, at 5. (no comma needed - only one part) 6. (no comma needed - only one part) 7. Charu Gera, Sr., South Extension, New Delhi 8. New Delhi, India Exercise 2 Test 3 1. a 2. d In C, rather than ‘running’ it should be ‘runs’ since the context talks about universal truth thus has to be simple present tense following the rules of parallelism. Similarly in D, instead of ‘cleared’ it should be ‘to clear’. In B ‘will have’ is incorrect because it is not parallel with ‘has launched’ and ‘have fought’. Since the frame of reference pertains to present perfect, the insertion of ‘will have’ is totally out of context here. Explanations: Fundamentals of Grammar 1. Football, basketball, track, and tennis require running. 2. The numbers 8, 16, 32, and 48 are called even numbers. 3. Eat, drink, and make merry, for you will soon die. 4. I like shopping, my friend likes dining, and the family likes activities. MBA Test Prep Page: 143 5. Working hard, saving some money, and providing for a family should be important for a father. 6. I saw him run up the mountain, jump off the cliff, and land in a pine tree. 7. He was from Mahableswar and she was from Kanyakumari. 8. She likes to sing, to play the piano, and to read novels. 9. The search party looked along the road, up the hill, and down the alleys for clues. Exercise 5 1. Chandan is tall; his brother is short. 2. He knocked several times; no one came to the door. 3. The siren blew loudly; I rushed to the window; the police raced past as I looked out. 4. I waited several hours for you; you did not return; I became concerned. 5. My sister loves mysteries; my brother likes technical manuals. Exercise 3 Exercise 6 1. Ila, indeed, is a good mother. 2. I hope, Vishal, that you don’t go to jail. 3. My son-in-law Salim will be able to vote in the coming election. (a closely related appositive or use commas around Salim if you thought it was a noun of address) My son-in-law, Salim, will be able to vote in the coming election. 4. Oh, Girish, I hope that you, on the other hand, will be happy with your decision, your move to Europe. 5. We sat in the shade beneath a broad green tree, Lara. 6. It was a lovely, happy, memorable time. 7. I know, after all, you will be successful. 8. 1. Since you asked my opinion, I will tell you; and I hope you will listen well. 2. Although he is highly qualified, he is not dependable; and I am afraid to hire him. 3. Because Preeti is absent a great deal, she has a hard time keeping up; but she is willing to work overtime. 4. Although I prefer English, I know that math is important; and I will work hard in both classes. 5. When you arrive on the train, take a taxi to the metro station; or I can meet you at the platform. Exercise 7 1. I am looking for the poem “The Path Not Taken”; I need it tomorrow. Mr. Dinesh Bobby, the boy next door, has been fighting with your brother Harish. (Harish is a closely related appositive) 2. Ram sings bass; Hari, tenor. 3. I have visited U.S.A., Australia, Canada and Bhutan. 9. Of course, we could hear immediately that you, after all, will be going to Santos, a great city in Brazil. 4. I will steal, cheat, and lie for you; but I will not kill for you. 10. Well, Vishal, I hope to see you, by the way, in Goa on our return from our vacation, a trip to Mumbai. 5. There was a sudden noise; everything stopped immediately. 6. Although we may need more time, I believe we will be victorious; and I believe you feel that way, too. 7. We can trust him implicitly; nevertheless, we should not be careless. Exercise 4 1. “I wish the election was over,” said Farah. 2. “Will they finish this week?” asked Farheen. 3. Avinash added, “It is becoming a joke!” 8. The house looked like what we wanted; on the other hand, we had not been inside. 4. “We can now see that every vote counts,” concluded Nidhi. 9. “Yes, we know that we should vote every time,” commented Rajiv. I had food, clothing, and furniture; but I didn’t have my family. 10. He was such a “bore”; I couldn’t stand him. 5. Page: 144 MBA Test Prep Solution Book-2 Exercise 8 5. The cow’s udder was cut from jumping the neighbour’s fence. 6. Rahul and Sanjeev’s store will be open on Diwali 7. Everybody else’s help will be appreciated by my mother’s family. 1. The boy’s bike is in the back yard. 2. Gaurav’s car was in the accident yesterday. 3. Mr. Gupta’s talk was the best yet. 4. What happened to that horse’s leg? 8. Just two days’ work will finish this room. 5. That woman’s umbrella is blowing away in the wind. 9. Anika’s and Rani’s costumes were the prettiest of everyone’s. 10. The women’s and girls’ ages were revealed to everyone. (could be girl’s) Exercise 9 1. These women’s hats are sold in this store. 2. The children’s party was a great success. Exercise 12 3. The mice’s tracks were everywhere in the dust. 1. we’re it’s you’ve who’s hasn’t 4. We followed the two deer’s tracks in the snow. 2. I’ll I’m she’ll she’ll I’ll 5. The geese’s flight was smooth and graceful. 3. I’ve we’ll they’re aren’t didn’t 4. he’s you’ll you’re isn’t hadn’t Exercise 10 5. wasn’t haven’t couldn’t we’d they’ll The men’s and boys’ boots were all mixed together. (separate ownership) 6. shouldn’t doesn’t there’s they’ve you’d 2. Shashi’s mother lives next door to us. 7. weren’t wouldn’t that’s I’d won’t 3. The dog’s growl scared the baby in the neighbor’s yard. 4. Both Vinod’s and Shyam’s hair is red. (separate ownership) 5. Meera’s and Seeta’s mother came to the performance. (joint ownership) 1. 6. The babies and the children’s fun ended with the parents’ return. (joint ownership) 7. The men’s hoods covered their faces. 8. The coop was covered with several chickens’ feathers. 9. I could hardly hear the puppy’s bark. 10. The wolves’ howls came sharply to the deer’s ears. Do not confuse the contractions (it’s, who’s, they’re, you’re) with the possessive pronouns (its, whose, their, your). Exercise 13 1. It’s about time you started looking for your shoes. 2. They’re coming at about nine for their children. 3. Its mouth was sore because it’s chewing all the time. 4. Whose briefcase will you be using for your papers? 5. You’re going to be late, but who’s going to be on time? Exercise 14 1. It used to be that one had to be twenty-one to vote. 2. When adding thirty-four and forty-two, you get seventy-six. 3. One hundred thirty-seven people were killed in that crash. 4. The sixty-fourth running of that race was cancelled due to weather. 5. Many more privileges come to people who are sixtyfive or older Exercise 11 1. Could I buy fifty rupees’ worth of sweets for the kids? 2. Somebody’s shoes have been left in the living room. 3. His shoes are here, but where are yours? 4. His aunt’s nephew will be on television with Ahuja’s group. Explanations: Fundamentals of Grammar MBA Test Prep Page: 145 Exercise 15 1. We will invite Sushma—she is the new girl next door— to our party. 2. The dog slid on the vinyl—his nails acting like skates— and crashed into the trash can. 3. When our stockpile was sold—indeed, dumped for surplus—all our sales were compromised. 4. Today has been—but I will not bore you with my troubles. Let me tell you about—watch where you are going! 5. Exercise 16 1. We fished (or should I say mashed worms) in the murky pond. 10. Correct. or colours; Test 2 1. “How,” I asked, “can you always be so forgetful?” 2. The girl who is standing there is his fiancée. 3. Correct. 4. Finish your job; it is imperative that you do. 5. You may, of course, call us anytime you wish. 6. You signed the contract; consequently you must provide us with the raw materials. 7. “Stop it!” I said. “Don’t ever do that again.” 2. They listened to the teacher’s stories (they were very dull) which gave some background for the book. 8. Because of his embezzling, the company went bankrupt. 3. Gauri and Prial (you remember them) moved to a new house last week. 9. Correct. 10. 4. Even though he was not qualified (according to his transcripts), he knew more than most of the others. Nature lovers will appreciate seeing whales, sea lions, and pelicans. 5. Another possibility (the possibilities seem endless) was suggested by a person at the back of the room. Test 3 1. c Uses the correct punctuation marks for distinction. 2. a Put a colon to introduce a list. 3. d Only commas are required to separate the item list, the colon is incorrect here; do not use apostrophes for sweets, marshmallows and toffees. Test 1 1. The girl’s (girls’) vitality and humor were infectious. 2. New clients’ accounts showed an 11 percent increase in sales. 3. These M.D.s’ credentials are excellent. (These M.D.s – plural) 4. e Semicolon is used to separate sentences which are closely connected in thought. 4. Several M.D.s (Or M.D.’s) agreed that one bacterial strain caused many of the symptoms. 5. b No interrogation mark is used after an indirect question, commas are not required. 5. You asked for forgiveness; he granted it to you. 6. e 6. We ask, therefore, that you keep this matter confidential. Use comma to separate co-ordinate clause in a compound sentence. 7. b The order was requested six weeks ago; therefore I expected the shipment to arrive by now. Use comma after adverbial phrases of absolute construction. 8. c I need a few items at the store: tissues, a bottle opener, and some milk. Use comma to separate words/ phrases/clause inserted into the body of a sentence. 9. d Use inverted commas for reported speech and put commas for short pauses. 7. 8. 9. I needed only three cards to win, namely, the ten of hearts, the jack of diamonds, and the king of hearts. (or win; namely,) Page: 146 10. b Use commas to mark off phrases in apposition. MBA Test Prep Solution Book-2 Unit – 8 3. c B is incorrect, the correct phrase is 'akin to' , akins as word does not exist, C is incorrect, correct usage is 'of' after the word silhouette. 4. b A is incorrect should be 'stop' instead of stops, error of subject verb agreement. B is incorrect; the correct phrase is day-to- day activities. Chapter 18 Test 1 1. d Options B, D & E are correct. A is incorrect as the correct phrase is 'as much as' , C is incorrect as the correct verb to be used is 'there are..'. 5. c 2. a Only A& B are correct, C is incorrect because it should be 'recent months', D is incorrect because it should be 'at the moment', E is incorrect because it should be 'pants'. A is incorrect should be 'about' instead of 'over', C is incorrect should be 'the' public not 'a' public. E is incorrect uses a comparative 'greater', correct usage is 'great'. 6. c B is incorrect, use 'dreamt ' instead of 'dream' in keeping with the tense of the sentence, C is incorrect uses 'which' instead of 'that', which is mostly used for inanimate objects. 7. d A is incorrect should be 'resonant' in the adjective form rather than 'resonance' in the noun form, C is incorrect, use 'have' after Patriarchs as the correct form of subject verb agreement. 3. d C is incorrect because the correct usage is 'their', E is incorrect an article should precede the word leader, should be 'a leader'. 4. d C uses incorrect parallelism, should be 'transporting', E use the article 'the' before China which is incorrect. 5. b A is incorrect should be 'Greeks 'since it's addressing the entire race, D is incorrect, the verb should be 'seek', E is incorrect because it should be 'than' not then. 8. c A is incorrect, incorrect usage of the preposition 'upon' which means on something, something elevated, correct usage is 'on'. B should be 'scores' instead of 'score', should agree with plural verb. 6. a A is incorrect, should be 'built', error of tense, C is incorrect should be 'on Monday' as the deed is already done. 9. a D is incorrect, requires an article before 'user's' , E is incorrect due to the incorrect usage of the phrase 'real time'. 7. e A is incorrect should be 'in business', C is incorrect due to incorrect subject verb agreement, and use 'has 'here. 10. e C is incorrect, should be the contraction 'it's' instead of 'its', D is incorrect, uses the verb 'is' instead of 'are'. 8. e B has incorrect usage of the pronoun should be 'in' instead of 'at', C is incorrect should be 'candidates instead of the singular 'candidate'- should agree with the verb, E is incorrect because the collective usage of 'skills; is being discussed here and should agree with the verb 'are'. 9.c C is incorrect, should be 'a good ...' the defining article is missing, D is incorrect should be 'friends' in the plural. 10. a B is incorrect, should be 'setting' instead of set: error of parallelism, D is incorrect should be 'said' instead of 'says' incorrect tense usage, E is incorrect because the article 'a' is missing before the word negative. Test 2 1. b A is incorrect, use the adverb form 'increasingly' here, in sentence B the correct usage is 'eight times' and in E the correct form is 'it's' instead of the possessive form 'its'. 2. a A is incorrect, use 'between' instead of 'among', D is incorrect, use the article 'the' before the word 'plant'. Explanations: Fundamentals of Grammar Chapter 19 1. Eating insects does far less damage. For one thing, the habit could help to protect crops. Some 30 years ago the Thai government, struggling to contain a plague of locusts with pesticides, began encouraging its citizens to collect and eat the insects. Officials even distributed recipes for cooking them. Locusts were not commonly eaten at the time, but they have since become popular. Today some farmers plant corn just to attract them. Stir-frying other menaces could help reduce the use of pesticides. But insect populations vary with the seasons, and it is hard to control the amount on offer at a given time. “There is very little knowledge or appreciation of the potential for managing and harvesting insects sustainably,” notes Patrick Durst, a Bangkok-based senior forestry officer at the FAO. Those looking for a reliable source of protein might prefer to farm them. Protein makes up a high proportion of most insects’ weight. That makes them much more efficient at MBA Test Prep Page: 147 converting feed to protein than livestock. For example, a cow yields only 10lb (4.5kg) of beef for every 100lb of feed it eats, whereas the same amount of feed would produce tens times as much cricket. 2. members. Some (or many) existing members have been setting a bad example for them. Nor was the fifth enlargement a simple matter of countries governed by former dissidents accepting the democratic embrace of the West. Plenty (or most or many) of ex-communists smoothly relabelled themselves and hung on to power across the block. Brussels is full of talk about “backsliding” to describe the way that politicians in the new member countries forgot, or actively undermined, reforms that the EU demanded during accession negotiations. Corruption and organised crime blight many of the newcomers. Parliaments and ministerial suites shelter too many bad men. While IT managers are understandably leery about recommending the iPhone for company employees, individual owners are going to be sneaking their prized possession into the office to access e-mail and corporate information. Bosses, too, who become enamoured of the iPhone will be insisting the firm support it. Against their better judgment, many IT managers will doubtless comply. No question, Apple’s innovative design has sent a wake-up call to smartphone makers everywhere. From tiny Palm, the struggling smartphone pioneer, to Research In Motion (RIM), the BlackBerry manufacturer, and the dozen or so handset makers who build smartphones based on Microsoft’s Windows Mobile, as well as the 800-pound gorilla of the business, Nokia—all have been forced to react to the iPhone’s touch screen, intuitive software and iconic appeal, not to mention its rapid penetration of the market. All this has led some to suggest that enlargement happened too soon, and that many of these problems could have been avoided by waiting until the accession countries were better prepared. This report will argue the opposite: that enlargement came in the nick of time. Inside the candidate countries the first victims of further delay would have been reformers who for years had been pushing painful changes as vital for achieving EU membership. Had the public started to doubt that entry was fairly imminent, the drive for reforms would have been undermined. Apple has sold more than 5m iPhones over the past year. Practically all have been bought by individuals at their own expense, rather than their employers’. The iPhone now accounts for 19% of the smartphone market in the United States. That’s ahead of Palm’s 13%, but still well behind RIM’s 45%. Over the past few months, competing 3G smartphones with touch screens and a host of features have been coming thick and fast. Sprint has started to offer Samsung’s Instinct, which seeks to trump the iPhone with a higher download speed, better video, picture messaging, navigation and applications, plus a battery that can be removed. 3. For the existing member countries, three big reasons would have made enlargement far more difficult if it had come any later than it did. These can be summarised as migration, money and Moscow. 4. The banks’ problems feed back into the credit markets. For if the banks cannot raise as much capital as they need, they may feel they have to sell assets. And that may mean off-loading corporate and asset-backed bonds, even if the banks must accept fire-sale prices. The ABX index of asset-backed bonds is below the level that it sank to in March. In truth, EU firms have been investing heavily in central and eastern Europe since soon after the Berlin Wall came down, and Italy was home to about 350,000 Romanian migrants before Romania joined the union. Yet public fears about Polish plumbers and other bogeymen are real enough. Even though German exporters have flourished by selling to the new member states, 63% of Germans, according to Eurobarometer, think that enlargement is making Europe as a whole less prosperous. Analysts tend to agree that spreads are much higher than are needed to compensate investors for the likely default rate. According to Moody’s, a ratings agency, the default rate over the past 12 months on global speculative, or junk, bonds was just 2%; the only company outside America to walk away from its debt was a Bulgarian steelmaker. Jim Reid, a credit strategist at Deutsche Bank, quips that, if the spreads on investment-grade debt accurately reflect default rates, everyone will end up living in caves. Some (0r most) of the newcomers have not helped their cause since joining. Nasty populists have done well in elections in several countries, and Romania, Bulgaria, Slovakia and the Czech Republic have shown prejudice against the Roma too. But then prejudice, bad government, corruption and organised crime are not the exclusive preserve of the new But spreads often fail to reflect the likelihood of defaults. This was true during the credit boom, when they fell to historically low levels, as investors chased anything with a yield higher than government bonds. And it is true now because, although an investor could make money by buying corporate bonds and Page: 148 MBA Test Prep Solution Book-2 1776. He found Hume in good cheer, pressed him on life after death but could not shake the great man of his disbelief. The thought of annihilation at death, Hume told Boswell, caused him no more uneasiness than the thought of non-existence before birth. holding them for five years, few are able to follow such a strategy. The big risk investors face is buying too soon. Traditional bond managers fear losing their clients if they have another poor quarter. Hedge-fund managers may find that the prime brokers who lend them money will cut off their funding if their positions deteriorate. Making a success out of such material requires style and wit. Simon Critchley’s “The Book of Dead Philosophers” shows leaden playfulness. His tone can be portentous or giggly, as if he is unsure who he is writing for or why. The reader will “die laughing”, he says. But most of the deaths he describes on his padded-out list of 190 philosophers are banal, and virtually none is funny. The real trouble is that Mr Critchley, a professor at the New School of Social Research in New York, cannot decide whether he is writing about philosophers’ own deaths, exemplary deaths or philosophers’ thoughts on death in general. Just like last summer, the risk is of a downward spiral, in which nervous investors sell bonds, driving prices down, only increasing the general nervousness. The system is still deleveraging, at least according to the latest figures from America—which show bank loans contracting at an annualised rate of 8% over the 13 weeks to June 25th. To make matters worse, the fundamentals for the corporate-bond market have worsened since last August. Expectations for corporate profits are being revised down, as margins come under pressure from slower growth and higher commodity prices. Headline inflation is well above target, so central banks are unlikely to ride to the rescue with interest-rate cuts; indeed, many prefer to tighten monetary policy. This is one sequel where the scriptwriters are going to have to work extra hard to come up with a happy ending. 5. IT came to Seneca by his own hand in a warm bath after falling foul of the emperor Nero, to Boethius by order of the Ostrogothic King Theodoric and to Giordano Bruno for affronting the Inquisition. Pneumonia saw off both Francis Bacon when he tried to deep-freeze a chicken with snow and René Descartes on having to rise before dawn in a Swedish winter to instruct Queen Christina. In modern times, Nietzsche’s friend Paul Rée fell off an Alp and Moritz Schlick, a logical positivist, was shot to death by a deranged student. An eye-catching list perhaps. But quite unrepresentative. Save when at work making distinctions and arguing relentlessly, philosophers are much like everyone else. They lead on the whole worthwhile but humdrum lives. Most die of old age in their beds. Philosophers’ deaths make an unpromising theme for a book. A handful of philosophers, it is true, have died philosophically, by which people usually mean they have figured in memorable accounts of exemplary ends. Socrates was one. Unjustly condemned, he could have left Athens for exile. But, as Plato recounts, he chose city over self, virtue over expediency, and drank the hemlock, surrounded by friends. Another philosopher who had a philosophical end in this sense was David Hume. James Boswell, a famous diarist, visited him as he was dying in Edinburgh in Explanations: Fundamentals of Grammar 6. All of these aspects of Noguchi’s career will be explored in an exhibition opening Friday at the Yorkshire Sculpture Park, near Wakefield in England. The stars of the show are a hundred or so of the paper and bamboo Akari light sculptures that he began making in the early 1950s, and that became his bestknown work. Lovely to look at and surprisingly robust, the Akari lights not only fuse Noguchi’s Japanese and American influences, but art and design, craftsmanship and industry. They were also the catalyst for economic regeneration of a declining Japanese industry and, last but not least, their dramatically shaped mulberry-bark paper shades emit a very beautiful light. The Akari project came about by chance, after Noguchi went back to Japan in 1950. By then his father was dead, and the Japanese welcomed him as a famous American artist. He visited the city of Gifu, where the traditional candlelit paper lantern industry was declining dramatically as more and more Japanese homes introduced electricity. The mayor asked Noguchi how to revive it. Noguchi’s solution was to modernize the old paper lanterns. Settling in an ancient teahouse with his thenwife, the Japanese movie star Yoshiko Yamaguchi, he designed a series of lamps powered by electricity, rather than candles. For the shades, he used the silky Mino paper that had been made in a nearby village from locally grown mulberry bark since the eighth century, but replaced the recently adopted wire frames with traditional bamboo. The design process was traditional, too. Noguchi began by making a wooden mold in the shape of the finished shade and wound fine strands of bamboo around it. Strips of Mino paper were glued to the bamboo, and the mold removed once the glue had dried. A slender metal structure was designed to hold the bulb and support the shade, both at the top and the bottom, where it seemed to float above the floor on spindly legs. MBA Test Prep Page: 149 7. Like Tempelhof, the Beijing air terminal boasts a sweeping concourse that evokes the glamour of air travel while enclosing a surprisingly intimate interior. But Foster pushes the ideal of mobility to a new extreme. Guided by twinkling lights embedded in the terminal’s ceiling, arriving visitors glide up ramped floors and across broad pedestrian bridges before spilling out onto the elevated concourse. From there they can disperse along a fluid network of roads, trains, subways, canals and parks whose tentacles extend out through the region. 9. “This is the big issue we had to deal with all the time,” he says. “The danger is that by collaborating with the subjects on the painful retrieval of these memories, you somehow make them say things. So, yes, there’s always a risk of something. This sprawling web has completely reshaped Beijing since the city was awarded the Olympic Games seven years ago. It is impossible not to think of the enormous public works projects built in the United States at midcentury, when faith in technology’s promise seemed boundless. Who would have guessed then that this faith would crumble for Americans, paving the way for a post-Katrina New Orleans just as the dream was being reborn in 21st-century China at 10 times the scale? Yet your sense of marvel at China’s transformation is easily deflated on the drive from the airport. A banal landscape of ugly new towers flanks both sides. Many of those towers are sealed off in gated compounds, a reflection of the widening disparity between affluent and poor. Although most of them were built in the run-up to the Olympics, the poor quality of construction makes them look decrepit and decades old. 8. There may be recession closing in around the world, but nobody has told the major soccer clubs. This may be the “closed season” of big European leagues, but the players, the teams, the organizers are trading at full pelt. Any moment now, Ronaldinho, the wonderfully gifted Brazilian who has obliged Barcelona to sell him because he has frequented parties more than he has played games over the past season, will get the transfer his agent brother has negotiated. When the bartering stops, AC Milan will announce its acquisition of Ronaldinho for a “cut price” •15 million, or $23.9 million. It will grant his wish to play in the Olympics, and then will start paying his $10 million a season salary. Milan will build its attacks through the much coveted Brazilian trio of Kaká, Ronaldinho and the teenager Pato. That’s show business. That’s where soccer has taken over from Hollywood as the industry that trades the talents of men for the length of contract bartered between them. To paraphrase Sam.uel Goldwyn, the owners sometimes pay too much but consider the players worth it. Page: 150 Some might say, of course, that when history loses its focus on the representation of straight facts, it ceases to be history and becomes historical fiction. Empathy, after all, is hardly a neutral subject in history circles, and has long been held responsible in some quarters for the dumbing-down of school history teaching. Is Figes not wary of inviting such charges to be brought to his own work? Does not the active engagement of his researchers and himself in the generation of the sources compromise their findings? “But you’ve got to remember that, without some kind of empathy, the retrieval process wouldn’t take place at all. The important thing is to prepare the ground so that the subjects can tell their stories in their own words. Those interviews, in several cases, lasted days, with many pauses where the discussion became too painful for the subject. But it’s only when you start discussing the content of the interview afterwards that you come to put your own intentions on the material. 10. I wasn’t expecting such a tender kiss on our first date, but April was determined. She had already signalled her intentions by swimming past me in the pool, sliding her smooth skin next to mine, flipping over to give me a tantalising glimpse of her white tummy. But I knew better than to be flattered. It wasn’t me she was interested in but the dead sardine in my hand. She nuzzled my cheek, her chin as slippery as a wet mushroom, then reared effortlessly out of the water. I slipped the fish past her rows of peg-like teeth and watched her plunge back beneath the surface... April, if you were in any doubt, is not a mermaid but a fouryear-old dolphin. She lives in a marine park called Xcaret on Mexico’s Caribbean coast and swimming with her and her friends in a large sea-pen is just one of the many grin-provoking magical experiences on offer within the wild haven of the park. Xcaret, pronounced Escaray, is a sustainable tourist development set up in 1990 on a picturesque inlet about 45 miles south of Cancún. Its far-reaching aim was to conserve both the flora and fauna of the area - some 4,000 species can be found on the 80-hectare site - and to promote the area’s colourful culture and Mayan heritage. All of which makes for a bizarre hybrid of serious conservation zone and thrill-seekers’ paradise. It’s what might happen if Mickey Mouse suddenly became an eco-warrior. MBA Test Prep Solution Book-2